Download 4176 metodological instructions - Сумський державний університет

Document related concepts

Hypothyroidism wikipedia , lookup

Metabolic syndrome wikipedia , lookup

Hypoglycemia wikipedia , lookup

Hyperthyroidism wikipedia , lookup

Graves' disease wikipedia , lookup

Epidemiology of metabolic syndrome wikipedia , lookup

Diabetes mellitus type 1 wikipedia , lookup

Epigenetics of diabetes Type 2 wikipedia , lookup

Diabetes mellitus wikipedia , lookup

Diabetes management wikipedia , lookup

Diabetes mellitus type 2 wikipedia , lookup

Diabetic ketoacidosis wikipedia , lookup

Diabetes in dogs wikipedia , lookup

Complications of diabetes mellitus wikipedia , lookup

Transcript
Ministry of Education and Science of Ukraine
Sumy State University
4176 METODOLOGICAL INSTRUCTIONS
for the practical course
“ENDOCRINOLOGY”
for the foreign students of the specialty
222 “Medicine”
of the full-time course of study
Sumy
Sumy State University
2016
Metodological instructions for the practical course
“Endocriniliogy” for the foreign students of the specialty
222 “Medicine” of the full-time course of study /
compiler N. V. Demikhova. – Sumy : Sumy State
University, 2016. – 208 p.
Department of the Family and Social Medicine
2
Contents
PREFACE ……..……………...……....................... 4
Unit 1. Diabetes mellitus………………………..…. 5
References for unit 1 ………………………………. 24
Unit 2. Thyreoid gland diseases…………………
References for unit 2 ……………………………
24
38
Unit 3. Adrenal gland diseases…………………… 40
References for unit 3 …………………………….. 50
Unit 4. Hypothalamic and pituitary diseases…….. 50
References for unit 4 ……………………………
54
Unit 5. Diseases of the reproductive system……
References for unit 5 ……………………………
55
63
Methodological Instruction to Module…………
65
Tests and Assignments for Self-assessment……..
Tests for unit 1 ...................................................
Tests for unit 2 ...................................................
Tests for unit 3 ...................................................
Tests for unit 4 ...................................................
3
73
73
140
169
189
PREFACE
The methodological instructions for the practical course
“Endocrinology” have been worked out for the fourth-year
foreign students of the specialty 222 “Medicine” of the fulltime study course, aiming to facilitate quality and efficiency of
obligatory and individual work and self-study foreseen by the
curriculum of the Family and Social Medicine Department.
The methodological instructions comprise set of
questions to lecture and various tasks on different topics.
The methodological instructions consist of five units.
4
Unit 1. Diabetes Mellitus
Methodological instruction to practicl class №1
Theme: Diabetes mellitus type 1, 2: differentiation
diagnostic, clinical assessment,
investigations.
Hours: 2.
Working place: classroom, endocrine department wards
of the Sumy Region Hospital.
Aim:
- to learn classification and pathogenic particularities of
some forms of DM;
- to learn pathomorphologic changes in organs and
systems in patients with DM;
- to learn signs, symptoms and clinical diagnostic criteria
of DM;
- to learn laboratory and instrumental diagnostic criteria
of DM.
Professional Motivation.
Type 1 diabetes is the form of the disease due
primarily to β-cell destruction. Thi s usually leads to
a type of diabetes in which insulin is required for
survival. Individuals with type 1 diabetes are
metabolically normal before the disease is clinicall y
manifest, but the process of β -cell destruction can be
detected earlier by the prese nce of certain
autoantibodies. Type 1 diabetes
usually is
characterized by the presence of anti -GAD, anti-islet
cell, or anti-insulin antibodies, which reflects the
autoimmune processes that have led to β -cell
destruction. Individuals who have one of more of
5
these antibodies can be subclassified as
having type 1A, immune-mediated type 1 diabetes.
Type 2 diabetes is the most common form of
diabetes. It is characterized by disorders of insulin
action and insulin secretion, either of which may be
the predominant feature. Usually, both are present at
the time diabetes becomes clinically manifest.
Although the specific etiology of this form of
diabetes is not known, autoimmune destruction of
the β-cells does not occur. Patients with type 2
diabetes usually have i nsulin resistance and relative,
rather than absolute, insulin deficiency. At the time
of diagnosis of diabetes, and often throughout their
lifetimes, these patients do not need insulin
treatment to survive, although ultimately many
require it for gl ycemic control. This form of diabetes
is associated with progressive β -cell failure with
increasing duration of diabetes
The currently recommended classification and
criteria for diabetes differ from those recommended
in earlier years. The major changes in classi fication
have been to move toward an etiologically based
classification because of increasing knowledge about
some of the specific underlying causes of diabetes.
This trend is likely to continue.
The terms insulin-dependent and non-insulindependent diabetes no longer appear in the
classification. These terms were often misused,
frequently reflecting the type of treatment that the
patients were receiving rather than their actual type
of diabetes. In addition, there is now greater
certainty that type 1 diabe tes is due primarily to
6
pancreatic β-cell destruction, and this
frequently can be documented by the presence of
specific
types
of
antibodies,
as
well
as
insulinopenia. The class of malnutrition -related
diabetes is no longer recognized, as this was
heterogenous, and one of its forms, fibrocalculous
pancreatic diabetes, is clearly secondary to a specific
form of pancreatic disease. The other form of
diabetes formerly described as “malnutrition related”
is now generally considered to be type 2 diabetes
that occurs in malnourished persons.
Knowledge of specific forms of diabetes
attributable to mutations in specific genes has
expanded enormously, and such cases can now be
classified very specifically if they are investigated in
detail. Importantly, this informat ion has led to
circumstances in which susceptibility to the
development of some types of diabetes can be
identified before hypergl ycemia is evident.
The introduction of clinical staging has been
prompted by the recognition that all forms of
diabetes pass through a number of stages associated
with different degrees of metabolic dysfunction.
Furthermore, it now has been established that
progression from normal glucose tolerance to overt
diabetes can be halted or delayed and that reversion
to less severe states of dysglycemia is possible. On
the other hand, hypergl ycemia may be present, but
specific information or knowledge enabling precise
etiologic classification may not be available for
many patients. Such individuals can be classified
according to clinical stage in the absence of specific
7
knowledge of causation.
Basic Level of knowledge and skills.
Students must know:
- Risk factors for DM.
- Diagnostic criteria for gestational diabetes, impaired
fasting glucose (IFG), impaired glucose tolerance (IGT).
- Differentiation diagnostic of DM. Diabetes and
abnormalities in glucose-stimulated insulin secretion.
- Clinical classification of DM: clinical classes, stages of
development, stages of severity and compensation.
- Manifestation particularity of Type 1 DM.
- Temporal model for development of type 1
diabetes.Clinical fitches.
- Particularity of Type 2 DM development and clinic.
Students should be able to:
1.
examine the patient with DM;
2.
estimate oral glucose tolerance test;
3.
do differential diagnosis of DM;
4.
evaluate dates of additional methods of
investigations for diagnosis of DM.
5.
make the basic clinical diagnosis of DM.
6.
make plan of examination of patient with DM
Organization structure of lesson:
I. Overview of topical problem of DM. 10 min.
II. Estimate basic level of knowledge –
10 min.
III. Practical work –
40 min.

Method of practical work
1.
Work with patients in the wards of
endocrinology department.
2.
Examination of patients with DM: do history
taking, palpation, percussion, auscultation.
8
3.
Working
out
symptoms
in
syndromes, emphasize the main syndrome.
4.
Make diagnosis plan of investigations.
5.
Estimate dates of additional methods of
investigations.
6.
Working out the diagnosis of DM
7.
Making and substantiation of the basis
clinical diagnosis.

Practical skills: students should acquire and use
practical skills according to the matriculs of practical skills of
the subject.
ІV. Control of students’ knowledge and skills.
1. Feed back of the practical work. – 10 min.
2 Feed back of theoretic questions. – 10 min.
V. Conclusion. Different, items. –
10 min.
Illustrations. Work with CDs, floopy discs, audi- and
video in the computers’ class-room.
Methodological Instruction to Lesson № 2.
Theme: Long-term (late) complications of diabetes
mellitus: classification and diagnostic criteria. Clinical
assessment, investigations and management.
Diabetic foot: classification and diagnostic criteria.
Clinical assessment, investigations.
Management of chronic complication. Diabetes mellitus
and pregnancy
Hours: 2 h.
Working place: classroom, endocrine department wards
of the Sumy Region Hospital.
Aim:
9
- to learn classification of DM chronic
complication;
- to learn pathogenic particularities of DM chronic
complication development;
- to learn signs, symptoms and diagnostic criteria of DM
chronic complication;
- to learn investigation methods of DM chronic
complication;
- to learn DM chronic complication management and
prevention.
Professional Motivation.
Both Type 1 and Type 2 diabetes are severe chronic
diseases that carry risks of serious chronic complications
associated with considerable morbidity and mortality. The
number of people with diabetes in the world is growing
because the world population is growing, and it is growing
older, and lifestyles are changing. The increase is greatest in
developing countries, because this is where these changes are
most dramatic. The prognosis of diabetes depends largely on
the quality of care given, the drugs and technology available,
and on the education of people with diabetes, their families and
their health care providers. Investment in this will reap rewards
in fewer and less severe complications, and longer and better
lives
The microvascular complications are specific for diabetes,
and the most ominous are retinopathy, which is a common
cause of blindness; nephropathy, which is a common cause of
renal failure; and neuropathy, which can cause risk of foot
ulcers and amputation. Also included are Charcot joints, and
features of autonomic dysfunction, including sexual
dysfunction. People with diabetes are at much increased risk
10
for arteriosclerotic diseases, which can lead to
coronary heart disease (myocardial infarction, angina pectoris
and heart failure), cerebrovascular disease leading to stroke,
and intermittent claudication and ischaemic gangrene of the
lower extremities. Certain hand abnormalities (carpal tunnel
syndrome, Dupuytren's contracture), capsulitis in the shoulder,
thick skin and limited joint mobility are also more common in
diabetes.
The microvascular complications are best prevented by
as near-normalisation of blood glucose as possible. Early
specific interventions can prevent or delay their progression,
e.g. laser therapy of the retina at the optimal stage can prevent
blindness. The atherosclerotic complications need a much
wider approach: smoking control, dietary changes to prevent
hyperlipidaemia, increased physical activity and, when
necessary, antihypertensive therapy. In addition to technology
and drugs, education of persons with diabetes and their families
is most important for success in reaching these goals.
Basic Level of knowledge and skills.
Students must know:
- Etiopatogenetic model for development of DM chronic
complication.
- Diagnostic criteria for DM chronic complication.
- Differentiation diagnostic of DM chronic complication
- Clinical classification of DM chronic complications
(Diabetic Retinopathy, Diabetic Nephropathy, Diabetic
Polyneuropathy/Mononeuropathy, Autonomic Neuropathy,
Cardiovascular complication, Diabetic Foot): clinical classes,
stages of development.
- Clinical fitches, glycemic control, long-term
complications treatment.
Students should be able to:
11
1.
examine of the patient with DM chronic
complication;
2.
estimate data of additional examination methods;
3.
do differential diagnosis of DM chronic
complication;
4.
make plan of examination of patient with DM
chronic complication;
5.
make the DM clinical complication diagnosis;
6.
make plan of DM chronic complication treatment.
Organization structure of lesson:
I. Overview of topical problem of DM. 10 min.
II. Estimate basic level of knowledge –
10 min.
III. Practical work –
40 min.

Method of practical work
1.
Work with patients in the wards of
endocrinology department.
2.
Examination of patients with chronic
complication of DM: do history taking, palpation, percussion,
auscultation, specific examination of foot.
3.
Making provisional diagnosis.
4.
Making diagnosis plan of investigations.
5.
Estimate dates of additional methods of
investigations.
6.
Making and substantiation of the basis clinical
diagnosis.
7.
Making and substantiation of the DM clinical
complication diagnosis.
8.
Making plan of long-term complications
treatment.
12

Practical skills: students should acquire
and use practical skills according to the matriculs of practical
skills of the subject.
ІV. Control of students’ knowledge and skills.
1. Feed back of the practical work. – 10 min.
2. Feed back of theoretic questions. – 10 min.
V. Conclusion. Different, items. –
10 min.
Illustrations. Work with CDs, floopy discs, audi- and
video in the computers’ class-room.
Methodological Instructions to practical class 3
Theme: Management of type 2 diabetes mellitus. The
diet. The oral hypoglycemic agents.
Hours: 2 h.
Working place: classroom, endocrine department wards
of the Sumy Region Hospital.
Aim:
- to learn principles of metabolic disorders on Type 2
DM ;
- to learn the main principles of diet therapy, exercise
program
and oral hypoglycemic agents administration;
- to learn classification oral hypoglycemic preparations;
- to learn pharmacokinetics of oral hypoglycemic
preparations;
- to learn complication of oral hypoglycemic
preparations.
Professional Motivation.
Combination therapy of oral antidiabetic agents (OAD)
with insulin is being increasingly considered in the treatment of
13
Type 2 diabetes. Although the strategies of antihyperglycaemic therapy have not changed significantly over the
past few years, there are now two major components driving the
increased application of combination therapy: a) the
commitment of physicians to good glycaemic targets and b) the
novel availability of a basal insulin with a 16- to 24-hour
plateau profile. An abundance of clinical trials have shown that
the fasting blood glucose levels and overall metabolic control
obtained in a given patient with Type 2 diabetes are dependent
on the targets set in the study. Also in primary care it has been
shown that metabolic control strongly depends on the belief of
physicians as to which would be the right targets. To achieve
these targets the conventional strategy of diet and exercise and
eventually OAD have to be employed in most cases. In Europe,
metformin and sulphonylureas are the best studied first line
drugs in monotherapy, but other options are alpha-glucosidase
inhibitors or repaglinide. When glycaemic goals are not met by
this regimen, a combination of OAD including nateglinide
may be applied. However, practical experience shows that the
need for more than two different OAD in a given patient calls
for a combination therapy with insulin in order to meet the
target.
The complexity of multiple risk reduction that is
intrinsic to the modern day management of Type 2 diabetes
has resulted in sub-optimal implementation of prevention and
treatment measures. The challenge of integrating guidelines and
protocols into everyday treatment of a complex disease like
diabetes is immense. This makes the need for populationbased systems of risk reduction even more pressing. The last
two decades have been the era of large-scale clinical trials and
molecular genetics. The next few years need to be the era of
research into delivering effective healthcare. The effective use
14
of information technology is one instrument that can
underpin quality care by providing accurate and timely
information on the results of individual patient treatment in
real time, and facilitate the tracking of patients over time for
purposes of continuous improvement of care for designated
populations or sub-populations. It is also invaluable for
purposes of epidemiological research and can provide robust
information in regard to quality of care and outcome data.
Only if the treatment of diabetes is viewed from the public
health as well as the individual perspective, are we likely to
succeed in our efforts to deliver consistent evidence-based
diabetes care.
Type 2 diabetes is a disease primarily affecting
pancreatic beta-cell function. The modern concept about
treatment of Type 2 diabetes is that drugs should be initiated
early on in the natural course of disease, as soon as HbA-|c
increases above 6.5% despite diet and physical exercise.
Metformin and a rapid-acting insulin secretagogue (nateglinide)
are the drugs of choice in initial treatment of Type 2 diabetes. If
HbA-|C increases over 6.5% despite oral drugs, insulin has to
be initiated. The first step is evening administration of insulin
glargine. However, in most cases a rapid-acting insulin analogue
at meals is needed as well. Diet and metformin should be
continued to prevent excessive increase in body weight.
Although blood glucose control is but one part of the
wider metabolic disturbance found in most people with Type 2
diabetes, the understanding that any level of hyperglycaemia
above the normal range is associated with increased risk of
arterial damage is driving earlier and earlier consideration of
initiation of insulin therapy. Accordingly, target levels for
people with Type 2 diabetes are now below those for people
with Type 1 diabetes, a reversal of perceived attitudes from
15
past decades.
Basic Level of knowledge and skills.
Students must know:
- Patophysiology of Type 2 DM.
- Classification and basis characteristics of oral
hypoglycemic preparations.
- Principles of oral hypoglycemic therapy.
- Nutritional recomendation.
Students should be able to:
1.
make plan of administrate oral hypoglycemic
preparations;
2.
use principles of glycaemic control;
3.
administrate hypocaloric diets.
Organization structure of lesson:
I. Overview of topical problem of treatment of Type 2
DM. - 10 min.
II. Estimate basis level of knowledge –
10 min.
III. Practical work –
40 min.

Method of practical work
1.
Examine patients with Type 2 DM.
2.
Estimate dates of additional methods of
investigations.
3.
Making and substantiation of the clinical
diagnosis.
4.
Making the plan of administrate oral
hypoglycemic preparations.
5.
Correct the caloric intake and exercises.

Practical skills: students should do practical
skills according to the matriculs of practical skills of the
subject.
ІV. Control of students’ knowledges and skills.
1. Discussion of the practical work. – 10 min.
16
2. Discussion of theoretic questions. – 10 min.
V. Conclusion. Organization question. –
10 min.
Illustrations. Work with compacktdiscs, floopy discs,
audi- and video in the computers class-room.
Methodological instructions to practical class 4
Theme: The main principles of Type 1 diabetes mellitus
therapy.
Insulin therapy and its complications.
Hours: 2 h.
Working place: classroom, endocrine department wards
of the Sumy Region Hospital.
Aim:
- to learn principles of physiological insulin secretion;
- to learn principles of insulin replacement therapy;
- to learn classification of insulin;
- pharmacokinetics of insulin preparations ;
- to learn schemes of insulin regimens;
- to learn complication of insulin therapy.
Professional Motivation.
The real milestone in diabetes care obviously is the
discovery of insulin and the start of treatment with insulin, a
little more than 80 years ago. Since then we have witnessed
great progress in diabetes care, indeed also related to insulin
therapy, where new modifications of the molecule make
treatment not only easier, but also more efficient, as discussed
in our meeting today.
The DCCTand UKPDS studies have definitely established
that in Type 1 as well as in Type 2 diabetes, long-term near-
17
normoglycaemia strongly protects against onset
and/or progression of microangiopathic complications.
Therefore, implementation of strategies to maintain long-term
near-normoglycaemia as early as possible, is of key importance
in the management of both types of diabetes. In Type 1
diabetes, rapid-acting insulin analogues should be given at each
meal and basal insulin substituted either as evening
administration of the long-acting insulin analogue glargine or
continuous subcutaneous insulin infusion.
Physiological insulin secretion is of two types:
continuous basal insulin secretion and incremental prandial
insulin secretion, controlling meal-related glucose excursions.
Designing insulin treatment programmes that mimic the
physiological insulin secretion profile has been a vexing
problem for clinicians and researchers. This has led to a
number of advances designed to more closely approximate an
ideal insulin absorption pattern.
The ability of patients to self-monitor blood glucose has
been a major development, which is constantly evolving in
sophistication for patient safety and convenience. Advances in
islet cell transplantation and the potential of stem cells may
essentially result in patients' independence from exogenous
insulin administration.
Basic Level of knowledge and skills.
Students must know:
1.
Physiological insulin secretion model.
2.
Classification and basis characteristics of
insulins.
3.
Methods of insulin injection.
4.
Methods of coordination the caloric intake with
the appropriate amount of insulin.
Students should be able to:
18
a.
do insulin injection;
b.
make plan of administrate and correct of insulin
regimens;
c.
use insulin penfill ;
d.
use glucometer;
e.
administrate medical nutrition therapy (MNT).
Organization structure of lesson:
I. Overview of topical problem of DM. 10 min.
II. Estimate basis level of knowledge –
10 min.
III. Practical work –
40 min.

Method of practical work
1.
Examine patients with Type 1 DM.
2.
Test the glucose blood level.
3.
Estimate dates of additional methods of
investigations.
4.
Making and substantiation of the clinical
diagnosis.
5.
Making the plan of administrate insulin
regimens.
6.
Correct the caloric intake with the appropriate
amount of insulin.

Practical skills: students should do practical
skills according to the matriculs of practical skills of the
subject.
ІV. Control of students’ knowledges and skills.
1. Discussion of the practical work. – 10 min.
2. Discussion of theoretic questions. – 10 min.
V. Conclusion. Organization question. – 10 min.
Illustrations. Work with compacktdiscs, floopy discs,
audi- and video in the computers class-room.
19
Methodological instructions to practical class 5
Theme: Regimens of insulin therapy. Complications of
insulin therapy. Curation of the patient to fill in a case history.
Hours: 2 h.
Working place: classroom, endocrine department wards
of the Sumy Region Hospital.
Aim: Curation of the patient to fill in a case history.
Professional Motivation.
Because endocrinology interfaces with numerous
physiologic systems, there is no standard endocrine history and
examination. Moreover, because most glands are relatively
inaccessible, the examination usually focuses on the
manifestations of hormone excess or deficiency, as well as
direct examination of palpable glands, such as the thyroid and
gonads. For these reasons, it is important to evaluate patients in
the context of their presenting symptoms, review of systems,
family and social history, and exposure to medications that
may affect the endocrine system. Astute clinical skills are
required to detect subtle symptoms and signs suggestive of
underlying endocrine disease. For example, a patient with
Cushing's syndrome may manifest specific findings, such as
central fat redistribution, striae, and proximal muscle
weakness, in addition to features seen commonly in the general
population, such as obesity, plethora, hypertension, and
glucose intolerance. Similarly, the insidious onset of
hypothyroidism—with mental slowing, fatigue, dry skin, and
other features—can be difficult to distinguish from similar,
nonspecific findings in the general population. Clinical
judgment, based on knowledge of disease prevalence and
pathophysiology, is required to decide when to embark on
20
more extensive evaluation of these disorders.
Laboratory testing plays an essential role in endocrinology by
allowing quantitative assessment of hormone levels and
dynamics. Radiologic imaging tests, such as computed
tomography (CT) scan, magnetic resonance imaging (MRI),
thyroid scan, and ultrasound, are also used for the diagnosis of
endocrine disorders. However, these tests are generally
employed only after a hormonal abnormality has been
established by biochemical testing.
Methodological instructions to practical class 6
Theme: Pathogenesis, diagnosis and treatment of
hyperglycemic and hypoglycemic coma in patients with
diabetes mellitus.
Defence of a case history.
Hours: 2 h.
Working place: classroom, endocrine department wards
of the Sumy Region Hospital.
Aim:
- to learn etiology, pathogenesis of acute complication of
DM;
- to learn diagnostic criteria, clinical manifistation and
management of diabetic ketoacidosis (DKA);
- to learn diagnostic criteria, clinical manifistation and
management of nonketonic hyperglycemic hyperosmolar coma
(NKHHC);
- to learn diagnostic criteria, clinical manifistation and
management of lactoacidosis (LA);
- to learn diagnostic criteria, clinical manifistation and
management of hypoglycemic coma (HC).
21
Professional Motivation.
DKA is the most frequent endocrine emergency seen by
the primary care physician. Mortality rates from 6 – 10 % have
been reported. All the abnormalities associated with DKA can
be traced to an absolute or relative insulin lack, which develops
over the period of several hours or days.
The symptoms and physical signs of DKA usually
develop over 24 hours. DKA may be the initial symptom
complex that leads to a diagnosis of type 1 DM, but more
frequently it occurs in individuals with established diabetes.
Nausea and vomiting are often prominent, and their presence in
an individual with diabetes warrants laboratory evaluation for
DKA. Abdominal pain may be severe and can resemble acute
pancreatitis or ruptured viscous.
Acute disorders are associated with potentially serious
complications if not promptly diagnosed and treated.
Using management strategies, early DKA can be
prevented or detected and treated appropriately on an
outpatient basis.
Basic Level of knowledge and skills.
Basic Level.
1.
Etiology and pathogenesis of DM acute
complication. Pathogenesis of insulin lack.
2.
Laboratory values in DKA, hyperglycemic
hyperosmolar state (HHS), LA, HC.
3.
Clinical features of DKA, HHS, LA, HC.
4.
Management strategies of DM patients with
coma.
5.
Differentional diagnostic of DM acute
complication.
22
Students should be able to:
1. examine patients with DM acute complication
2. confirm diagnosis of DKA, HHS, LA, HC;
3. make plan of DM acute complication administrate;
4. use principles of clinical-laboratory monitoring;
Organization structure of lesson:
I.
Overview of topical problem of DM acute
complication. 10 min.
II. Estimate basis level of knowledge –
10 min.
III. Practical work –
40 min.

Method of practical work
1.
Examine
patients
with
DM
acute
complication.
2.
Confirm diagnosis. Assess patient. Estimate
dates of investigations.
3.
Making the plan of administrate: replace
fluids, administer regular
insulin.
4.
Measure capillary glucose every 1–2 h;
measure electrolytes.
5.
Monitor blood pressure, pulse, respirations,
mental status, fluid intake.

Practical skills: students should do practical
skills according to the matriculs of practical skills of the
subject.
ІV. Control of students’ knowledges and skills.
1. Discussion of the practical work. – 10 min.
2. Discussion of theoretic questions. – 10 min.
V. Conclusion. Organization question. –
10 min.
Illustrations. Work with compacktdiscs, floopy discs,
audi- and video in the computers class-room.
23
References for unit 1:
1.
Manual of Endocrinology and Metabolism
(Second Edition)/ Norman Lavin. – Little, Brown and
Company.- Boston-New York-Toronto-London, 1994. - P. 111
– 142, 173 - 180..
2.
Endocrinology (A Logical Approach for
Clinicians (Second Edition)). William Jubiz.-New York: WC
Graw-Hill Book, 1985. - P. 38 – 42, 144 –164, 198 – 205.
3.
Short Textbook of Medical Diagnosis and
Management (Third Edition). Mohammad Inam Danish. –
Pakistan, 2002. – P.459 – 462, 504 – 505.
Unit 2. Thyreoid gland disease
Methodological instructions to practical class 8
Theme: Hyperthyroidism.
Graves'
disease.
Classification, etiopathogenesis, diagnostic criteria, treatment.
Hyperhtyroid crisis.
Hours: 2 h.
Working place: classroom, endocrine department wards
of the Sumy Region Hospital.
Aim:
- to learn anatomy, physiology (localization, normal sizes
and weight) of thyroid gland,
- to learn regulation of thyroid gland function,
- to learn biological effects of thyroid hormones’ action,
- to learn hormone excess causes;
- to learn etiology, pathogenesis and classification of the
hyperthyroidism;
- to learn signs, symptoms and diagnostic criteria of
hyperthyroidism,
24
- to learn diagnostic criteria and treatment
Graves' disease,
- to learn diagnostic and management of hyperhtyroid
crisis,
- to learn medical preparations which can be used for
patients with hyperthyroidism.
Professional Motivation.
Most cases of hyperthyroidism are caused by Grave’s
diseases, multinodular goitre or autonomously functioning
thyroid nodule (toxic adenoma). Graves' disease accounts for
60 to 80% of thyrotoxicosis, but the prevalence varies among
populations, depending mainly on iodine intake (high iodine
intake is associated with an increased prevalence of Graves'
disease).
Grave’s disease is distinguished clinically from other
forms of hyperthyroidism by the presence of diffuse thyroid
enlargement, ophthalmopathy and rarely pretibial myxoedema.
It can occur at any age but is unusual before puberty and most
commonly affects the 30-50 year-old age group. Clinical
features generally worsen without treatment; mortality was 10
to 30% before the introduction of satisfactory therapy. Some
patients with mild Graves' disease experience spontaneous
relapses and remissions. Rarely, there may be fluctuation
between hypo- and hyperthyroidism due to changes in the
functional activity of TSH-R antibodies.
Life-threatening increase in the severity of the clinical
features of hyperthyroidism leads to hyperthyroid crisis. It is a
medical emergency and, despite early recognition and
treatment, the mortality rate is high.
25
Basic Level of knowledge and skills.
Students must know:
1.
Thyroid hormone secretion and action
mechanisms.
2.
Etiologic classification of hyperthyroidism.
3.
Clinical classification of hyperthyroidism: stages
of development, goiter degree.
4.
Clinical features of hyperthyroidism.
5.
Criteria for the diagnosis of hyperthyroidism.
6.
Main principles of thyreostatic therapy.
7.
Grave’s disease: etiopathogenesis, autoimmune
factors, environmental factors, immunological markers,
differential diagnosis and management.
8.
Hyperthyroidism complications.
9.
Hyperhtyroid crisis.
Students should be able to:
1.
Examine the patient with hyperthyroidism.
2.
Emphasize clinical sings of hyperthyroidism.
3.
Evaluate dates of clinical methods of
investigations for diagnosis of hyperthyroidism.
4.
Make provisional diagnosis of hyperthyroidism.
5.
Make the plan of examination of a patient with
hyperthyroidism.
6.
Estimate dates of additional methods of
investigations.
7.
Make the plan of hyperhtyroid crisis
management.
Organization structure of lesson:
I. Overview of topical problem of DM. 10 min.
II. Emphasize basic level of knowledge –
10 min.
III. Practical work –
40 min.

Method of practical work
26
1.
Work with patient in the wards of
endocrinology department.
2.
Examination of patients with hyperthyroidism:
do history taking, palpation, percussion, auscultation.
3.
Work out symptoms in syndromes, emphasize
on the main syndrome.
4.
Making diagnosis plan of investigations.
5.
Making and substantiation of the basic clinical
diagnosis.
6.
Making the plan of the hyperthyroidism
treatment.

Practical skills: students should acquire and use
practical skills according to the matriculs of practical skills of
the subject.
ІV. Control of students’ knowledge and skills.
1. Feed back of the practical work. – 10 min.
2. Feed back of theoretical questions. – 10 min.
V. Conclusion. Different, items. –
10 min.
Illustrations. Work with CDs, floopy discs, audi- and
video in the computers’ class-room.
Methodological instructions to practical class 9
Theme: Hypothyroidism: diagnostic criteria, treatment.
Thyroiditis: classification, differentiation diagnostic,
clinical assessment, investigations and management.
Myxoedema coma.
Hours: 2 h.
Working place: classroom, endocrine department wards
of the Sumy Region Hospital.
Aim:
27
- to learn hormone deficiency causes;
- to learn etiology, pathogenesis and classification of the
hypothyroidism;
- to learn signs, symptoms and diagnostic criteria of
hypothyroidism and chronic complications,
- to learn classification, diagnostic criteria and treatment
of the thyroiditis,
- to learn diagnostic and management of myxoedema
coma.
Professional Motivation.
The prevalence of primary hypothyroidism is 1:100 but
increases to 5:100 if patients with subclinical hypothyroidism
(normal T4, raised TSH) are included. The female: male is
approximately 6:1. There are various causes of primary
hypothyroidism, but spontaneous atrophic hypothyroidism,
thyroid failure following I131 or surgical treatment of
hyperthyroidism, and the hypothyroidism of Hashimoto’s
thyroiditis account for over 90% of cases in those parts of the
world which are not significantly iodine-deficient.
In some patients there is a history of Graves' disease
treated with antithyroid drugs 10-20 years earlier and. very
occasionally, patients with this form of hypothyroidism
develop Graves' disease. As with any of the immunologically
mediated thyroid disorders, patients are at risk of developing
other organ-specific autoimmune conditions such as type 1
diabetes mellitus, pernicious anaemia and Addison's disease,
and autoimmune disease is not uncommon in first- and seconddegree relatives
This is the most common cause of goitrous
hypothyroidism. It typically affects 20-60-year-old women
who present with a small or moderately sized diffuse goitre
28
which is characteristically firm or rubbery in
consistency. Thyroid status depends upon the relative degrees
of lymphocytic infiltration, fibrosis and follicular cell
hyperplasia within the gland but 25% of patients are
hypothyroid at presentation.
1.
2.
3.
4.
5.
6.
7.
8.
9.
1.
2.
3.
4.
5.
6.
7.
Basic Level of knowledge and skills.
Students must know:
Thyroid hormone secretion and action mechanisms.
Etiologic classification of hypothyroidism.
Clinical classification of hypothyroidism.
Clinical features of hypohyroidism.
Criteria for the diagnosis of hypothyroidism
Mane principles of replacement therapy.
Thyroiditis: classification, etiopathogenesis, differential
diagnosis and management.
Hypohyroidism complications.
Myxoedema coma.
Students should be able to:
Examination of the patient with hypothyroidism.
Emphasize clinical sings of hypothyroidism.
Evaluate dates of clinical methods of investigations for
diagnosis of hyporthyroidism.
Make the plan of examination of patient with hypothyroidism.
Estimate dates of additional methods of investigations.
Make diagnosis of patients with hypothyroidism.
Make the plan of myxoedema coma management.
Organization structure of lesson:
I. Overview of topical problem of the hypothyroidism. 10 min.
II. Estimate basis level of knowledge – 10 min.
III. Practical work – 40 min.
29

Method of practical work
8.
Work near the bed of patient in the wards of
endocrinology department.
9.
Examine patients with hyperthyroidism: do
history taking, palpation, percussion, auscultation.
10.
Combine symptoms in syndromes, emphasize
the main syndrome.
11.
Make diagnosis plan of investigations.
12.
Differential the diagnosis of hyperthyroidism.
13.
Making and substantiation of the basis clinical
diagnosis.
14.
Make the plan of the hyperthyroidism treatment.

Practical skills: students should do practical
skills according to the matriculs of practical skills of the
subject.
ІV. Control of students’ knowledges and skills.
1. Discussion of the practical work. – 10 min.
2. Discussion of theoretic questions. – 10 min.
V. Conclusion. Organization question. – 10 min.
Illustrations. Work with compacktdiscs, floopy discs,
audi- and video in the computers class-room.
Methodological instructions to practical class 10
Theme: Nodule goiter. Diseases of the parathyroid
glands.
Hours: 2.
Working place: lection hall of the Sumy Region
Hospital.
Aim:
- to learn anatomy and physiology of parathyroid glands.
30
- to learn main functions of сalcitonin and
parathyroid hormone.
- to learn biological effects of calcium on organism.
- to learn causes parathyroid hormone excess and
deficientcy;
- to
learn
etiology,
pathogenesis
of
the
hypoparathyroidism, hyperparathyroidism;
- to learn signs, symptoms and diagnostic criteria of
hypoparathyroidism, hyperparathyroidism;
- to learn iodine deficiency: clinical assessment,
investigations, differentiation diagnostic and management;
- to learn thyroid nodules, malignant tumors.
Professional Motivation.
Patients with primary hyperparathyroidism may have a
chronic, non-specific history. Their symptoms are brought to
mind by the adage 'bones, stones and abdominal groans'.
However, about 50% of patients with primary
hyperparathyroidism are asymptomatic. In others, symptoms
may go unrecognized until patients present with renal calculi
(5% of first stone formers and 15% of recurrent stone formers
have primary hyperparathyroidism), with or without impaired
renal function, or acute dehydration and profound
hypercalcaemia.
Hypertension
is
common
in
hyperparathyroidism. Parathyroid tumours are almost never
palpable.
The most common cause of hypoparathyroidism is
damage to the parathyroid glands (or their blood supply) during
thyroid surgery, although this complication is only permanent in
1% of thyroidectomies. Transient hypocalcaemia develops in
10% of patients 12—36 hours following subtotal thyroidectomy
31
for Graves' disease.
Idiopathic hypoparathyroidism may develop at any age,
and is sometimes associated with autoimmune disease of the
adrenal, thyroid or ovary, especially in young people.
Simple multinodular goiter is rare before middle age. The
patient may have been aware of the goitre for many years,
perhaps slowly increasing in size. Rarely, medical advice may
have been sought because of painful swelling lasting a few days
caused by haemorrhage into a nodule or cyst. The goitre is
nodular or tabulated on palpation and may extend
retrosternally. Very large goitres may cause mediastinal
compression with stridor, dysphagia and obstruction of the
superior vena cava. Hoarseness due to recurrent laryngeal nerve
palsy can occur but is far more suggestive of thyroid
carcinoma.
Primary thyroid malignancy is rare, accounting for less
than 1% of all carcinomas, and has a prevalence of 25 per
million. With the exception of medullary carcinoma, thyroid
cancer is always more common in females. Papillary
carcinoma is the most common of the malignant thyroid
tumours and account for 90% of irradiation-induced thyroid
cancer. Metastases are most often found in bone, lung and
brain. Even for patients with distant metastases at presentation,
the 10-year survival is approximately 40%.
Myxoedema coma is a medical emergency in which
there is a depressed level of consciousness, usually in an
elderly patient who appears myxoedematous. The mortality
rate is 50% and survival depends upon early recognition and
treatment of hypothyroidism and other factors contributing to
the altered consciousness level, e.g. drugs such as
phenothiazines, cardiac failure, pneumonia, dilution
hyponatraemia, hypoxaemia and hypercapnia due to
32
hypoventilation.
Mane questions of the lection:
1.
Hypoparathyroidism:
classification,
etiopathogenesis, differential diagnosis and management.
2.
Hyperparathyroidism:
classification,
etiopathogenesis, differential diagnosis and management.
3.
Iodine
deficiency:
clinical
assessment,
investigations and management.
4.
Thyroid nodules: classification, pathogenesis,
differential diagnosis and management.
5.
Malignant
tumors:
clinical
assessment,
investigations, differentiation diagnostic and management.
Technical equipment: multi-media presentation.
Methodological instructions to practical class 11
Theme: Hypoparathyroidism.
Hyperparathyroidism.
Diagnostic criteria.
Clinical assessment, investigations and management.
Iodine deficiency.
Thyroid nodules. Malignant tumors. Clinical assessment,
investigations,
differentiation
diagnostic
and
management.
Hours: 2 h.
Working place: classroom, endocrine department wards
of the Sumy Region Hospital.
Aim:
- to learn anatomy and physiology of parathyroid glands.
- to learn main functions of сalcitonin and parathyroid
hormone.
33
- to learn biological effects of calcium on
organism.
- to learn causes parathyroid hormone excess and
deficientcy;
- to
learn
etiology,
pathogenesis
of
the
hypoparathyroidism, hyperparathyroidism;
- to learn signs, symptoms and diagnostic criteria of
hypoparathyroidism, hyperparathyroidism;
- to learn iodine deficiency: clinical assessment,
investigations, differentiation diagnostic and management;
- to learn thyroid nodules, malignant tumors.
Professional Motivation.
Investigation of calcium metabolism is usually
straightforward. Most laboratories measure total calcium in
serum. Differential diagnosis of disorders of calcium
metabolism requires measurement of phosphate, alkaline
phosphatase and sometimes PTH (for which the blood
sample has to be taken to the laboratory 'on ice' and
centrifuged rapidly). Hvpercalcaemia is one of the most
common biochemical abnormalities. It is detected most
frequently during routine , biochemical analysis in
asymptomatic patients. However, it can present with chronic
symptoms, and occasionally patients present as acute
emergencies with severe hvpercalcaemia and dehydration.
Hvpercalcaemia is one of the most common
biochemical abnormalities. It is detected most frequently
during routine, biochemical analysis in asymptomatic
patients. However, it can present with chronic symptoms,
and occasionally patients present as acute emergencies with
severe hvpercalcaemia and dehydration.
Patients with primary hyperparathyroidism may have a
34
chronic, non-specific history. Their symptoms are
brought to mind by the adage 'bones, stones and abdominal
groans'. However, about 50% of patients with primary
hyperparathyroidism are asymptomatic. In others, symptoms
may go unrecognized until patients present with renal calculi
(5% of first stone formers and 15% of recurrent stone formers
have primary hyperparathyroidism), with or without impaired
renal function, or acute dehydration and profound
hypercalcaemia.
Hypertension
is
common
in
hyperparathyroidism. Parathyroid tumours are almost never
palpable.
A family history of renal tract stones and/or neck
surgery raises the possibility of multiple endocrine neoplasia.
Familial hypocalciuric hypercalcaemia is a rare but important
catch for the unwary. This autosomal dominant disorder is
associated with a defective calcium receptor in the
parathyroid gland, but is almost always asymptomatic and
uncomplicated. Occasionally, these patients have had their
parathyroid glands removed unnecessarily.
Hypocalcaemia is much less common than hypercalcaemia.
Although almost all laboratories routinely report total serum
calcium concentrations, it is the ionised concentration which is
biologically important. The most common cause of hypocalcaemia
is a low serum albumin with normal ionised calcium
concentration.
The most common cause of hypoparathyroidism is
damage to the parathyroid glands (or their blood supply) during
thyroid surgery, although this complication is only permanent in
1% of thyroidectomies. Transient hypocalcaemia develops in
10% of patients 12—36 hours following subtotal thyroidectomy
for Graves' disease.
Idiopathic hypoparathyroidism may develop at any age,
35
and is sometimes associated with autoimmune disease
of the adrenal, thyroid or ovary, especially in young people.
In certain parts of the world, such as the Andes, the Himalayas and central Africa, where there is dietary iodine
deficiency, thyroid enlargement is common (more than 10% of
the population) and is known as endemic goitre. Most
patients are euthyroid and have normal or raised TSH
levels. In general the more severe the iodine deficiency, the
greater the incidence of hypothyroidism.
Simple multinodular goiter is rare before middle age. The
patient may have been aware of a goitre for many years,
perhaps slowly increasing in size. Rarely, medical advice may
have been sought because of painful swelling lasting a few days
caused by haemorrhage into a nodule or cyst. The goitre is
nodular or tabulated on palpation and may extend
retrosternally. Very large goitres may cause mediastinal
compression with stridor, dysphagia and obstruction of the
superior vena cava. Hoarseness due to recurrent laryngeal nerve
palsy can occur but is far more suggestive of thyroid
carcinoma.
Primary thyroid malignancy is rare, accounting for less
than 1% of all carcinomas, and has a prevalence of 25 per
million. With the exception of medullary carcinoma, thyroid
cancer is always more common in females. Papillary
carcinoma is the most common of the malignant thyroid
tumours and account for 90% of irradiation-induced thyroid
cancer. Metastases are most often found in bone, lung and
brain. Even for patients with distant metastases at presentation,
the 10-year survival is approximately 40%.
Myxoedema coma is a medical emergency in which
there is a depressed level of consciousness, usually in an
elderly patient who appears myxoedematous. The mortality
36
rate is 50% and survival depends upon early
recognition and treatment of hypothyroidism and other factors
contributing to the altered consciousness level, e.g. drugs such
as phenothiazines, cardiac failure, pneumonia, dilution
hyponatraemia, hypoxaemia and hypercapnia due to
hypoventilation.
Basic Level of knowledge and skills.
Students must know:
10.
Hypoparathyroidism:
classification,
etiopathogenesis, differential diagnosis and management.
11.
Hyperparathyroidism:
classification,
etiopathogenesis, differential diagnosis and management.
12.
Iodine
deficiency:
clinical
assessment,
investigations and management.
13.
Thyroid nodules: classification, pathogenesis,
differential diagnosis and management.
14.
Malignant
tumors:
clinical
assessment,
investigations, differentiation diagnostic and management.
Students should be able to:
8.
Examination of the patient with hypothyroidism.
9.
Emphasize clinical sings of hypothyroidism.
10.
Evaluate dates of clinical methods of
investigations for diagnosis of hyporthyroidism.
11.
Make the plan of examination of patient with
hypothyroidism.
12.
Estimate dates of additional methods of
investigations.
13.
Make diagnosis of patients with hypothyroidism.
14.
Make the plan of myxoedema coma
management.
Organization structure of lesson:
I. Overview of topical problem of DM. 10 min.
37
II. Estimate basis level of knowledge –
10
min.
III. Practical work –
40 min.

Method of practical work
15.
Work near the bed of patient in the wards of
endocrinology department.
16.
Examine patients: do history taking, palpation,
percussion, auscultation.
17.
Combine symptoms in syndromes, emphasize
the main syndrome.
18.
Make diagnosis plan of investigations.
19.
Making and substantiation of the basis clinical
diagnosis.
20.
Make the plan of the treatment.

Practical skills: students should do practical
skills according to the matriculs of practical skills of the
subject.
ІV. Control of students’ knowledges and skills.
1. Discussion of the practical work. – 10 min.
2. Discussion of theoretic questions. – 10 min.
V. Conclusion. Organization question. –
10 min.
Illustrations. Work with compacktdiscs, floopy discs,
audi- and video in the computers class-room.
References for unit 2:
1.
Weetman A.P: Graves' disease. N Engl J Med
343:1236, 2000
2.
Schwarnz K.M. et al: Dermopathy of Graves'
disease (pretibial myxedema): Long-term outcome. J Clin
Endocrinol Metab 87:438, 2002
38
3.
Manual
of
Endocrinology
and
Metabolism (Second Edition)/ Norman Lavin. – Little, Brown
and Company.- Boston-New York-Toronto-London, 1994. - P.
111 – 142, 173 - 180..
4.
Endocrinology (A Logical Approach for
Clinicians (Second Edition)). William Jubiz.-New York: WC
Graw-Hill Book, 1985. - P. 38 – 42, 144 –164, 198 – 205.
5.
Short Textbook of Medical Diagnosis and
Management (Third Edition). Mohammad Inam Danish. –
Pakistan, 2002. – P.459 – 462, 504 – 505.
6.
Best, C. H., and Taylor, N. B. (1966): The Thyroid
gland, Chapter 75. In: The Physiological Basis of Medical
Practice, 8th Edition, pp. 1529-1554. Williams & Wilkins Co.,
Baltimore.
7.
Ladenson P.W. et al: American Thyroid
Association guidelines for detection of thyroid dysfunction.
Arch Intern Med 160:1573, 2000
8.
Braga M. et al: Efficacy of ultrasound-guided
fine-needle aspiration biopsy in the diagnosis of complex
thyroid nodules. J Clin Endocrinol Metab 86:4089, 2001
9.
Degroot L.J. et al: Thyroid gland (Part III), in
Endocrinology, 4th ed, LJ DeGroot, JL Jameson (eds).
Philadelphia, Saunders, 2000
10.
Gharib H: Changing concepts in the diagnosis
and management of thyroid nodules. Endocrinol Metab Clin
North Am 26:777, 1997
11.
Klein I., Ojamaa K: Thyroid hormone and the
cardiovascular system. N Engl J Med 344:501, 2001
12.
Rapoport B., Mclachlan S.M.: Thyroid
autoimmunity. J Clin Invest 108:1253, 2001
39
Unit 3. Adrenal gland diseases
Methodological instructions to practical class 12
Topic: Hypofunction
of
the
adrenal
cortex.
Classification of adrenocortical insufficiency. Clinical
assessment, investigations and management of the chronic
adrenocortical
insufficiency.
Acute
adrenocortical
insufficiency. Congenital adrenal hyperplasia. Adrenal
tumours. Phaeochromocytoma. Aldosteronism. Clinical
assessment, diagnostic criteria, treatment.
Hours: 2 h.
Working place: classroom, endocrine department wards
of the Sumy Region Hospital.
Aim:
- to learn anatomy, physiology (localization, normal sizes
and weight) of the of the adrenal,
- to learn regulation of endocrine function,
- to learn biological effects of glucocorticoid hormones’
action,
- to learn etiology, pathogenesis and classification of
adrenocortical insufficiency;
- to learn signs, symptoms and diagnostic criteria of the
chronic adrenocortical insufficiency;
- to learn diagnostic and management of congenital
adrenal hyperplasia, adrenal tumours,
- to learn diagnostic criteria and management of
phaeochromocytoma;
- to learn diagnostic criteria and treatment of
aldosteronism;
- to learn diagnostic and management of acute
adrenocortical insufficiency.
40
Professional Motivation.
Adrenal insufficiency results from inadequate secretion
of cortisol and/or aldosterone. It is potentially fatal and
notoriously variable in its presentation. A high index of
suspicion is therefore required in patients with unexplained
fatigue, hyponatraemia or hypotension.
The most common is ACTH deficiency (i.e. secondary
adrenocortical failure), usually because of inappropriate
withdrawal of chronic glucocorticoid therapy or a pituitary
tumour. Congenital adrenal hyperplasias and Addison's disease
(i.e. primary adrenocortical failure) are rare, although in areas
where AIDS and tuberculosis are common associated
Addison's disease is increasing in prevalence.
In Addison's disease, either glucocorticoid or
mineralocorticoid deficiency may come first, but eventually all
patients fail to secrete both classes of corticosteroid. Similar
features occur in different combinations with other causes of
adrenocortical insufficiency.
Patients may present with chronic features and/or in
acute circulatory shock. With a chronic presentation, initial
symptoms are often misdiagnosed (e.g. as chronic fatigue
syndrome or depression). Adrenocortical insufficiency should
also be considered in patients with hyponatraemia, even in the
absence of symptoms. Vitiligo occurs in 10-20% of patients
with autoimmune Addison's disease.
Features of an acute adrenal crisis include circulatory
shock with severe hypotension, hyponatraemia, hyperkalaemia
and, in some instances, hypoglycaemia and hypercalcaemia.
Muscle cramps, nausea, vomiting, diarrhoea and unexplained
fever may be present. The crisis is often precipitated by
intercurrent disease, surgery or infection.
41
Patients with adrenocortical insufficiency always
need glucocorticoid replacement therapy and usually, but not
always, mineralocorticoid. Other treatments depend on the
underlying cause.
Most often, causes of excessive activation of
mineralocorticoid receptors results from enhanced secretion of
renin (secondary hyperaldosteronism) in response to
inadequate renal perfusion (e.g. in heart failure,
hypoalbuminaemia or renal artery stenosis). Secondary
hyperaldosteronism is not dealt with here. Less commonly,
mineralocorticoid excess occurs in the face of suppressed renin
secretion (primary hyperaldosteronism and rare disorders of
mineralocorticoid action). These disorders are usually
diagnosed in patients presenting with hypertension. Indications
to test for primary hyperaldosteronism in hypertensive patients
include hypokalaemia (including hypokalaemia induced by
thiazide diuretics), poor control of blood pressure with
conventional therapy, or presentation at a young age.
The prevalence of primary hyperaldosteronism is
controversial. If only hypertensive patients with hypokalaemia
are investigated, then fewer than 1% of patients with
hypertension
will
be
found
to
have
primary
hyperaldosteronism. Around half of these have an adrenal
adenoma secreting aldosterone (Conn's syndrome). However,
recent studies in which hypertensive patients have been
screened using aldosterone/renin ratios (see below) suggest that
the prevalence may be as high as 5%. Most of these 'extra'
patients have bilateral adrenal hyperplasia rather than Conn's
syndrome, and many have normal plasma potassium. Although
spironolactone would be the antihypertensive agent of choice
in such patients, it remains to be determined whether
investigation of all hypertensive patients for bilateral adrenal
42
hyperplasia is worth while.
Excessive secretion of catecholamines can be confirmed
by measuring the hormones (adrenaline, noradrenaline and
dopamine) in plasma or their metabolites (e.g. vallinylmandelic
acid, VMA; conjugated metanephrine and normetanephrine) in
urine. However, catecholamine secretion is usually paroxysmal
and sometimes the paroxysms are infrequent; in a patient with
classical symptoms, phaeochromocytoma can only be excluded
if the 24-hour urinary catecholamine excretion is normal on a
day on which symptoms have occurred.
Increased urinary catecholamine excretion occurs in
stressed patients (e.g. after myocardial infarction or major
surgery) and is induced by some drugs (notably β-blockers and
antidepressants). For this reason, a suppression test may be
valuable. Normal adrenomedullary secretion is suppressed by
administration of drugs which interfere with sympathetic
outflow, such as
clonidine or pentolinium.
In
phaeochromocytoma these drugs do not suppress plasma
catecholamines. Provocative tests of catecholamine release
should not be used.
Congenital adrenal hyperplasia: defects in the cortisol
biosynthetic pathway result in impaired negative feedback and
increased ACTH secretion. ACTH then stimulates the
production of steroids up to the enzyme block. This produces
adrenal hyperplasia and a combination of clinical features
which depend on the severity and site of the defect in
biosynthesis. All of these enzyme abnormalities are inherited
as autosomal recessive traits. There is therefore a 1:4 chance
that the sibling of an affected child will also have the disease,
but a low risk of passing the disease to the next generation.
The enzyme block leads to insufficiency of hormones
'distal' to the block (glucocorticoids and mineralocorticoids),
43
and impaired negative feedback suppression of ACTH
leading to accumulation of precursor hormones 'proximal' to
the block which 'spill over' into the adrenal androgen
biosynthetic pathway. The severity of the mutation in the 21hydroxylase (21 OHase) gene determines which features are
present. The most severely affected 'classical' patients present
in infancy (salt-wasting in boys; ambiguous genitalia in girls).
The least severely affected 'late-onset' patients present as adults
(hirsutism in women). (DHEA = dehydroepiandrosterone)
most common enzyme defect is 21-hydroxylase
deficiency. In about one-third of cases this defect is severe. In
the other two-thirds, mineralocorticoid secretion is not affected
but there may be features of cortisol insufficiency and/or
androgen excess.
1.
2.
3.
4.
5.
6.
Basic Level of knowledge and skills.
Students must know:
Glucocorticoid hormones’ secretion and action mechanisms.
Etiologic classification of chronic adrenocortical insufficiency.
Clinical features of chronic adrenocortical insufficiency.
Criteria for the diagnosis of phaeochromocytoma,
aldosteronism, congenital adrenal hyperplasia.
Main principles of adrenal tumours therapy.
Acute
adrenocortical
insufficiency:
etiopathogenesis,
differential diagnosis and management.
Students should be able to:
1.
Examine the patient with chronic adrenocortical
insufficiency.
2.
Emphasize clinical sings of adrenocortical
insufficiency.
3.
Evaluate dates of clinical methods of
investigations.
44
4.
Make provisional diagnosis.
5.
Make the plan of examination of a patient.
6.
Estimate dates of additional methods of
investigations.
7.
Make the plan of chronic adrenocortical
insufficienc management.
8.
Make the plan of diagnosis and management of
acute adrenocortical insufficiency.
Organization structure of lesson:
I. Overview of problem of adrenocortical insufficiency
and adrenal tumours 10 min.
II. Emphasize basic level of knowledge –10 min.
III. Practical work –40 min.
Methods of practical work:

Work with patient in the wards of endocrinology
department.

Examination of patients with adrenocortical
insufficiency or adrenal tumours: do history taking, palpation,
percussion, auscultation.

Work out symptoms in syndromes, emphasize
on the main syndrome.

Making diagnosis plan of investigations.

Making and substantiation of the basic clinical
diagnosis.

Making the plan of the adrenocortical
insufficiency and adrenal tumours diseases treatment.

Practical skills: students should acquire and use
practical skills according to the matriculs of practical skills of
the subject.
ІV. Control of students’ knowledge and skills.
1. Feed back of the practical work. –
10 min.
2. Feed back of theoretical questions. –
10 min.
45
V. Conclusion. Different, items. – 10 min.
Illustrations. Work with CDs, floopy discs, audi- and
video in the computers’ class-room.
Methodological instructions to practical class 13
Topic: Hyperfunction of the adrenal cortex.
Classification. Cushing’s syndrome. Hypothalamic syndrome
of pubertal period. Clinical assessment, differentiation
diagnostic and treatment.
Hours: 2 h.
Working place: classroom, endocrine department wards
of the Sumy Region Hospital.
Aim:
- to learn biological effects of glucocorticoid hormones’
exess,
- to learn etiology, pathogenesis and classification of
adrenocortical hyperfunction;
- to learn signs, symptoms and diagnostic criteria of the
Cushing’s syndrome;
- to learn diagnostic and management of Cushing’s
syndrome,
- to learn diagnostic criteria and management of
hypothalamic syndrome of pubertal period.
Professional Motivation.
Amongst endogenous causes, pituitary-dependent cortisol
excess (by convention, called Cushing's disease) accounts for
∼80% of cases. Both Cushing's disease and adrenal tumour are
four times more common in women than men. In contrast,
46
ectopic ACTH syndrome (often due to a small-cell
carcinoma of the bronchus) is more common in men.
Many of the diverse manifestations of glucocorticoid
excess are not specific to Cushing's syndrome and, because
spontaneous Cushing's syndrome is rare, the positive predictive
value of any one feature alone is low. Moreover, some
common disorders can be confused with Cushing's syndrome
because they are associated with alterations in cortisol
secretion: for example, obesity and depression. Features which
have the best predictive value in favour of Cushing's syndrome
in an obese patient are bruising, myopathy and hypertension.
Any clinical suspicion of cortisol excess is best resolved by
further investigation.
Some clinical features are more common in ectopic
ACTH syndrome. Unlike pituitary tumours secreting ACTH,
ectopic tumours have no residual negative feedback sensitivity
to cortisol, and both ACTH and cortisol levels are usually
higher than with other causes. Very high ACTH levels are
associated with marked pigmentation. Very high cortisol levels
overcome the barrier of 11β-HSD in the kidney and cause
hypokalaemic alkalosis. Hypokalaemia aggravates both
myopathy and hyperglycaemia (by inhibiting insulin secretion).
When the tumour secreting ACTH is malignant (e.g. pancreatic
or small-cell lung carcinomas), then the onset is usually rapid
and may be associated with cachexia. For these reasons, the
classical features of Cushing's syndrome are less common in
ectopic ACTH syndrome, and if present suggest that a benign
tumour (e.g. bronchial carcinoid) is responsible.
In Cushing's disease the pituitary tumour is almost
always a microadenoma (< 10 mm in diameter); hence other
features of a pituitary macroadenoma (hypopituitarism, visual
failure or disconnection hyperprolactinaemia) are rare.
47
Once the presence of Cushing's syndrome is
established, measurement of plasma ACTH is key in
establishing the differential diagnosis. In the presence of excess
cortisol secretion, an undetectable ACTH indicates an adrenal
tumour while any detectable ACTH is pathological. Tests to
discriminate pituitary from ectopic sources of ACTH rely on
the fact that pituitary tumours, but not ectopic tumours, retain
some features of normal regulation of ACTH secretion. Thus,
in Cushing's disease ACTH secretion is suppressed by
dexamethasone, albeit at a higher dose than in health, and
ACTH is stimulated by corticotrophin-releasing hormone
(CRH).
This is essential, as untreated Cushing's syndrome has a
50% 5-year mortality. Most patients are treated surgically with
medical therapy given for a few weeks prior to operation. The
type of surgery depends on the cause
1.
2.
3.
4.
5.
Basic Level of knowledge and skills.
Students must know:
Action mechanisms of glucocorticoid hormones’ exess.
Etiologic
classification
of
chronic
adrenocortical
hyperfunction.
Clinical features of chronic adrenocortical hyperfunction.
Criteria for the diagnosis of Cushing’s syndrome, hypothalamic
syndrome of pubertal period.
Main principles of therapy adrenocortical hyperfunction.
Students should be able to:
1.
Examine the patient with adrenocortical
hyperfunction.
2.
Emphasize clinical sings of adrenocortical
hyperfunction.
48
3.
Evaluate dates of clinical methods of
investigations.
4.
Make provisional diagnosis.
5.
Make the plan of examination of a patient.
6.
Estimate dates of additional methods of
investigations.
7.
Make the plan of chronic adrenocortical
hyperfunction management.
Organization structure of lesson:
I. Overview of problem of adrenocortical insufficiency
and adrenal tumours 10 min.
II. Emphasize basic level of knowledge –
10 min.
III. Practical work – 40 min.

Method of practical work
1.
Work with patient in the wards of
endocrinology department.
2.
Examination of patients with Cushing’s
syndrome or hypothalamic syndrome of pubertal period: do
history taking, palpation, percussion, auscultation.
3.
Work out symptoms in syndromes,
emphasize on the main syndrome.
4.
Making diagnosis plan of investigations.
5.
Making and substantiation of the basic
clinical diagnosis.
6.
Making the plan of Cushing’s syndrome or
hypothalamic syndrome of pubertal period treatment.

Practical skills: students should acquire and use
practical skills according to the matriculs of practical skills of
the subject.
ІV. Control of students’ knowledge and skills.
1. Feed back of the practical work. – 10 min.
2. Feed back of theoretical questions. –
10 min.
49
V. Conclusion. Different, items. –
10 min.
Illustrations. Work with CDs, floopy discs, audi- and
video in the computers’ class-room.
References for unit 3:
1.
Manual of Endocrinology and Metabolism
(Second Edition)/ Norman Lavin. – Little, Brown and
Company.- Boston-New York-Toronto-London, 1994. - P. 111
– 142, 173 - 180..
2.
Endocrinology (A Logical Approach for
Clinicians (Second Edition)). William Jubiz.-New York: WC
Graw-Hill Book, 1985. - P. 38 – 42, 144 –164, 198 – 205.
3.
Short Textbook of Medical Diagnosis and
Management (Third Edition). Mohammad Inam Danish. –
Pakistan, 2002. – P.459 – 462, 504 – 505.
Unit 4. Hypothalamic and pituitary diseases.
Methodological instructions to practical class 14
Theme: Hypothalamic
and
pituitary
diseases.
Hypopituitarism. Obesity. Acromegaly. Gigantism. Pituitary
dwarfism. Diabetes insipidus. Syndrome of inappropriate ADH
secretion. Prolactinoma. Clinical assessment, investigations
and management.
Hours: 2 h.
Working place: classroom, endocrine department wards
of the Sumy Region Hospital.
Aim:
50
- to learn anatomy, physiology (localization,
normal sizes and weight) of the pituitary gland,
- to learn regulation of endocrine function,
- to learn biological effects of thyroid hormones’ action,
- to learn hormone excess causes;
- to learn etiology, pathogenesis and classification of the
hypopituitarism;
- to learn signs, symptoms and diagnostic criteria of
hypopituitarism, obesity;
- to learn diagnostic and management diabetes insipidus,
syndrome of inappropriate ADH secretion,
- to learn diagnostic criteria and management pituitary
dwarfism;
- to learn diagnostic criteria and treatment of acromegaly,
gigantism;
- to learn diagnostic and management of prolactinoma.
Professional Motivation.
The anterior pituitary is often referred to as the “master
gland” because, together with the hypothalamus, it orchestrates
the complex regulatory functions of multiple other endocrine
glands.
Pituitary tumors cause characteristic hormone excess
syndromes. Hormone deficiency may be inherited or acquired.
Fortunately, efficacious treatments exist for the various
pituitary hormone excess and deficiency syndromes.
Nonetheless, these diagnoses are often elusive, emphasizing
the importance of recognizing subtle clinical manifestations
and performing the correct laboratory diagnostic tests.
The anterior pituitary is often referred to as the “master
gland” because, together with the hypothalamus, it orchestrates
the complex regulatory functions of multiple other endocrine
51
glands. The anterior pituitary gland produces six major
hormones: (1) prolactin (PRL), (2) growth hormone (GH), (3)
adrenocorticotropin hormone (ACTH), (4) luteinizing hormone
(LH), (5) follicle-stimulating hormone (FSH), and (6) thyroidstimulating hormone (TSH). Pituitary hormones are secreted in
a pulsatile manner, reflecting stimulation by an array of
specific hypothalamic releasing factors. Each of these pituitary
hormones elicits specific responses in peripheral target tissues.
The hormonal products of these peripheral glands, in turn,
exert feedback control at the level of the hypothalamus and
pituitary to modulate pituitary function. Pituitary tumors cause
characteristic hormone excess syndromes. Hormone deficiency
may be inherited or acquired. Fortunately, efficacious
treatments exist for the various pituitary hormone excess and
deficiency syndromes. Nonetheless, these diagnoses are often
elusive, emphasizing the importance of recognizing subtle
clinical manifestations and performing the correct laboratory
diagnostic tests.
Diseases of the hypothalamus and pituitary are rare,
with an annual incidence of ∼1:50 000. They are usually
diagnosed in patients presenting with a classical syndrome of
hormone excess (e.g. acromegaly or prolactinoma), hormone
deficiency (e.g. hypopituitarism, isolated secondary
hypogonadism or adrenal insufficiency) or a space-occupying
lesion (headache and/or visual disturbance). The pituitary plays
a central role in several major endocrine axes, so that
investigation and treatment involve several other glands. The
reader may usefully use the following section in part as
revision of disorders of each gland which have been described
above.
52
1.
2.
3.
4.
5.
-
Basic Level of knowledge and skills.
Students must know:
Pituitary hormone secretion and action mechanisms.
Etiologic classification of hypopituitarism.
Clinical features of hypopituitarism, diabetes insipidus,
syndrome of inappropriate ADH secretion,
Criteria for the diagnosis of pituitary dwarfism, acromegaly,
gigantism, prolactinoma. Main principles of therapy.
Hypopituitarism: etiopathogenesis, differential diagnosis and
management.
Students should be able to:
Examine the patient with hypothalamic and pituitary diseases.
Emphasize clinical sings of hypothalamic and pituitary
diseases.
Evaluate dates of clinical methods of investigations.
Make provisional diagnosis.
Make the plan of examination of a patient.
Estimate dates of additional methods of investigations.
Make the plan of hypothalamic and pituitary diseases
management.
Organization structure of lesson:
I. Overview of topical problem of hypothalamic and
pituitary diseases. - 10 min.
II. Emphasize basic level of knowledge –
10 min.
III. Practical work – 40 min.

Method of practical work

Work with patient in the wards of endocrinology
department.

Examination of patients with hypothalamic and
pituitary diseases: do history taking, palpation, percussion,
auscultation.
53

Work out symptoms in syndromes,
emphasize on the main syndrome.

Making diagnosis plan of investigations.

Making and substantiation of the basic clinical
diagnosis.

Making the plan of the hypothalamic and
pituitary diseases treatment.

Practical skills: students should acquire and use
practical skills according to the matriculs of practical skills of
the subject.
ІV. Control of students’ knowledge and skills.
1. Feed back of the practical work. –
10 min.
2. Feed back of theoretical questions. –
10 min.
V. Conclusion. Different, items. –
10 min.
Illustrations. Work with CDs, floopy discs, audi- and
video in the computers’ class-room.
References for unit 4:
1. Manual of Endocrinology and Metabolism (Second Edition)/
Norman Lavin. – Little, Brown and Company.- Boston-New
York-Toronto-London, 1994. - P. 111 – 142, 173 - 180..
2. Endocrinology (A Logical Approach for Clinicians (Second
Edition)). William Jubiz.-New York: WC Graw-Hill Book,
1985. - P. 38 – 42, 144 –164, 198 – 205.
3. Short Textbook of Medical Diagnosis and Management (Third
Edition). Mohammad Inam Danish. – Pakistan, 2002. – P.459 –
462, 504 – 505.
54
Unit 5. Diseases of the Reproductive system.
Methodological instructions to practical class 14
Topic: Diseases of the Reproductive system. Delayed
puberty. Sexual precocity. Disorders of sexual differentiation.
Primary and secondary hypogonadism. The hermaphroditism.
The menopause. Diagnosis and the differential diagnosis,
management.
Hours: 2 h.
Working place: classroom, endocrine department wards
of the Sumy Region Hospital.
Aim:
- to learn biological effects of sex hormones,
- to learn male and female reproductive physiology, and
the normal menstrual cycle,
- to learn etiology, pathogenesis and classification of
sexual differentiation disorders;
- to learn etiology, pathogenesis and classification
primary and secondary hypogonadism;
- to learn signs, symptoms and diagnostic criteria of the
hermaphroditism;
- to learn diagnostic and management of menopause.
Professional Motivation.
Clinical practice in reproductive medicine is shared
between several specialties, including gynaecology, urology,
psychiatry and endocrinology. The following section focuses
on aspects that are commonly managed by endocrinologists.
The clinical features of primary (failure of the testis) and
secondary (failure of the hypothalamus or anterior pituitary)
hypogonadism are identical. These include loss of libido,
lethargy with muscle weakness, and decreased frequency of
shaving. Patients commonly present with gynaecomastia,
55
erectile impotence, infertility or delayed puberty.
Male hypogonadism is confirmed by demonstrating a low
serum testosterone level. The distinction between primary and
secondary hypogonadism is made by measurement of random
LH and FSH. Patients with hypogonadotrophic hypogonadism
(i.e. secondary hypogonadism) should be investigated as
described
for
pituitary
disease.
Patients
with
hypergonadotrophic
hypogonadism
(i.e.
primary
hypogonadism) should have the testes examined for
cryptorchidism or tumours, measurement of serum ferritin (to
exclude haemochromatosis) and a karyotype (to identify
Klinefelter's syndrome, i.e. 47, XXY). If there is no obvious
cause, then no further investigations are necessary.
Psychotherapy which includes the sexual partner is most
useful for psychological problems. Neuropathy and vascular
disease are unlikely to improve, but several treatments are
available. First-line therapy is usually with oral sildenafil, a
phosphodiesterase inhibitor which potentiates the vasodilator
action of nitric oxide on cyclic guanosine monophosphate
(cGMP). Coadministration of sildenafil with nitric oxide
donors ('nitrate' drugs) is contraindicated because of the risk of
severe hypotension. Caution should also be exercised in
patients with chronic disease including ischaemic heart disease,
principally because the unaccustomed stress of sexual activity
may precipitate cardiac ischaemia or dysrhythmia. Other
treatments
for
impotence
include
self-administered
intracavernosal injection or urethral gel administration of
prostaglandin E1; vacuum devices which achieve an erection
which is maintained by a tourniquet around the base of the
penis; and prosthetic implants, either of a fixed rod or of an
inflatable reservoir. Many patients elect not to use these
methods, but unfortunately even more are unaware of their
56
availability.
Around 10% of couples have difficulty in conceiving
children. This is attributable in roughly equal thirds to
infertility in the female, infertility in the male, and idiopathic
cases. So, although it is common for women to present with
this problem, early assessment of both partners is essential to
avoid unnecessary investigations and delay. This should
include establishing that the couple are having intercourse
when the woman is likely to be fertile.
Further assessment of women includes a menstrual
history. Oligomenorrhoea suggests that the cycles are
anovulatory. This can be confirmed by measurement of serum
progesterone 21 days after the start of the last menstrual period
(ovulation indicated by level >15 nmol/l). Subsequent tests are
similar to those for secondary amenorrhoea above. If the
woman has regular menses, and no abnormality is found in the
man, then further gynaecological investigation may be
required.
The male should be examined for a varicocele or other
testicular abnormality. A semen analysis should be performed.
If he has oligospermia, then blood should be taken for
prolactin, testosterone, FSH and LH and these interpreted as
described for male hypogonadism above. If the only
biochemical abnormality is a high FSH, then an irreversible
failure of spermatogenesis is likely (the FSH rises because of
lack of β-inhibin). Testicular biopsy is rarely indicated.
In patients with gonadotrophin deficiency, fertility can be
induced over several months. This is usually performed once,
and sperm stored for subsequent artificial insemination.
Basic Level of knowledge and skills.
Students must know:
1. Action mechanisms of sex hormones.
57
2. Normal male and female reproductive physiology, and
the normal menstrual cycle.
3. Etiologic classification of primary and secondary
hypogonadism.
4. Clinical features of sexual differentiation disorders.
5. Criteria for the diagnosis of sexual differentiation disorders,
menopause.
6. Main principles of therapy sexual dysfunction.
Students should be able to:
1.
Examine the patient with hypogonadism or
menopause.
2.
Emphasize clinical sings of hypogonadism or
menopause.
3.
Evaluate dates of clinical methods of
investigations.
4.
Make provisional diagnosis.
5.
Make the plan of examination of a patient.
6.
Estimate dates of additional methods of
investigations.
7.
Make the plan of hypogonadism and menopause
management.
Organization structure of lesson:
I.
Overview of problem of hypogonadism and
menopause - 10 min.
II. Emphasize basic level of knowledge – 10 min.
III. Practical work – 40 min.

Method of practical work
1.
Work with patient in the wards of
endocrinology department.
2.
Examination
of
patients
with
of
hypogonadism and menopause: do history taking, palpation,
percussion, auscultation.
58
3.
Work out symptoms in syndromes,
emphasize on the main syndrome.
4.
Making diagnosis plan of investigations.
5.
Making and substantiation of the basic
clinical diagnosis.
6.
Making the plan of of hypogonadism and
menopause treatment.

Practical skills: students should acquire and use
practical skills according to the matriculs of practical skills of
the subject.
ІV. Control of students’ knowledge and skills.
1. Feed back of the practical work. – 10 min.
2. Feed back of theoretical questions. – 10 min.
V. Conclusion. Different, items. – 10 min.
Illustrations. Work with CDs, floopy discs, audi- and
video in the computers’ class-room.
Methodological instructions to practical class 15
Theme: Principles of endocrine system organization.
Diabetes mellitus definition. Clinical features and
diagnosis of diabetes mellitus.
Hours: 2 h.
Working place: classroom, endocrine department wards
of the Sumy Region Hospital.
Aim:
- to learn hormone secretion, action, maintenance of
homeostasis;
- to learn principles of hormonal feedback regulatory
systems;
59
- to learn paracrine and autocrine control;
- to learn causes hormone excess and deficiency,
hormone resistance;
- to learn etiology, pathogenesis of the diabetes mellitus
(DM);
- to learn signs, symptoms and diagnostic criteria of DM.
Professional Motivation.
An understanding of principles of endocrine system
organization is important to diagnosis and treatment.
Endocrinology concerns the synthesis, secretion and action of
hormones. These are chemical messengers which have diverse
molecular structures, are released from endocrine glands and
coordinate the activities of many different cells. Endocrine
disease therefore has a wide range of manifestation affecting
many other major organs. Most endocrine glands are controlled
by hormones released from the pituitary gland. These
integrated endocrine systems are called “axes”, but some major
endocrine glands are not controlled by the pituitary. The
classical model of endocrine function involves hormones
which are synthesised in endocrine glands, released into the
circulation, and act at sites distant from those secretions.
However, additional levels of complex regulation have now
been recognised. A wide variety of molecules act as hormones.
The clinical implications of this complexity of hormone action
are only now being appreciated.
The worldwide prevalence of DM has risen
dramatically over the past two decades. Likewise,
prevalence rates of impaired fasting glucose (IFG)
are also increasing. Although the prevalence of both
type 1 and type 2 DM is increasing worldwide, the
prevalence of type 2 DM is expected to rise more
60
rapidly in the future because of increasing
obesity and reduced activity levels. DM increases
with aging. In 2000, the prevalence of DM was
estimated to be 0.19% in people <20 years old and
8.6% in people >20 years old . In individuals >65
years the prevalence of DM was 20.1%. The
prevalence is similar in men and women throughout
most age ranges but is slightly greater in men >60
years. There is considerable geographic variation in
the incidence of both type 1 and type 2 DM.
Scandinavia has the highest incidence of type 1 DM
(e.g., in Finland, the incidence is 35/100,000 per
year). The Pacific Rim has a much lower rate (in
Japan and China, the incidence is 1 to 3/100,000 per
year) of type 1 DM; Northern Europe and the Uni ted
States share an intermediate rate (8 to 17/100,000 per
year). Much of the increased risk of type 1 DM is
believed to reflect the frequency of high -risk HLA
alleles among ethnic groups in different geographic
locations. The prevalence of type 2 DM and i ts
harbinger, impaired glucose tolerance (IGT), is
highest in certain Pacific islands, intermediate in
countries such as India and the United States, and
relatively low in Russia and China. This variability is
likely due to genetic, behavioral, and environ mental
factors. DM prevalence also varies among different
ethnic populations within a given country.
Gestational diabetes mellitus (GDM) occurs in
approximately 4% of pregnancies; most women revert
to normal glucose tolerance post -partum but have a
substantial risk (30 to 60%) of developing DM later
in life.
61
Basic Level of knowledge and skills.
Students must know:
1. Hormone secretion and action mechanisms
2. Main principles of hormonal regulatory systems
3. Insulin biosynthesis, secretion, and action. Regulation of
glucose blood level.
4. Epidemiology. Screening type 1, 2 DM.
5. Criteria for the diagnosis of DM. Diabetes and abnormalities in
glucose-stimulated insulin secretion.
6. Etiologic classification of DM.
7. Etiopathogenesis type 1 DM. autoimmune factors,
environmental factors, immunological markers.
8. Etiopathogenesis type 2 DM. Metabolic changes during the
development of type 2 diabetes mellitus. Insulin resistance.
Impaired insulin secretion. Increased hepatic glucose
production.
9. Genetically defined, monogenetic forms of DM.
Students should be able to:
1. Examine the patient with DM.
2. Call syndromes for DM.
3. Evaluate dates of clinical methods of investigations for
diagnosis of DM.
4. Make provisional diagnosis of DM.
5. Make the plan of examination of patient with DM
Organization structure of lesson:
I. Overview of topical problem of DM. II. Estimate basic level of knowledge –
III. Practical work –

Method of practical work
62
10 min.
10 min.
40 min.

Work with patients in the wards of
endocrinology department.

Examination of patients with DM: do history
taking, palpation, percussion, auscultation.

Working out symptoms in syndromes,
emphasize on the main syndrome.

Making the provisional diagnosis.

Making diagnosis plan of investigations.

Practical skills: students should acquire and use
practical skills according to the matriculs of practical skills of
the subject.
ІV. Control of students’ knowledge and skills.
1. Feed back of the practical work. – 10 min.
2. Feed back of theoretical questions. – 10 min.
V. Conclusion. Different, items. –
10 min.
Illustrations. Work with CDs, floopy discs, audi- and
video in the computers’ class-room.
References for unit 5.
1.
Endocrinology (A Logical Approach for
Clinicians (Second Edition)). William Jubiz.-New York: WC
Graw-Hill Book, 1985. - P. 38 – 42, 144 –164, 198 – 205.
2.
Short Textbook of Medical Diagnosis and
Management (Third Edition). Mohammad Inam Danish. –
Pakistan, 2002. – P.459 – 462, 504 – 505.
3.
Manual of Endocrinology and Metabolism
(Second Edition)/ Norman Lavin. – Little, Brown and
Company.- Boston-New York-Toronto-London, 1994. - P. 111
– 142, 173 - 180.
63
4.
Endocrinology (A Logical Approach
for Clinicians (Second Edition)). William Jubiz.-New York:
WC Graw-Hill Book, 1985. - P. 38 – 42, 144 –164, 198 – 205.
5.
Short Textbook of Medical Diagnosis and
Management (Third Edition). Mohammad Inam Danish. –
Pakistan, 2002. – P.459 – 462, 504 – 505.
64
Methodological Instruction to Module.
Theme: Control of the students’ knowledges and skills
on DM.
Hours: 2 h.
Working place: classroom.
Aim: to control students’ knowledges, confirm skills on
diabetes mellitus, DM long-term and acute complication.
PROGRAME FOR THE ISSUE OF
DIFFERENTIATED CREDITS ON ENDOCRINOLOGY
FOR 4th COURSE STUDENTS
FIRST CATEGORY
1. Acromegaly. Etiology, pathogenesis, clinical
manifestations,
diagnosis
and
treatment.
2. Cushing’s Disease. Etiology, pathogenesis, clinical
manifestations,
diagnosis
and
treatment.
3. Diabetes insipidus. Etiology, pathogenesis, clinical
manifestations,
diagnosis
and
treatment.
4. Hypopituitarism in children. Differential diagnosis of growth
retardation in children.
5. Treatment of endocrine -induced growth disorders in
children.
6. Iodine deficiency conditions and diseases caused by lack of
iodine.
7. Diffuse toxic goiter. Etiology, pathogenesis, clinical
manifestations, diagnosis and treatment.
8. Hypothyroidism. Etiology, pathogenesis, classification,
clinical
picture,
diagnosis
and
treatment.
9. Peculiarities of hypothyroidism, congenital hypothyroidism,
65
hypothyroidism in the elderly, clinical course of
atypical
forms
of
hypothyroidism.
10. Acute thyroiditis. Etiology, clinical manifestations,
differential diagnosis, treatment.
11. Chronic thyroiditis. The etiology, classification, diagnosis,
treatment.
12. Cancer of the thyroid gland. Classification. The role of the
Chernobyl accident in the occurrence of thyroid cancer.
13. Hyperparathyroidism. Etiology, pathogenesis, clinical
manifestations, diagnosis and treatment.
14. Hypoparathyroidism. Etiology, clinical manifestations,
diagnosis and treatment.
15. Diabetes. Determining the prevalence of the disease.
Diagnosis of diabetes.
16. The etiology of diabetes. Classification of glucose
disorders.
17. The clinical course of diabetes. Differences clinical course
depending
on
type
of
diabetes.
18. Classification of chronic complications of diabetes.
Prevention of chronic complications of diabetes.
19. Glycated Hemoglobin. Diagnostic value of glycated
hemoglobin.
20. Indications for glucose tolerance test . Evaluation of the
test.
21. Diet therapy of diabetes. The glycemic index of foods. The
value of food glycemic in the development of chronic
complications of diabetes.
22. Hypoglycemic sulfonamides. Classification of drugs. The
mechanism of hypoglycemic sulfonamides impact .
23. Prescribe treatment with hypoglycemic sulfonamides. Side
effect of medications. Features of hypoglycemic conditions
caused by sulfonamides.
66
24. Biguanide. Prescribe for the treatment of diabetes.
Indications and contraindications for use, adverse reactions of
Biguanide.
25. Postprandial stimulators of insulin secretion. Classification.
Methods of use.
26. Thiazolidinedione’s in the treatment of diabetes.
Indications, adverse reactions to drugs, the control treatment.
27. Indications for insulin therapy in patients with diabetes.
Prescribe insulin therapy for patient newly diagnosed of
diabetes. Mode of insulin therapy. Performance indicators
insulin treatment.
28. Complications of insulin therapy. Hypoglycemic states.
29.Causes and urgent measures for hypoglycemia. Preventive
measures for hypoglycemia.
30. Causes of decompensation of diabetes. Diabetic ketonuria,
ketoacidosis.
Diagnosis
and
treatment.
31. Diabetic coma. Causes. Diagnosis and treatment.
32. Chronic adrenal insufficiency. Etiology, pathogenesis,
clinical
manifestations,
diagnosis
and
treatment.
33. Cushing’s Syndrome. Etiology, pathogenesis , clinical
manifestations, diagnosis and treatment.
34. Pheochromocytoma. Etiology, clinical features, diagnosis
and treatment.
35. Congenital adrenal hyperplasia. Clinical manifestations,
diagnosis, treatment.
36. Cryptorchidism, etiology, pathogenesis, diagnosis and
treatment.
37. Syndrome Turner. Clinical manifestations, diagnosis and
treatment.
38. Klinefelter syndrome. Clinical manifestations, diagnosis
and treatment.
67
39. Disorders of sexual development. Premature
sexual development, delayed sexual development. Clinical
manifestations, diagnosis and treatment.
40. Obesity. Definition , etiology , classification of obesity by
body mass index. Treatment.
41. Obesity in children. Hypothalamic -pituitary obesity.
Etiology . Clinical manifestations . Differential diagnosis .
Treatment.
SECOND CATEGORY
1. Different types of hormones and their place of
establishment in the body.
2. Classification hormone chemical structure.
3. Basic functions of hormones, their genomic and epigenomic
effects.
4. The synthesis of hormones and their secretion, transport in
the blood and metabolism.
5.
Mechanisms
of
action
of
hormones.
6.
Regulation
of
endocrine
function.
7.
Basic
mechanisms
of
endocrine
diseases.
7. Anatomical and physiological data on the pituitary and
hypothalamus.
8. Classification of hypothalamic- pituitary disease.
9. Hyperprolactinemia syndrome. Etiology, pathogenesis,
clinical
manifestations,
diagnosis
and
treatment.
10. Hypopituitarism . Etiology, pathogenesis, clinical
manifestations,
diagnosis
and
treatment.
11. Anatomical and physiological data on the thyroid gland.
12. The concept of goiter. The reasons for the increase in
thyroid size.
13. Laboratory and instrumental methods of examination of
68
thyroid gland. Indications , contraindications for use,
the diagnostic value of each method .
14. The methods of iodine prophylaxis .
15. The impact of man-made pollution of the environment on
the development of thyroid cancer.
16. Nodules in the thyroid gland. Etiology. Clinic differential
diagnosis .
17. Indications and contraindications for surgical treatment of
thyroid
cancer.
18. Differential diagnosis of hyperthyroidism syndrome.
19. Thyrotoxicosis crisis . Causes , clinical manifestations,
diagnosis and treatment.
20. The definition and classification of thyroiditis .
21. Autoimmune thyroiditis. Classification , clinical course,
diffusion.
diagnosis
and
treatment.
22. The active detection of thyroid cancer , treatment,
replacment therapy.
23. Anatomical and physiological characteristics of the
parathyroid glands and their role in the regulation of calcium
homeostasis.
24. Metabolic osteopathy in endocrine diseases. The etiology
and pathogenesis , differential diagnosis , treatment and
prevention.
25. Anatomical and physiological data on endocrine part of the
pancreas.
26. The use of sweeteners in the diet of diabetics. Classification
of sweeteners . The use of dietary fiber in the diets of patients
with diabetes.
27. The physical load in patients with diabetes. Purpose
graduated
exercise
based
on
blood
glucose.
28. Hlyukomodulyatoriv use in the treatment of diabetes
mellitus.
69
29. Use of herbal products in the treatment of diabetes
mellitus.
30.
Classification
of
oral
hypoglycemic
drugs.
31. Diabetes and pregnancy. Contraindications to pregnancy in
patients with diabetes. Terms of diabetes in pregnancy.
32. Classification of insulin drugs, duration of action.
33. Coma in patients with diabetes. Hyperosmolar ( nonacidotic ) coma. Lactic acidosis and coma. Causes. Diagnosis.
Urgent measures. Prevention.
34. Diabetic angiopathy of lower extremities. Stages of
development, diagnosis, preventive measures for angiopathy.
35. Diabetic polyneuropathy. Manifestations of autonomic
diabetic neuropathy.
36. Diabetic foot syndrome . Diffusion. diagnosis of
neuropathic
and
ischemic
foot
lesions
form.
37. Labour and medical- social examination of patients with
diabetes.
38. The anatomic features of the adrenal glands, the adrenal
hormones each layer .
39. The physiological action of adrenal hormones .
40. Acute adrenal insufficiency. Etiology, clinical
manifestations,
diagnosis
and
treatment.
41. Addison crises. Causes, clinical manifestations, diagnosis,
emergency care.
42. Classification hormonal active tumors of the adrenal
cortical layer.
43. The morphological structure of the endocrine testis and
ovary.
Physiological
effects
of
sex
hormones.
44.
Classification
of
disorders
of
puberty.
45. Menopause. Pathological course of menopause. Clinical
manifestations in women and men. Treatment of pathological
manifestations
of
menopause.
70
46. The impact of obesity on the occurrence of lesions
of the organs and systems of the human body.
47. Differential diagnosis of clinical forms of obesity.
48. The metabolic syndrome. Diagnosis. Treatment.
49. Autoimmune Polyglandular Syndrome. Definition, clinical
manifestations,
diagnosis
and
treatment.
50. Multiple endocrine neoplasia syndrome, definitions,
diagnosis, treatment.
THIRD CATEGORY
1. Physiology of the pineal gland and its functional role
in the human.
2. Classification and clinical manifestations of the pineal gland
pathology.
3. Anatomy and physiological role of the thymus.
4. Immunological failure is caused by thymic hypoplasia.
5. Myasthenia gravis. The clinical manifistation. Features of
myasthenia gravis in children. Diagnosis and treatment of
myasthenia gravis.
6. Tumors of the thymus. Clinical course . Diagnosis .
Treatment.
7. Principles of the thyroid surgery.
8. Complications after thyroid surgery.
9. Indications for surgical treatment of nodular forms of goiter .
10. Surgical treatment of adrenal tumors, preoperative
preparation, postoperative period, the rehabilitation of patients
after adrenalectomy.
71
LIST of practical skills that should have
STUDENT
1.
Assess
glucose
tolerance
test.
2.
Assess
glycemic
and
glycosuria
profile.
3. Determine the type of diabetes, its clinical course and
condition of compensation.
4. Determine blood glucose and acetone in the urine method.
Determine
blood
glucose
rapid
method.
5.
Prescribe
dietary
treatment
of
diabetes.
6.
Prescribing
of
oral
hypoglycemic
agents.
7. Identify secondary sulfanilamide resistance and, to be able to
treat it.
8.
Rehabilitation
of
endocrine
patients.
9. Assign mode of insulin therapy in diabetes.
10. Assign treatment of patients with ketoacidosis.
11. Assign scheme of treatment for diabetic coma.
12. Able to use syringe pen.
13. Develop a plan for self-management of diabetic patients.
14. Determine the degree of goiter.
15. Estimate data and Doppler ultrasonography of the thyroid
gland.
16. Evaluate the results of radioisotope examination and
thermography thyroid gland.
17. Assess the state of thyroid system according to
radioimmunoassay and enzyme immunoassay tests.
18. Assign treatment of patients with toxic goiter.
19. Evaluate the results of ECG and reflex meter to
characterize the function of the thyroid gland.
20. Diagnose hypothyroidism.
21. Assess the condition of the adrenal glands of the clinical
data, the results of hormonal tests, ultrasound, arteriography,
72
CT , MRI .
22.
Prescribe
treatment
of
Addison
crisis.
23. Evaluate X-ray and data CT scan, MRI of the skull.
24. Determine the morphotype.
25. Determine the degree of obesity and BMI.
26. Determine the degree somatosexual development.
27. Define “bone age" according to radiography.
28.
Estimate
data
chromosomal
analysis.
29. Identify types of disorders of sexual differentiation.
30. Diagnose menopause treatment regimen and schedule
pathological menopause.
31. Determine the extent of disability of patients with
endocrine disorders.
Tests and Assignments for Self-assessment.
UNIT 1
A.
1. Patient L., 29 y-r-old, takes prednisolon in
treatment of SLE during last 4 years. She is complains of
weight gain, polyuria, thirst. Plasma glucose concentration
is 12,4 mmol/l. Put diagnosis, please.
A. Diabetes mellitus type 1
B. Diabetes mellitus type 2
*C. Steroid diabetes
D. Impaired glucose tolerance
E. Impaired fasting glycemia
2. Diabetic K., 31 y-r-old, is pregnant. What
laboratory data we have to control in pregnant diabetes,
EXCEPT fast glycemia?
The level of postprandial glucemia.
73
B.
C.
D.
E.
B.
C.
D.
E.
F.
A.
B.
C.
D.
E.
A.
B.
C.
D.
E.
The level of glucosurea.
*The level of glicolize Hb
The level of the insulin in the blood.
The level of cholesterol.
3. Patient B., 40 years, has the type 1 of DM for 19
years. Choose the degree of severity of the diabetes mellitus
in a case of the level of fasting glucose – 6,8 mmol/l,
aglucosurea, the level of creatinine 0,36 mmol/l.
Mild.
Moderate.
*Severe
Stabile.
Progressive.
4. 30. In patient R., 19 years old, was found first
time glucosurea 5 g/l, the level of the glucemia is 5,5 mmol/l.
What investigation can exclude diabetes mellitus?
Daily fluctuation of glucemia level.
*Glucose tolerance test.
The level of the insulin in the blood.
The level of postprandial glucemia.
Daily glucosurea
5. 30. In patient R., 19 years old, was found first
time glucosurea 5 g/l, the level of the glucemia is 5,5 mmol/l.
What investigation can exclude diabetes mellitus?
Daily fluctuation of glucemia level.
*Glucose tolerance test.
The level of the insulin in the blood.
The level of postprandial glucemia.
Daily glucosurea
6. A 28-year-old man is diagnosed with diabetes
mellitus.
Which of the following findings would point most
74
A.
B.
C.
D.
E.
A.
B.
C.
D.
E.
convincingly to type 1 rather than type 2 diabetes?
He is not obese.
Severe polydipsia and polyuria preceded the diagnosis,
*Ketoacidosis was present at the time of diagnosis.
The serum C-peptide level was normal at the time of diagnosis.
Glucose levels rose above 32 mmol/l (600 mg/dL) before
treatment was started.
7. A 31-year-old female is diagnosed with diabetes
mellitus. Type 1 diabetes mellitus is usually associated with
which one of the following?
Onset in middle age
Obesity
Insulin resistance
*Requirement for insulin therapy
Absence of autoimmune manifestations
8. A 37-year-old male is diagnosed with diabetes
mellitus Very high serum level of anti-islet cell antibodies is
most closely associated with:
*Type 1 diabetes mellitus
Type 2 diabetes mellitus
Impaired fasting glucose
Impaired glucose tolerance
Diabetic ketoacidosis
9. A 47 y-r-old overweight man complains of
paradontosis. The most sensitive test for diabetes mellitus:
Fasting serum glucose level
Random serum glucose level
*Oral glucose tolerance test
Serum level of hemoglobin A1C
Urine glucose concentration
10. 62. A 14-yr-old girl presents with 3 weeks of
weight loss, polyuria and polydipsia.
75
A.
B.
C.
D.
E.
Prescribe investigations, please.
Fasting blood glucose*
Urinary ketones
T3, T4 and TSH levels
HbAlc levels
C-peptide levels
11. Interpret GTT. Fasting glucose level is 7.1
mmol/1; 2 hours after glucose ingestion 14.7 mmol/1 in
plasma.
A.
Diabetes mellitus
B.
Impairment of carbohydrate tolerance
C.
Normal
D.
Necessary to repeat test
E.
Additional laboratory investigations are
indicated
A.
B.
C.
D.
12. Woman 62 years, which is ill by Diabetes Mellitus
type 2, complains about a trophic ulcer on the leg. The origin
of ulcer binds by carrying of uncomfortable shoe. At a review
the feet are dry and brilliant; the pulsation on the back arteries
of foot is satisfactory. The ulcer on the right foot is unsickly.
What is your previous diagnosis?
Diabetes mellitus type 2, diabetic angiopathy, foot ulcers
Diabetes mellitus type 2, diabetic foot, neuro-ischemic form
Diabetes mellitus type 2, diabetic foot, ischemic form
Diabetes mellitus type 2, diabetic neuropathy
13. Woman 64 years, which is ill by Diabetes Mellitus
type 2, treats by Maninil in a dose 5 mg in the morning. The
fasting level of glucose is 6.5 mmol/L. Advised by the oculist.
The conclusion is “diabetic retinopathy”. What is the
subsequent method of treatment?
76
A. Maninil in a dose 5 mg
B. insulin of short action
C. insulin of the prolonged action
D. insulin of short and prolonged action
E. combined therapy
14.Patient N., 56 years old. Duration of Diabetes
Mellitus type 2-10 years. Regularly receives glibenclamide
0,005 1 tablet 3 times per day. In the last three years, patient
was not examined by endocrinologist. Foot was chilly, and
pulse was faint. Fasting blood glucose- 15,0 mmol/1. Glucose
in urine - 2,0 %
Substantiate diagnosis.
A. Diabetes mellitus type 2, inadequate control
B. Diabetes insipidus
C. Diabetes mellitus type 2, adequate control
D. Diabetes mellitus type 1, adequate control
E. Diabetes mellitus type 2, inadequate control,
ketoacidosis
15. Interpret GTT. Glycemia: I trial - 5.4 mmol/1, II
trial - 7.8 mmol/1, III trial - 5.5 mmol/1
A.
Normal
B.
Impairment of carbohydrate tolerance
C.
Diabetes mellitus
D.
Necessary to order additional laboratory tests.
E.
Additional laboratory investigations are indicate
16. Patient 56 years old is ill by Diabetes Mellitus type
2, complains about dizziness, darkening in the eyes at the
getting up. These phenomena accompany perspiration on the
face, neck, thorax. The reception of the meal does not give the
77
a.
b.
c.
d.
e.
facilitation. He ills by diabetes for 42 years. The last
year he adopted Mannil in a dose 1 or 2 pills per day.
Compensation of the disease is satisfactory. The indicated
complaints coincided with resistance tachycardia. AP – 140/95
mm.hg., standing – 90/60 mm.hg. Your previous diagnosis?
Diabetes Mellitus type 1. Diabetic vegetative neuropathy of a
heart
Diabetes Mellitus type 1, climacteric syndrome. Dismetabolic
cardiopathy
Diabetes Mellitus type 1. Diabetic miocardiopathy
Diabetes Mellitus type 1,atherosclerotic cardiosclerosis
Non of this variant.
17. Patient with persistent dermatomyositis examined
glucose and appearance such results: the fasting level – 5,55
mmol/L and 6,68 mmol/L, during the day 7,85;9,11;11,13
mmol/L. Your conclusion.
A.
Diabetes mellitus
B.
Impairment of carbohydrate tolerance
C.
Normal
D. Necessary to order additional laboratory tests.
E.
Needed an add examination
18. Which of the statement given below is correct
relatively to GTT?
A.
Correlates with determination of potential
abnormalities of glucose tolerance
B.
Helpful to choose the most appropriate treatment
C.
Used to differentiate type of diabetes
D.
Indicate stage of diabetes
E.
Useful in the seeking of early diabetic
complications
78
19. A patient K., 66 years old, which
complains on the headache, eyestrain, loss of appetite. He is ill
during 11 years on the diabetes mellitus 1. During the
examination: pale, swell face, heart increase in the left sight on
1,5 sm. Pulse – 70/min, rhythmical, pressure 180/100 mm.hg..
Stomach is soft, liver doesn’t increased. Pasternackyj
symptom’s is negative with both sight. Urination –
accelerating, mainly in the night. Analysis of the blood: Hb 98
g/l, er. 3* 1012 /L.lk. 8,2 *109 /l, urea 15,8 mM/l. Cholesterin
7,6 mM/l, albumen – 81 g/l, creatynine 177 mcM/l, glucose 8,2
mM/l.
Analysis of urin : specific gravity 1010, albumen 0,6
g/l, er. 1-3, l. 10-15, glucose 2 g/l, ECG: hypertrophy of the left
ventricle. Your preliminary diagnosis?
A. Diabetes mellitus 2, hard form in the stage of
decompensation. Diabetic nephropathy.
B. Diabetes mellitus 2, high pressure disease, stage 3.
C. Chronic glomerulonephritis.
D. Diabetes mellitus 2, hard form in the stage of compensation.
Chronic pyelonephritis, high pressure disease.
E. Non of this variant.
20. A patient W., which is ill for 15 years by Diabetes
Mellitus type 2, during last year determine weakness,
hypertonia, swelling of lower limbs and face. In the general
analis of urine: albumen – 0,99 g/L, glucose 11 mmol/L,
leukocyte 3 – 5, creatunine, urea are norma. The fasting level
of glucose – 11,5 mmol/L. Established the most suitable
diagnose?
A. Diabetes Mellitus type 2, adequate control,
diabetic nephropathy, persistent proteinuria
79
B.
Diabetes Mellitus type 2, inadequate
control, diabetic nephropathy, persistent
proteinuria
C. Diabetes Mellitus type 2, adequate control, diabetic
nephropathy, renal impairment
D. Diabetes Mellitus type 2, inadequate control,
diabetic nephropathy, microalbuminuria
E. Non of this variant
21. In case of what disease is the number of insulin receptors
reduced:
a. acromegaly
b. insulinoma
c. diabetes mellitus, type 2
d. adiposity
e. thyrotoxicosis
22. In the pathogenesis of the insulinoresistance the
following factors play part:
a. hyperlipidemia
b. Antibodies to insulin
c. hormone of growth
d. cortisol
e. antibodies to receptors of insulin
23. Mark what characterizes the diabetes mellitus of
middle degree of gravity:
a. the necessity to be treated with insulin
b. the compensation of diabetes mellitus only with a diet
c. the necessity of taking tabletlike sugar lowering
preparations
d. absence of angiopathy
e. presence of stable complications of diabetes mellitus
80
24. A light form of the diabetes mellitus is
characterized by:
a. absence of ketoacidosis
b. the necessity of injecting nearly 10 units of insulin
c. presence of hepatopathy
d. the necessity of taking of tabletlike sugar reducing
preparations
e. the compensation of the diabetes mellitus is achieved
only by a diet
25. Mark the characteristic symptoms of the first type of
diabetes mellitus:
a. young age
b. rapid beginning of the disease
c. inclination to ketoacidosis
d. growing thin on the background of the increased appetite
e. all mentioned above
26. The patient is 54 years old, the diabetes mellitus
was suspected and a test on the tolerance to glucose was
prescribed: on an empty stomach – 7,0 mmol/litre, in 2 hours –
11,5 mmol/litre. Evaluate the results of tolerance test to
glucose.
a. breaking tolerance to glucose
b. obvious diabetes mellitus
c. normal test
d. breaking glycemia on the empty stomach
e. repeated examination is necessary
27. A woman, 46 years old, has been suffering from
diabetes mellitus for 6 years. She is being treated with peroral
sugar reducing preparations. At present, being examined, does
81
not put forward any complaints. Which methods can
be used to evaluate the compensation of the diabetes mellitus?
a. determining the glycemic profile
b. determining the level of glucose in blood on the empty
stomach
c. making a tolerance test to hydrocarbonas
d. Determining of the glycosylated haemoglobin
e. determining glucose level in the blood during 24 hours
28. The patient of 32 years old, with the diabetes
mellitus revealed for the first time, is keeping up glycemia on
the empty stomach less than 6 mmol/litre, after the meal less
than 9 mmol/litre by means of the diet. The patient refrains
from insulin therapy. What investigation must be carried out
for receiving differential diagnostics being the diabetes
mellitus type1 or type 2?
a. assigning glycosylated haemoglobin
b. assigning antibodies to the cells of the pancreas
insular
c. test of glucose tolerance
d. assigning the level of glucagon
e. assigning glycemia on the empty stomach
29. The patient, 48 years old has been treated from the
hypertension. Being hospitalized complained of dryness in the
mouth, polydipsia, increased appetite, polyuria. His father
suffered from the diabetes mellitus. In his blood the level of
glucose – 14,6 mmol/litre was detected. Your preliminary
diagnosis is:
a. breaking glucose tolerance
b. nonmellitus diabetes
c. diabetes mellitus, type 1
82
d. renal glucosuria
e. diabetes mellitus type 2
30. A 36 years old woman complains of dryness in the
mouth, thirst, polyuria losing 5 kg of weight during half a year.
Has been ill for 7 months. Her skin and mucus membranes are
dry. Glucosuria is 20 gr/litre, glycemia on the empty stomach
is up to 10 mmol/litre. Urine reaction to acetone is negative.
Which of the investigations will be most reasonable to define
the type of the diabetes mellitus?
a. determining С-peptide level
b. glycemic profile
c. determining glycosylated haemoglobin
d. determining HLA-antigens
e. glucose tolerance test
31. Patient, 45 years, complaints about dry mouth,
thirst, polyuria. Estimate the results of glucose tolerance test:
fasting - 7,2mmol/l, after 1 hour - 13,2mmol/l, after 2 hrs 11,5mmol/l.
A. Evident diabetes mellitus
B. Impaired glucose tolerance
C. Impaired fasting glycemia
D. It's need repeated examination
E. It's need to conduct a glucose-prednizolone test.
The right answer: A.
32.A patient, 56 years, was hospitalized in endocrinol
ogy department with complaints about dry mouth, thirst,
polyuria. Estimate the results of glucose tolerance test: fasting 5,8mmol/l, after 1 hour - 9,6mmol/l, after 2hrs - 7,7 mmol/l.
A. Impaired fasting glycemia
83
B. Impaired glucose tolerance
C. Evident diabetes mellitus
D. It's need repeated examination.
E. It's need to conduct a glucose-prednizolone test
The right answer: A.
33. A patient, 38 years, came to ambulatory with
complaints about the weight loss, general weakness, dry
mouth, polyuria. Estimate the results of glucose tolerance test:
fasting - 5,8 mmol/l, after 1 hour - 9,8mmol/l, after 2 hrs 8,3mmol/l.
A. Impaired glucose tolerance
B. Impaired fasting glycemia
C. Obvious saccharine diabetes
D. Necessary repeated examination
E. It is necessary to conduct a glucose-prednisolone test
The right answer: A.
34. Man, 45 years., has diabetes mellitus type 2. He
intakes amaryl 3mg a day. Which methods of treatment
efficacy control are the most adequately represent the state of
compensation in ambulatory terms?
A. Assessment of glycated haemoglobin
B. Definition of glycemic profile
C. Definition of fasting glucose
D. Examination of glucose tolerance test
E. Definition of blood lipoproteins
The right answer: A.
35. Youth, 18 years, is ill with diabetes mellitus of
moderate severity during 2 years and he uses insulinotherapy.
He appealed to endocrinologist with the purpose of advice
84
about the choice of future profession. What from these
professions aren't contra-indicated for this patient?
A. Teacher at school
B. Work at badtime, business trips
C. Height works
D. Machinist
E. Pilot, driver
The right answer: A.
36. A man, 26 years, complains about thirst, polyuria,
general weakness, weight loss. Objectively: a skin is dry, red
cheeks, breathing is vesicular. Cardiac tones are sounding.
Tongue is dry. The symptoms of irritation of peritoneum are
not present. What examination is the most informing for
establishment of diagnosis?
A. Glucose blood test
B. Common blood count
C. Common analysis of urine
D. Analysis of urine by Zemnitsky
E. Blood test on hepatic probesВірні відповіді : A
The right answer: A.
37. Patient, 36 years, appealed to district internist with
complaints about general weakness, weight loss to 5 kg for the
last 3 months, strong thirst. Laboratory data: fasting glucose 9,2mmol/l, glucosuria 1,5%, ketonuria is absent. What
examination is need for determination of type of diabetes?
A. Definition of С-peptide
B. Common blood count
C. Definition of glycated haemoglobin
D. Level of postprandial glucose
E. Blood test on hepatic probes
85
The right answer: A.
38. The most typical sign of diabetes mellitus type 1 is:
A. Presence of antibodies to pancreatic islet tissue
B. Association with HLA-antigens
C. Decreasing of amount of receptors to insulin
D. Increase of insulinoresistance
E. Increase of plasma osmolarity
The right answer : A
39. What from clinical symptoms are not typical for
hypoglycemia?
A. Diarrhea
B. Hyperhydrosis
C. Sense of the hunger
D. Dizziness
E. Palpitation
The right answer : A
40. What basic intermediate substance which appears in
the process of biosynthesis of insulin will represent
insulinproduced function of pancreas in patient who intakes
insulin?
A. С-peptide
B. Proinsulin
C. Insulin like growth factor - 1
D. Glucagon
E. Preproinsulin
The right answer: A
41. A 59 years old man, with height 175 cm, weight 74
kg, (and his father has been suffering from diabetes mellitus for
86
last 24 years), is tested against the glucose tolerant
test: glycemia on an empty stomach - 4,8 mmol/l, in 2 hours
after glucose load (pp) (75 g) - 6,8 mmol/l. Choose a glucose
tolerant test interpretation to the results from below:
A. *NORMAL,
B. Impaired tolerance to glucose,
C. Type 1 Diabetes mellitus,
D. Type 2 Diabetes mellitus,
E. The glucose tolerant test indications are absent.
42. A woman being 24 complains to raised thirst,
frequent urination and skin itch. She went to a local polyclinic.
They did glucose tolerant test to her: glucose level on an empty
stomach - 8,7 mmol/l, in 2 hours after glucose load (pp) (75 g)
- 14,3 mmol/l. Choose a glucose tolerant test interpretation to
the results from below:
A. *Type 1diabetes mellitus,
B. Type 2 diabetes mellitus.
C. Impaired tolerance to glucose
D. Normal.
E. Allergetic dermatitis.
43. A 48 years old man, with height 179 cm, weight 94
kg, having done the glucose tolerant test on: glucose level on
an empty stomach - 5,9 mmol/l, in 2 hours after glucose load
(pp) (75 g) - 8,4 mmol/l. Choose a glucose tolerant test
interpretation to the results from below:
A. *Glycosylated hemoglobin,
B. Level of total cholesterol,
C. Level of triglyceride,
D. Measurement of blood pressure,
E. All listed above.
87
44. Patient, 30 years, suffers on diabetes mellitus Type1
during 13 years. He noticed the appearance of edema of the
face, lower extremities, increase of arterial pressure, up to
180/110 mm. Sugar of blood on an empty stomach - 11,6
mmol/l.
Doctor-endocrinologist
considered
these
complaints as exposures of nephropathy. What researches do
need to be conducted for the exposure of the first signs of
worsening of function of buds?
A.* Presence of microalbuminuriya;
B. Ultrasound kidney;
C. The level of glycosylated of HbAlc;
D. Determining the level of blood urea;
E. Determining the level of creatinine.
45. Patient, 56 years, an engineer, is ill diabetes mellitus
Type 2 during 3 years, began to notice deterioration of visual
acuity of variable character. Sugar of blood during conducting
of test of tolerance to glucose of 9,9 mmol/l, 10,1 mmol/l, 8,7
mmol/m. To set the degree of weight of diabetes and
complication:
A.* Diabetes mellitus is type 2, moderate , phase
compensation. Diabetic cataract I;
B. Diabetes mellitus is type 2, moderate, phase of sub
compensation. Diabetic retinopathy II;
C. Diabetes mellitus, moderate, phase of compensation.
Diabetic cataract;
D. Diabetes mellitus type 2, moderate, phase of sub
compensation;
E. Diabetes mellitus type 2, easy motion, phase of sub
compensation. Diabetic retinopathy .
88
46. Patient., 27 years, contacted doctor with
the complaints about the increased thirst, increase of amount of
urine, increased appetite, gradual emaciation for several
months. During an inspection level of glucose on an empty
stomach – 8,8 mmol/l. Test of tolerance to glucose – in 2 hours
glycemia of 12,6 mmol/l. The most likely diagnosis?
A. * Diabetes mellitus type 1;
B. Diabetes mellitus type 2;
C. Non- Diabetes mellitus;
D. Violation of tolerance to glucose;
E. Broken glycemia on an empty stomach.
47. What inspection does need to be conducted for the
exposure of latent diabetes mellitus?
A.* Glucose tolerance test;
B. Daily glucosuria ;
C. Determination of glycemic profile;
D. Determination of glycosylated hemoglobin;
E. Determination of c-peptide.
48. The ill M., 12 years old, after the carried windy pox
in a heavy form there was the promoted thirst (drinks to 5 liters
in a day), poliuriya , dry skin, pruritus. The grandfather of
patient is ill diabetes mellitus. Level of glycemia – 15 mmol/l,
acetone of urine +. To define a diagnosis.
A.* Diabetes mellitus
is
type 1, stage of
decompensation, ketosis ;
B. Diabetes mellitus type 1, subindenifications;
C. Diabetes mellitus 2 types, easy motion;
D. Diabetes mellitus type 2, to middle weight, stage of
indemnification;
E. Diabetes mellitus type 1, heavy form, ketoacidosis.
89
49. A patient 19 years had diabetes mellitus the
type 2 «masonic type». Why is this type of diabetes «adult
type» in young people?
A. * Presence of mutations of gene of glucokinasa;
B. Stresses;
C. Alcoholic intoxication;
D. Drug addiction;
E. Smoking.
A.
B.
C.
D.
E.
50. For patient A., 50 years, obesity type 3. Inspects on
diabetes mellitus. Table of contents of glucose in blood on an
empty stomach - 6,0 mmol/l. Sugar in urine is absent. In urine
tracks of ketone bodies. Glycosylated HbAlc – 6%. IRI insulin
–– 180 mmol/l. What inspection must be done?
A.* Glucose tolerance test;
B. Daily gluсosuria;
C. Daily proteinuria;
D. Glycemic profile;
E. Level of C-peptide.
51. A 49-year-old female, insulin dependent diabetic, been
on treatment for 19 years, presents with urinary frequency
but no dysuria or urgency. Her blood glucose is 18.6 mmo/l.
Why patient has urinary problems?
Hyperglycaemia*
Hypoglycaemia
Urinary tract infection
Somatic neuropathy
Autonomic neuropathy
52. 17-year-old female complains of polyuria,
polydipsia, weight loss, and blurred vision. The level of
fasting serum glucose is 15,9 mmol/L.
What is your diagnosis?
90
A.
B.
C.
D.
E.
A.
B.
C.
D.
E.
A.
B.
C.
D.
E.
Health.
Impaired glucose tolerance.
*Diabetes mellitus type 1.
Diabetes mellitus type 2.
Impaired fasting glycemia.
53. Patient J., 34 years old, has diabetes mellitus
during 7 years, takes insulin (53 Units). Patient sometimes
has diabetic nonproliferative retinopathy. Fasting glucemia
is 11,3 mmol/l.
What is your previous diagnosis?
*Diabetes mellitus type 1, moderate degree.
Diabetes mellitus type 1, severe degree.
Diabetes mellitus type 1 mild degree.
Diabetes mellitus type 2, moderate degree.
Diabetes mellitus type 2, severe degree.
54. A 29-year-old female in her first trimester of
pregnancy complains of polyuria, polydipsia. Fasting
plasma glucose 8,7 mmol/L was found. She was treated
with a diet and insulin which resulted in a reduction in
plasma glucose to normal.
Which method is the most important to establish the
stage of compensation of diabetes mellitus?
The level of fast glucemia.
The level of postprandial glucemia.
The level of glucosurea.
*The level of glicolize Hb
The level of the insulin in the blood.
55. Patient M., 22 y.o., has first time diagnosed DM.
Fasting glucose is 15,2 mmol/l, glucosurea – 2,5 g/l, glicolize
Hb – 10 %.
What treatment is necessary?
А. Glitazones
91
B. Diet therapy
C. Biguanides
D. Sulfonylurea medicines II generation
E. Insulin therapy*
56. Patient is 13 years old. Felt ill rapidly. Complaints
on thirst, polyurea, general weakness, during last 2 weeks
lost 2 kg of weight. Fasting blood glucose level – 32 mmol/l,
in the urine – 6 %, acetone (+).
Your treatment?
А. Diet therapy
B. Phytotherapy
C. Biguanides
D. Sulfonilurea medicines
E. Insuline therapy*
57. Patient 24 years old is ill with DM during 7 years.
Uses injections of semi-long acting insulin. Variability of
blood glucose level during day is 15,2 – 22,0 mmol/l, fasting
glucose – 28,6 mmol/l.
What treatment should be prescribed
A.
Intensive insulin therapy*
B.
Biguanides
C.
To increase dose of semi-long acting insulin
D.
To prescribe short-acting insulin twice a day
E.
Sulfonilurea medicines
58. Patient R. of 16 is ill with DM type 1. In the morning
takes Actrapid 16 U, Protaphan 20 U, before dinner –
Actrapid 6 U, in the evening - Actrapid 4 U, Protaphan 12
U. Height 179 sm, weight 60 kg. Glucose level: fasting – 6,4
mmol/l, 13.00 – 8,8 mmol/l, 18.30 – 9 mmol/l, 22.00 – 7,1
mmol/l. Glcosurea 5 g/l.
What has to be changed in treatment?
A. To increase evening dose of Protaphan
92
B.
C.
D.
E.
To increase morning dose of Protaphan *
To increase dose of Actrapid before dinner
To not change doses of insulin
To increase morning and evening doses of Actrapid
59. Patient P. 42 y.o. suffers from DM type 1, takes
insulin Farmasulin 30/70 before breakfast – 32 U, before
supper – 16 U. Glucemia during day – 8.00 – 7,5, 13.00 –
12,0, 18.00 – 14,2, 21.00 – 16,0, 3.00 – 9,0 mmol/l.
What changes of treatment have to be made?
A. To change doses of Protaphan in the morning to 30 U, in the
evening to 12 U
B. To decrease evening amount of carbohydrates
C. To decrease day amount of carbohydrates
D. To change doses of Protaphan in the morning to 36 U, in the
evening to 20 U *
E. To change insulin
60. Patient C. 17 years old with DM takes intensive
scheme of insulin therapy. Presents with nightmares, in the
morning – headache, general weakness. Glucemis: 8.00 –
10,1, 12.00 – 6,6, 17.00 – 4,3, 21.00 – 3.7 mmol/l.
What is wrong in the insulin therapy scheme?
A. Too high dose of semi-long acting insulin in the evening
*
B. Not enough dose of semi-long acting insulin in the
evening
C. Not enough dose of semi-long acting insulin in the
morning
D. Too high dose of short acting insulin in the evening
E.
Not enough dose of semi-long acting insulin per day
61. Child, 11 years old presents with recidivating of
furunculous. Fasting blood glucose is 7,9 mmol/1, glucose and
acetone absent in urine. What is the most likely diagnosis?
93
A. Diabetes mellitus type 1
B. Chronic infections
C. Diabetes mellitus type 2
D. Dermatopathi
E. Diabetic ketoacidosis
62. Patient V., 26 years old, complains of an itch about
the genitals, frequent urination, tiresome thirst, weight gain.
These symptoms were evidenced over several months. Fasting
blood glucose - 12,0 mmol/1, glucose in urine - 1,5 %. What is
the most likely diagnosis?
A. Diabetes mellitus type 1
B. Diabetes insipidus
C. Diabetes mellitus type 2
D. Dermatopathi
E. Chronic infections
63. Patient with persistent dermatomyositis examined
glucose and appearance such results: the fasting level – 5,55
mmol/L and 6,68 mmol/L, during the day 7,85;9,11;11,13
mmol/L. Your conclusion.
A.
Diabetes mellitus
B.
Impairment of carbohydrate tolerance
C.
Normal
D. Necessary to order additional laboratory tests.
E.
Needed an add examination
64. Patient M., 27 years, after recovering from the flu,
complaining of thirst, frequent urination, and weight loss.
Blood glucose is - 12,3 mmol/1, in urine - 3%, acetone - +.
What is your previous diagnosis?
A. Diabetes mellitus type 1, inadequate control,
ketosis
B. Diabetes insipidus
C. Diabetes mellitus type 2
94
D. Diabetes mellitus type 1, adequate control
E. Diabetes mellitus type 2, inadequate control,
ketoacidosis
65. Which of the statement given below is correct
relatively to GTT?
A.
Correlates with determination of potential
abnormalities of glucose tolerance
B.
Helpful to choose the most appropriate treatment
C.
Used to differentiate type of diabetes
D.
Indicate stage of diabetes
E.
Useful in the seeking of early diabetic
complications
66. Interpret GTT. Glycemia: I trial - 5.3 mmol/1, II
trial - 7.8 mmol/1, III trial - 4.8 mmol/1
A.
Normal
B.
Impairment of carbohydrate tolerance
C.
Diabetes mellitus
D. Necessary to order additional laboratory tests.
E.
Additional laboratory investigations are
indicated
67. Patient 30 years is being ill by Diabetes Mellitus for
10 years. He accepts the insulin in a dose 46 OD in two
injections. There are frequent hypoglycemic commas, decline
of the sight, pain in lower extremities in anamnesis. The micro
aneurisms, hemorrhage, expansion of veins of the retina are
determined on the eyeing bottom. The fasting level of glucose
is 13.2 mmol/L. What is your previous diagnosis?
A. diabetes mellitus type 1, diabetic background retinopathy,
diabetic foot syndrome
B. diabetes mellitus type 2, diabetic proliferative retinopathy,
diabetic foot syndrome
95
C. diabetes mellitus type 1, diabetic proliferative
retinopathy, diabetic neuropathy
D. diabetes mellitus type 1, diabetic preproliferative retinopathy,
diabetic neuropathy, diabetic nephropathy
E. Non of this variant
68. A patient W., which is ill for 15 years by Diabetes
Mellitus type 1, during last year determine weakness,
hypertonia, swelling of lower limbs and face. In the general
analis of urine: albumen – 0,99 g/L, glucose 11 mmol/L,
leukocyte 3 – 5, creatinine, urea are normal. The fasting level
of glucose – 11,5 mmol/L. Established the most suitable
diagnose?
A. Diabetes Mellitus type 1, adequate control,
diabetic nephropathy, persistent proteinuria
B. Diabetes Mellitus type 1, inadequate control,
diabetic nephropathy, persistent
proteinuria
C. Diabetes Mellitus type 1, adequate control, diabetic
nephropathy, renal impairment
D. Diabetes Mellitus type 1, inadequate control,
diabetic nephropathy, microalbuminuria
E. Non of this variant
69. A patient 32 years, for the first time diagnosed the
Diabetes Mellitus type 1, complained of hypersensitive in the
both of legs, especially in the region of thighs, touch of the
clothes caused unbearable pain. Connecting appearance of pain
with symptoms of diabetes and lost of weight. Your
preliminary diagnosis?
A. Diabetes mellitus type 1, diabetic distal symmetrical
sensorimotor polyneuropathy
B. Diabetes mellitus type 1, diabetic angiopathy
C. Diabetes mellitus type 1, diabetic autonomic neuropathy
D. Diabetes mellitus type 1, diabetic amyotrophy
96
E. Non of this variant
70. Patient 42 years old is ill by Diabetes Mellitus type
1, entered permanent establishment with with complaints about
the loss of the weight, growing the weakness. He ills by
diabetes for a year. He treats by insulin in a dose 12 OD
Humudari R of rapid actuion and 18 OD Humudari B of
prolong action. Diabetes is stable, hypoglycemia is not present.
At a review: considerable grow thin, especially muscles.
Pulsation on the distal parts of arteries of the feet is good, the
trophic changes are not exposed. Achilles' and knee’s reflexes
are considerably slow from the both sides. Vibration, tactile,
thermal sensitiveness are not damage. Your previous
diagnosis?
a. Diabetes Mellitus type 1 in the stage of decompensation
b. Diabetes Mellitus type 1, asthenia of somatogene.
c. Diabetes Mellitus type 1, peripheral polyneuropathy
d. Diabetes Mellitus type 1, total motor polyneuropathy
e. Diabetes Mellitus type 1, total sensor polyneuropathy
71. What food caloric content must the patient have if
he suffers from diabetes mellitus and has adiposity of the
second stage?
a. caloric content must be reduced on 20-30% on the
account of limiting of hydrocarbons and fats.
b. dose starvation prescribing
c. caloric content must be reduced on 10 % on the account
of hydrocarbons limiting
d. physiological diet
e. caloric content must be increased on 20-30 % on the
account of hydrocarbons
72. Which of the enumerated complications can be
caused by taking therapeutic doses of sulfanilamide sugar
reducing preparations?
97
a. dysbacteriosis
b. allergy
c. breaking of hemopoesis
b. gastrointestinal breaking
e. all mentioned above
73. Name preparations of rapid- acting insulin:
a. protafan NPH
b. humodar B
c. acktropide
d. lantus
e. humulin
74. Name preparations of intermediate-acting insulin:
a. protafan NPH
b. monotard NPH
c. humulin
d. insuman basal
e. all mentioned above
75. Which of the below mentioned preparations of
sulfanilurine belong to the second generation:
a. glibenclamide
b. glipizid
c. gliclazid
d. glicvidone
e. all mentioned above
76. What number of bread units is necessary for a
patient with an excessive mass of body?
a. 20-24 BU
b. 15-19 BU
c. 10-15 BU
d. 24-28 BU
e. 29-32 BU
98
77. The principal purpose of diabetes mellitus,
1 type treating is:
a. glycemia on the empty stomach 5,1-6,5 mmol/litre
b. glycosylated haemoglobin 6,1-7,5 %
c. normal physical condition of the patient
d. prophylaxis of specific diabetes mellitus complications
e. all mentioned above
78. What is the mechanism of metformin acting
connected with?
a. with an amplified glucose absorption by skeletal
muscles and with inhibition of the hepatic gluconeogenesis
b. with the increase produced of the quantity of the insulin
c. with stimulation of pancreas
d. with stimulation of hepatic gluconeogenesis
e. with reducing produced the quantity of the insulin
79. A 23 years old patient suffers from the diabetes
mellitus, 1 type. Disease was detected for the first time.
Complaints of dryness and thirst. Ketoacidosis is absent. What
dose of insulin a day does he need to normalize his
hydrocarbon metabolism?
a. 0,3 unit/kg/ a day
b. 0,1 unit/kg/ a day
c. 0,5 unit/kg/ a day
d. 1,5 unit/kg/ a day
e. 0,7 unit/kg/ a day
80. The patient, 55 years old, has suffered from the
diabetes mellitus of the 2 type for 10 years. Complains of
dryness, thirst and frequent excretion of urine. Has lost 8 kg of
his weight during 6 month. His father also suffered from the
diabetes mellitus/ During examination: glycemia on the empty
stomack – 11.2 mmol/litre. During 24 hours – from 15.2 up to
20 mmol/litre.glycosylated hemoglobin – 14%. The last 6 years
99
this patient was treated with glibenclamide: 1 tablet, 3
times a day. Your tactics of treatment:
a. to add behuanides
b to increase the dose of preparations of sulfanilurine
c to prescribe insulinotherapy
d to prescribe combined sugar reducing reparations
e. to prescribe another preparations of sulfanilurine
81. Patient, 28 years, is ill with diabetes during 5 years,
intakes insulin in daily dose 38U. What scheme of
insulinoterapy is indicated for patient with stable clinical
course of disease?
A. Traditional
B. Intensifyed
C. Only with insulins of short action
D. Only with insulins of the prolonged action
E. Some method from transferred is possible
The right answer: A.
82. Patient, 28 years, is ill with diabetes during 5 years,
intakes insulin in daily dose 38U. He was hospitalized with
signs of ketoacidosis. What scheme of insulinotherapy is
indicated for patient with ketoacidotic state?
A. Only with insulin of the short action
B. Intensifyed
C. Traditional
D. Only with insulins of the prolonged action
E. Some method from transferred is possible
The right answer: A.
83. Patient, 28 years, is ill with diabetes during 5 years,
intakes insulin in daily dose 38U. There were frequent
hypoglycemic and ketoacidotic states in anamnesis. Level of
100
the glycated haemoglobin 10,5%. What scheme of
insulinotherapy is indicated for patient with labile clinical
course of disease?
A. Intensifyed
B. Traditional
C. Only with insulins of short action
D. Only with insulins of the prolonged action
E. Some method from transferred is possible
The right answer: A.
84. Patient, 28 years, asked for help to endocrinologist
with the first diagnosed diabetes without the clinical signs of
ketoacidosis. Define the approximate daily amount of insulin
for this patient?
A. 0,5 U/kg
B. 0,7-0,9 U/kg
C. 1,0-1,5 U/kg
D. 50 U a day
E. 2 U on each 1 mmol/l of fasting glycemia
The right answer: A.
85. Patient (woman), 46 years, is ill with diabetes
mellitus during 9 years, inakes Humodar B - 26 U in the
morning and 18 U in the evening. She complains about
weakness, languor in the morning after sleep, headache,
hyperhydrosis in the night. Objectively: pulse-72/min., BP 125/70 mmHg. Cardiac borders are normal. Liver +4cm.
Glycemia: 8.00 - 14 mmol/l; 12.00 - 9 mmol/l; 17.00 - 11
mmol/l; 2.00 - 3,1 mmol/l. What are the reason of these
complaints?
A. Overdose of insulin in the evening.
B. Underdose of insulin in the evening
C. Presence of hepatosis
D. Climacteric syndrome
101
E. Underdose of insulin in the morning
The right answer: A.
86. Youth, 19 years, is ill with diabetes mellitus during
two years. Objectively: tongue is dry. There is angular
stomatitis. Diabetic blush. Liver +4cm, soft. Fasting glycemia 12,3 mmol/l. Glucosuria - 25g/l. There is increased level of
triglicerides in plasma. It is necessary for the prophylaxis of
complications of diabetes:
A. Rich of normal level of glycemia
B. Prescribe angioprotectors
C. Prescribe the А, Е, С D vitamins
D. Prescribe aldosoreductase inhibitors
E. Prescribe hypolipidemic drugs
The right answer: A.
87. Patient is ill with diabetes mellitus during 28 years.
Dose of insulin was decreased on 14U last year. Analysis of
urine: protein - 1,7 g/l, sugar - 0,8 %, many WBC, cylinders.
These indicated signs are the display:
A. Diabetic nephropathy
B. Insulinoresistance
C. Decompensation of diabetes
D. Pyelonephritis
E. Syndrome of the chronic overdosing of insulin
The right answer: A.
88. Patient is ill with diabetes mellitus type 1 during 28
years. What scheme of insulinotherapy uses by patient,
if he intakes 10U of Protaphan and 6U Aktrapid in the
morning, in the evening - 8U of Protaphan and 6U of Aktrapid?
A. Traditional
B. Intensifyed
C. Basis-bolus
D. DAFNE method
102
E. Untraditional
The right answer : A
89. Patient is ill with diabetes mellitus type 1 during 28
years. What scheme of insulinotherapy uses by patient, if he
intakes 12U of Aktrapid in the morning, 10U of Aktrapid in the
afternoon, 8U of Aktrapid and 12U of Protaphan in the
evening?
A. Intensifyed
B. Traditional
C. Combined
D. DAFNE Method
E. Untraditional
The right answer : A
90. A 25 years old woman, has went for the first time
with complaints of strong thirst, dryness in a mouth, frequent
urination, growing thin along with the raised appetite. These
complaints have appeared 2 months ago after her stressful
situation. During examination the diabetes mellitus was
revealed. What treatment is necessary to be administered to the
patient?
A. *Insulin therapy.
B. Medicines of sulphonylureas.
C. Biguanides.
D. Diabetic diet only.
Е. The patient does not require special treatment.
91. A 47 years old patient has been suffering from type 1
diabetes mellitus for last 5 years. The diabetes is at
compensation stage. Patient was been directed on sanatorium
therapy. What medical factors of a resort will help to improve a
condition of the patient?
А. *Mineral water.
В. Climatic factors.
103
С. Therapeutic mud.
D. Physiotherapeutic training.
Е. All listed above.
92. Patient В is 26 years old and he has been suffering
from type 1 diabetes mellitus. Uses insulin from group of
porcine insulins. 2 months ago there was atrophy of
subcutaneous fat in the places of insulin injections, especially
on hips (where the insulin injections were done more often).
What recommendations will help in eliminating the insulin
lipodystrophy?
A. *To use human insulin,
B. To change places of injections necessarily,
C. To inject the needle strictly perpendicular to the skin.
D. To heat up the insulin dose till it reaches the body
temperature before its use
E. All listed above.
93. Patient Р has been suffering from type 1 diabetes
mellitus for last 8 years. She went to an endocrinologist for
getting the directions to the sanatorium therapy.
What laboratory examinations will give the information
about the compensation stage of the diabetes?
A. *Level of glucose in blood on an empty stomach and
after meal (pp),
B. Level of glycosylated haemoglobin,
C. Level of total cholesterol and triglycerides,
D. The body mass index,
E. All listed above.
94. A patient entered the clinic with diabetes mellitus of
type 1, heavy form in the stage of indemnification, which
complicated nephropathy , myocardial dystrophy and stagnant
cardiac insufficiency of the II item Preparation of what group is
it expedient to plug in complex therapy of patient for the
104
prophylaxis of thromboses?
A.* Low molecular weight heparin ;
B. Fibrinolitics;
C. Plasminogen activators ;
D. Non-steroidal anti-inflammatory agent;
E. Indirect anticoagulants.
95. A patient, 27 years, is under surveillance of
therapist with a diagnosis diabetes mellitus type 1. He works
in the production carpenter, moderate physical activity. What
calculation of bread units on days does require patient in
accordance with his loadings?
A.* 30-35 kcal/kg in a day – 21 BU;
B. 40-45 kcal/kg – 30 - 35 BU;
C. 35-40 kcal/kg – 25-30 BU;
D. 20 - 25 kcal/kg – 14 BU;
E. Contraindicated work moderate.
96. Patient with diabetes mellitus type 1, 27 years old.
At present in a period of subindentification. The weight of
patient does not differ from ideal. Choose the necessary dose of
insulin from offered.
A.* 0,7 UN/kg of weight;
B. 0,2 UN/kg of weight;
C. 0,5 UN/kg of weight;
D.1,0 UN/kg of weight;
E. 1,5 UN/kg of weight.
97. For a patient, 22 years old , diabetes mellitus is set
type 1. Glukozuria makes 150 g on daily. Choose the necessary
dose of insulin on the level of glukosuria.
À.* 30 UN (1 UN insulin on 5 g of day's glukosuria,
150:5=30 UN)
B. 75 UN insulin (1 UN insulin on 2 g of day's
glukosuria, 150:2=75 UN)
105
C. 15 UN insulin (1 UN insulin on 10 g of day's
glukosuria, 150:10=15 UN)
D. 150 UN insulin (1 UN insulin on 1 g of day's
glukosuria, 150:1=150 UN)
E. 300 UN insulin (1 UN insulin on 0,5 g of day's
glukosuria, 150:0,5=300 UN)
98. For a patient, 24, diabetes mellitus type 1. Gets 40
UN insulin a day. Feeling is satisfactory. What criteria of the
right calculation of dose?
A. The absence of the syndrome of Samadhi;
B. The absence of the syndrome of «phosphor»;
C. The absence of the syndrome of hunger, sweating ,
cramps;
D.* All listed;
E. Nothing from listed.
99. The sick, 20 years old, after the carried Rubella is
set diabetes mellitus of type 1 . Weight of patient higher than
middle norm. From anamnesis of sick – sick likes sweets too
much. The grand-dad of sick also suffers diabetes mellitus.
Name a main provocative factor in development of this disease
for a patient
A. * Viral infection;
B. Genetic;
C. Violations in a diet;
D. Age of patient;
E. Obesity.
100. Patient L., 49 years old, height 163 cm, weight
76 kg, level of fast (on an empty stomach) glucose is 6,3-7,28,7 mmol/l.
What is the possible diagnosis?
A. Normal.
B. Impaired glucose tolerance.
106
C. Diabetes mellitus type 1
D. Impaired fasting glycemia.
E. *Diabetes mellitus type 2
101. An overweight 56 y-r-old woman complains of
having to go to toilet more frequently. She says she does
drink a lot of tea but that she is always thirsty and tired.
She needs the energy.
Prescribe investigations, please.
A. Fasting blood glucose*
B. Urinary ketones
C. T3, T4 and TSH levels
D. HbAlc levels
E. C-peptide levels
102. A 31-year-old Hispanic female is hospitalized with
polyuria, thirst, and blurred vision. Her is 161 cm (63 in)
tall and weighs 121 kg (265 lb). Acanthosis nigricans is
noted on his neck. Plasma glucose concentration is 17
mmol/l (323 mg/dL); serum ketones are negative and
arterial blood gases are normal. The patients grandmother
has type 2 diabetes mellitus.
What is the most likely cause of this patient’s
hyperglycemia?
A. Type 1A diabetes mellitus
B. Type 1 B diabetes mellitus
*C. Type 2 diabetes mellitus
D. Secondary diabetes
E. Maturity-onset diabetes of youth
103. A 50 year old male has type 2 DM. Which one of the
following statements about non-insulin dependent
diabetes mellitus (type 2 DM, NIDDM) is NOT true ?
A.
Circulating islet cell antibodies are usually
found*
107
B.
There is not HLA association
C.
Ketosis is rare
D.
Relative resistance to insulin is present
E.
Obesity is common
104. Diabetic N., 50 years old, takes metformin 3000
mg/day. Was founded level of fasting glucose – 6,9 mmol/l,
aglucosurea, angiopathy of lower extremities - functional
stage, nonproliferative retinopathy. Choose the degree of
severity of the diabetes mellitus, please.
A.
Mild.
B.
*Moderate.
C.
Severe
D.
Stabile.
E.
Progressive.
105. A 63-year-old woman presents with a fasting plasma
glucose level of 9,8 mmol/l and no ketones on routine
examination. She is asymptomatic and has no evidence of
microvascular or macrovascular disease. She is 157 cm (62
in) tall and weighs 51 kg (110 Ib). She takes no medications
and has no family history of diabetes.
What treatment is necessary in this case?
A. Diet therapy*
B. Insulin therapy
C. Does not need any treatment
D. Biguanides
E. Sulphonilurea medicines
106. Patient R., 45 years old, height 160 sm, weight
83 kg. 3 monthes ago was diagnosed DM type 2, used diet
therapy, and lost 5 kg of body mass. Fasting glucose – 12
mmol/l.
What is the next step in his treatment?
А. Sulfonilurea medicines I generation
108
B. Insulin therapy
C. Sulfonilurea medicines III generation
D.Biguanides*
E. Sulfonilurea medicines II generation
107. Patient P. 52 y.o., height 162 sm, weight 92 kg.
He is ill with diabetes mellitus type 2 during 4 years. Uses
diet therapy and metformin 3000 mg/day. Fasting glucose –
10,4 mmol/l, postprandial glucose – 12,8 mmol/l.
What treatment has to be used in this case?
А. To add sulfonylurea medicines I generation
B. To prescribe short-acting insulin
C. To prescribe semilong-acting insulin
D. Biguanides
E. To add sulfonylurea medicines iI generation*
108. Patient D. 65 years old, suffers from diabetes
mellitus type 2, got viral hepatitis A. During last 2 years
used Glibenclamid 15 mg/day. Fasting glucose – 13,6
mmol/l.
What treatment has to be prescribed?
А. To prescribe glimepirid (sulfonylurea medicine III
generation)
B. To add biguanides
C. To increase dose of Glibenclamid to 20 mg per day
D. To add Acarbosa
E. To prescribe insulin therapy*
109. Patient F. 62 years old is ill with diabetes
mellitus type 2. To receive compensation it is enough for
him to use diet and maninil (glibenclamid). He is going to
have operation of stomach resection.
What has to be changed in the treatment of DM?
А. To change maninil for glimepirid
B. To change maninil for glurenorm
109
C. To prescribe short acting insulin*
D. To prescribe long acting insulin
E. To prescribe biguanides
110. Patient K., suffering from Diabetes Mellitus for the
past 10 years, fell into a coma. His skin is dry, breathing is loud
and there is an acetone smell around him. Pulse 130 /minutes,
the arterial pressure 70/45 mmHg. The reaction of the urine to
acetone is positive (+++), the glucose of the blood - 30 mmol/1.
What type of coma is this?
A) ketoacidosis ;
B) hyperosmolar;
C) lactic acidosis;
D) hypoglycemic;
E) brain.
111. Patient X., 67 years old. Duration of Diabetes
Mellitus type 2 during 9 years. Patient keeps to the prescribed
diet, receives regular treatment with oral drugs, and several
times in the few last years was treated by sulfonylureas with
maximal therapeutic doses.
What complication should you consider?
A.
Hyperglycaemia
B.
Hypoglycaemia
C.
Diabetic ketoacidosis
D.
Hyperosmolality
E.
Lactic acidosis
112. Patient E, 24 years, is suffering from diabetes
mellitus type 1 during 3 years. Several days ago she canceled the
insulin. Patient lost consciousness in the evening. She is
hospitalized. The skin is dry. Kussmaul's respiration, the odor
of acetone from the mouth, tongue is dry, with fur of brown
colour. Pulse 120 /minutes, the arterial pressure 80/45 mmHg.
The abdomen does not react to the palpation. The liver is + 3 cm.
110
B.
C.
D.
E.
B.
C.
D.
E.
A.
B.
C.
D.
E.
The reaction of urine to acetone is positive, the glucose
of blood - 28 mmol/1. The preliminary diagnosis is:
A. Diabetic ketoacidotic coma
Lactacidemic coma
Infectious - toxic shock
Hepatic coma
Hyperosmolar coma
113.The unconscious woman of 25 years is delivered to
hospital. She is suffering from Diabetes Mellitus for the past 8
years. In the morning the patient could not wake. The skin is
dry, the turgor is reduced, eyeballs are mild. Kussmaul's
respiration, the odor of acetone from the mouth. The pulse - 130
/minutes, the arterial pressure - 100/65 mmHg. The liver is +2
cm. The glucose of blood- 25,5 mmol/1. The emergency treatment will consist of introduction:
A.Rapid-acting insulin in the dose of 10 U hourly i/v
Insulin of prolonged acting 10 U hourly i/v
40% glucose solution
Reopolyglukin up to 1,0 L
0.9% NaCl solution
114. Patient C, 24 years with the 10-years experience of
diabetes type 1 is unconsciousness. He has lost consciousness
after intensive physical activity. Objective: a patient is pale, the
skin is wet, tongue is wet too. Respiration is superficial, the
arterial pressure - 140/70 mmHg. Tone of muscles and tendon
reflexes are raised. Establish the diagnosis:
Hypoglycemic coma
Lactoacidotic coma
Hyperosmolar coma
Diabetic ketoacidotic coma
Cerebral coma
115. The patient with diabetes type 1, the severe form, is
111
B.
C.
D.
E.
F.
B.
C.
D.
E.
compensated. After game of football he has lost
consciousness. Objective: a patient is pale, the skin is wet,
tongue is wet too. Respiration is superficial, the arterial pressure
- 110/70 mmHg. Tone of muscles and tendon reflexes are raised,
meningeal signs are absent. The emergency treatment will
consist of introduction:
40-80 ml 40% glucose solution
Reopolyglukin up to 1,0 L
Rapid-acting insulin in the dose of 10 U hourly i/v
0.45% NaCl solution
0.9% NaCl solution
116. One of the main causes of hypoglycaemia is
A.
Diarrhoea
B.
Stress
C.
Weight loss
D.
Weight gain
E.
Unaccustomed exercise
117. The patient of 63 years suffers from insulindependent form of diabetes. On the background of acute
infringement of the cerebral circulation the high hyperglycemia56 mmol/1, severe dehydration, hyper-chloremia, hypernatremia
has developed. The ketonemia and acetonuria are absent.
Establish the correct diagnosis:
A
hyperosmolar coma
Diabetic hyperketonemic coma
Acute renal failure
Hyperlactacidemic coma
Chronic renal failure
118. Point out to the features which are characteristic
for the 3 stage of the diabetic nephropathy:
a. proteinuria
b normoalbuminuria
112
c. hyperfiltration
d. mikroalbuminuria
e. hyperperfusion
119. Point out to the features which are characteristic
for the diabetic proliferative retinopathy:
a. hemorrhages into the vitreus body
b. rubeosis
c neovascularization of the disk of the optic nerve
d. exfoliation of the retina
e. all mentioned above.
120. What changes on the eye bottom must take place
in case of the preproliferative diabetic retinopathy to make
lasercoagulation necessary.
a. all mentioned below
b. sinuosity of the vessels
c. uneven expansion of the venulae
d. soft exudates
e. There are a lot of microaneurysma and retinal
hemorrhages.
121. To the neuropathic form of the syndrome of
diabetic foot belong:
a. acral micosis
b. pale skin
c. absence of pulsation on the feet’s arteriae
d. cyanosis of skin
e. dry skin, areas of hyperkeratosis
122. The duration of treatment for a patient having the
third stage of ulcerous lesions must be:
a. from 2 weeks to 1,5 month
b. from 2 to the 3th months
c. from 6 to 12 months
d. from 4 to 5 months
113
e. from 5 to 6 months
123. The patients who belong to the risk category with
the neuropathy and the ischemia of extremities need
examination:
a. once in 6 months
b. once in 9 months
c. once in 1-3 months
d. once in 12 months
e. once in 24 months
124. Treatment the diabetic autonomous neuropathy
includes:
a. adequate sugar reducing therapy
b. sulfur-containing preparations
c. neurotropic vitamins
d. using elastic high socks
e. all mentioned above
125. The basic principles of treatment of the trophic
lesions of lower extremities for patients suffering from diabetes
mellitus are:
a. optimization of the metabolic control
b. antibakterial therapy
c. local treatment of the ulcer
d. all mentioned above
e. unloading of the wounded extremity
126. Conservative treatment of the diabetic retinopathy
is accomplished on the account of:
a. keeping to dietotherapy
b. all mentioned
c. adequate sugar reducing therapy
d. correcting the hemodynamics of retina
e. correction the hyperlipidemia
114
127. To treat patients suffering from the
diabetic nephropathy on the stage of microalbuminuria the
following must be done:
a. correction of hydrocarbonic metabolism
b. correction of the remal hemodynamics
c. correction of the arterial tension
d. correction of the hyperlipidemia
e. all mentioned above
128. Patient (woman), 26 years. She is ill with diabetes
mellitus type 1 during 5 years and intakes insulinotherapy. She
complains about hyperhydrosis, sense of hunger, palpitation in
bedtime after last hospitalization during last half a year. She
put on yhe weight on 4kg during last 3 months. Glycemia: 8.00
- 12mmol/l; 12.00 - 9mmol/l; 17.00 - 8.5mmol/l; 2.00 2.9mmol/l. Specify possible complication of insulinotherapy?
A. Somoji's syndrome
B. Lipodystrophy
C. Intolerance to insulin
D. Insulin edemata
E. Insulinoresistance
The right answer : A
129. Patient, 48 years, is ill with diabetes mellitus
during 8 years. He intaked metformin 850mg twice a day. The
last three months he notes increase of intensity of diabetic
complaints, fasting glycemia - 9-12mmol/l. Patient lost 6kg of
body mass during these time. This is decreased level of Cpeptide in blood. What medical tactic is indicated to this
patient?
A. Prescribe insulin
B. To double the daily dose of metformin
C. To appoint thiazolidindiones
D. To cancel metformin and prescribe sulphonylureas
115
E. The state of patient is not related with
diabetes
The right answer: A.
130. Patient, 48 years, is ill with diabetes mellitus
during 8 years. He intaked metformin 1000mg twice a day.
The last three months he notes increase of intensity of diabetic
complaints, fasting glycemia - 9-12mmol/l. Patient lost 2kg of
body mass during these time. This is normal level of C-peptide
in blood. What medical tactic is indicated to this patient?
A. To add insulin secretion stimulatos
B. To double the daily dose of metformin
C. To appoint short action insulin
D. To cancel metformin, to prescribe a-glucosidase
inhibitors
E. The state of patient is not related with diabetes
The right answer: A.
131. Patient, 48 years, is ill with diabetes mellitus
during 8 years. He intaked metformin 1000mg twice a day.
Fasting glycemia – 8-9 mmol/l. He prepares for surgery of
cholelitiasis. What tactic of treatment of diabetes during
surgical treatment is need for this patient?
A. To transfer on short action insulin
B. To double daily dose of metformin
C. To prescribe sulphonylureas
D. To cancel metformin, to prescribe a-glucosidase
inhibitors
E. To leave treatment without changes
The right answer: A.
132. Patient, 48 years, is ill with diabetes mellitus
during 8 years. He intaked metformin 1000mg twice a day.
Fasting glycemia – 8-9 mmol/l. He was hospitalized with
diagnosis macrofocal pneumonia. Which adverse events may
116
A.
B.
C.
D.
E.
be in treatment by metformin?
A. Lactatacidosis
B. Hypoglycemic coma
C. Abscess formation in lungs
D. Pneumothorax
E. Pulmonary embolism
The right answer: A.
133. Diabetes mellitus was diagnosed in patient, 48
years. Fasting glycemia – 8-9 mmol/l. Which from listed states
insulinotherapy is not indicated to this patient in?
A. First diagnosed diabetes. Obesity III degree
B. Diabetes mellitus type 1, moderate severity,
compensation. Pregnancy
C. Diabetes mellitus type 1, moderate severity,
decompensation. Diabetic ketoacidosis
D. Hematologic diseases
E. Diabetes mellitus type 2. Acute myocardial
infarction
The right answer: A.
134. Diabetes mellitus was diagnosed in patient, 48
years. Fasting glycemia – 8-9 mmol/l. Which from listed states
oral sulphonylureas are indicated to this patient in?
Diabetes mellitus type 2, moderate severity. Diabetic
angioretinopathy 1degree
Diabetes mellitus, mild form, compensation
Diabetes mellitus type 1, compensation. Diabetic nephropathy,
nephrotic stage
Diabetes mellitus type 2, moderate severity, compensation.
IHD, acute myocardial infarction, heart failure 1 functional
class
Diabetes mellitus type 1, moderate severity, compensation in
cholecystectomy
117
A.
B.
C.
D.
E.
The right answer: A.
135. Diabetes mellitus was diagnosed in patient
(woman), 39 years. Fasting glycemia – 8-9 mmol/l. Which
from listed states oral sulphonylureas are indicated to this
patient in?
Extraction of a tooth
Cytopenic states of any genesis
Acute hepatitis
Lactation and pregnancy
Severe exacerbation of chronic infection
The right answer: A.
136. What is the main indication to prescribing of
sulphonylureas?
A. Diabetes mellitus type 2 in adults
B. Juvenile diabetes mellitus
C. Labile clinical course of diabetes mellitus
D. Diabetes mellitus type 2 in pregnant
E. Diabetic ketoacidosis
The right answer : A
137. What from these preparations increase utilization
of glucose in peripheral tissues?
A. Metformin
B. Trental
C. Glibenclamide
D. Nicotine acid
E. Аcarbose
The right answer : A
138. A 65 years old man (of 169 cm height and 84 kg
weight) has come to polyclinic to an oculist with complaints to
aggravation of vision. Type 2 Diabetes mellitus was revealed
after examination. What medicine for treatment of Diabetes
Mellitus are the most rational for this patient?
118
A.* Biguanides.
B. Insulin.
C. Medicines of sulphonylureas.
D. Thiazolidinediones.
E. Inhibitors of glucose absorption UNthe gastrointestinal
tract (α-glucosidase inhibitors).
139. A patient of type 2 diabetes mellitus has been taking
Metformin (Dymethylbiguanide) for 3 years, drank alcohol
during a friendly meeting. After this he felt accrued general
weakness, nausea, muscle pain, palpitation. There was a
unitary vomiting. What is the most probable reason of these
symptoms?
A. *Development of hidden lactate-acidosis.
B. An alcoholic poisoning.
C. Overdose of biguanides.
D. An insufficient dose of biguanides.
E. Development of a reactive pancreatitis.
140. Woman D, being 68 years old, has been suffering
UNtype 2 diabetes mellitus and receiving Gliclazide for last 8
years. Level of glycemia is increased, the patient has lost
weight. What an endocrinologist’s tactic can be in this case?
A. *To discontinue Gliclazide and to administer with
insulin.
B. To increase the dose of Gliclazide.
C. To administer Biguanides along with Gliclazide.
D. No change in the treatment.
E. To administer Thiazolidinediones along with
Gliclazide.
141. A patient with type 2 diabetes mellitus orally takes
Glimepiride 5 mg daily and has got a good results. Patient has
got cholelithiasis (a biliferous disease). Symptoms of
mechanical jaundice arose 5 days ago. The conservative
119
therapy of jaundice has not given its results yet.
Cholecystectomy should be done on the patient. What an
endocrinologist’s tactic can be for use of Glimepiride in this
case?
A. *To discontinue the drug during the treatment at
surgical department.
B. To increase the dose of Glimepiride up to 8 mg a day.
C. To decrease the dose of Glimepiride up to 3 mg a day.
D. To administer biguanides with it.
E. No change in the treatment.
142. A patient, 45 years, is observed during 3 years
concerning diabetes mellitus type 2. What must be criteria of
indemnification (in accordance with offered the European
group of policy of treatment of diabetes)?
A. Absence of glucosuria , absence of ketosis , and the
hypoglycemia states;
B. Glycemia on empty stomach : adequate level - 5,1–
6,5 mmol/l, satisfactory - to 7,8 mmol/l, glycemia after a meal:
adequate level - 7,6–9 mmol/l, satisfactory - to 10,0 mmol/l;
C. Level glycosylated HbAlc – to 6,1-7,5 %, table of
contents of cholesterol - to 6,5 mmol/l;
D.* All of the above-mentioned;
E. Nothing UNthe above-mentioned.
143. A 55-year-old type 2 diabetic patient has lost
weight and has had good control of his blood sugar on oral
agents. He has a history of mild hypertension and
hyperlipidemia. He asks for advice about an exercise
program. Which one of the following statements is correct?
A. A stress test should be recommended prior to beginning an
exercise program
B. Exercise should be avoided because it may cause foot trauma
C. An active lifestyle cannot slow the complications of diabetes
120
D. Vigorous exercise cannot precipitate hypoglycemia
E. Start an exercise program an any cases
144. A newly diagnosed type 2 diabetic patient asks for
clarification about dietary management. Which of the
following is good advice?
A. Less than 10% of caloric intake should be saturated fat
B. Restrict carbohydrates and eat a high-protein diet
C. Avoid sucrose altogether
D. Caloric intake should be very consistent from one day to
another
E. Eat a high-carbohydrates diet
145. At sick Н, 44 years, are adiposity of 2 degrees, an
arterial hypertensia of II stage. Has addressed to the
gynecologist concerning an itch in perineum. As a result of
inspection the diabetes 2 types is revealed. There are initial
displays of an angiopathy and a polyneuropathy. Choose the
most rational therapy for the given patient.
А.* Diabeton MR - 30 mg once a day;
B.Humalog - 10 UN in day;
C.Novorapid - 20 UN in day;
D.Humodar - 30 UN in day;
E. Sofor of 500 mg 2 times a day.
146. At patient К, 50 years, are a diabetes 2 types the
easy form, growth - 165 sm, weight - 70 kg. Choose, that
corresponds to principles dietotherapy for the given patient.
A. Limitations easy digestible carbohydrates;
B. Valid receipt of fats, fibers, carbohydrates;
C. Except sugar, sweets, honey, мороженное;
D. Use of sugar substitutes - sorbite, ксилит, fructose,
steviosid;
E *All aforesaid.
121
A.
B.
C.
D.
E.
A.
B.
C.
D.
147. At patient М, 50 years, the diabetes 2 types
of average weight is revealed. There is postprandeal
hyperglycaemia. Choose a preparation from offered for
monotherapy and elimination of this complication
А.* It is novo-norm;
B. Metformin;
C. Glibenklamid;
D. Insulin;
E. Gljurenorm.
148. Patient B., 60 years, has the type 2 of DM for 14
years. Complaints on headache, itching of the skin,
appearing of the edema on the legs and face. He has
hypertension during 6 years. In the urine test: proteinurea
is 2,99 gm/l, glucosurea is 20 gm/lt. the level of creatinine is
0,27 mmol/l, urea – 14,3 mmol/l. Fasting serum glucose
level is 12,1 mmol/l.
What oral hypoglycaemic agent have you to choose for
the patient?
maninil
chlorpropamid
metformin
*glurenorm
acarbose
149. Diabetic H., 44 years old, admitted to the
endocrinology department with complains of weakness,
polydypsia, dyspnea. Objective examination reveals dry
skin and tongue, Kuss – Maul’s breath.
What investigation you have to prescribe?
Daily glucosurea
Glucose tolerance test.
The level of postprandial glucemia.
*The level of acetone in urea
122
A.
B.
C.
D.
E.
E. The level of bilirubinemia
150. Diabetic F., 52 years old, takes maninil 3,5 mg
twice a day. After activization of chronic bronchitis began
to take biseptol 480 mg (2 tabl. twice a day). He complains
of increased appetite, tremor of fingers, headache.
What is your previous diagnosis?
Allergic reaction
Lactic acidosis
Diabetic ketoacidosis
*Hypoglycemic status.
Hyperosmolar coma
151. Patient P., 34 years, was hospitalized. She is
unconscious. Her relatives told you, that she has type 1 DM
and felt herself bad 3 days ago after influenza, when she
decreased the dose of insulin. She had weakness, fatigue,
nausea and vomiting. Examination: dry skin and tongue,
hyporeflexia. Laboratory findings: blood glucose level – 29
mmoll/l, acetone in urine is positive.
What is your previous diagnosis?
A. Hypoglycemic coma
B. *Diabetic ketoacidosis
C. Infectious endocarditis
D. Hyperosmolar coma.
E. Lactic acidosis
152. A 32-year-old female, insulin dependent
diabetic, presents with failure to pass urine.
Which complication of diabetus can be found?
A. Hyperglycaemia
B. Hypoglycaemia
C. Urinary tract infection
D. Kidney calculi
E. Autonomic neuropathy*
123
A.
B.
C.
D.
E.
A.
B.
C.
D.
E.
153. Patient F., 24 years old, has diabetes
mellitus during 14 years complains of tachycardia, which
can’t treated by β-blockers. During physical examination
was found blood pressure: 190/110 mm of Hg in horizontal
position and 80/ 60 mm of Hg in vertical position.
What is your previous diagnosis?
Diabetic peripheral neuropathy
Thyrotoxicosis
Hypertensive disease
Myocarditis
*Diabetic cardioneuropathy
154. Diabetic R., 46 years old complains of changes
of the foot: it becomes shorter and wider. During
examination was found painless swelling of the feet without
edema or signs of infection, external rotation, and
flattening of the longitudinal arch.
Which complication of diabetus can be found?
*Diabetic arthropathy.
Diabetic neuropathy.
Diabetic angiopathy.
Diabetic foot.
Diabetic myelopathy
155. Patient K., 31 years old, has diabetes mellitus
during 7 years, takes insulin (54 Units) in 2 injections. Was
found such changes on retina: microaneurtisms, dilation of
vessels, haemorrhages, soft and hard exudates.
What stage of retinopathy does patient have?
Background retinopathy.
*Preproliferative retinopathy.
Proliferative retinopathy.
Retina is normal.
Nonproliferative retinopathy.
124
A.
B.
C.
D.
E.
A.
B.
C.
D.
E.
156. Patient B., 41 years, has the type 1 of
DM for 16 years. Complaints on periodical hypertension
(during the last year), appearing of the edema on the legs
and face. In the urine test: proteinurea is 0,98 gm/l,
glucosurea is 30 gm/l. Creatinine, urea within the pale of
norm. Fasting serum glucose level is 16,1 mmol/l.
What is your diagnosis?
*DM type 1, severe degree, stage of the subcompensation.
Diabetic nephropathy, III stage.
DM type 1, moderate degree, stage of the subcompensation.
Diabetic nephropathy, III stage
DM type 1, severe degree, stage of the compensation. Diabetic
nephropathy, III stage
DM type 1, severe degree, stage of the subcompensation.
Diabetic nephropathy, IV stage
DM type 1, moderate degree, stage of the subcompensation.
Diabetic nephropathy, IV stage
157. Diabetic R., 49 years old, complains of
symmetrical sensory loss, pain at night and during the rest,
hyporeflexia, decreased response touch, burning of heels
and soles. The skin becomes atrophic, dry and cold.
What is your previous diagnosis?
*Diabetic distal polyradiculoneuropathy.
Diabetic truncal polyradiculoneuropathy.
Diabetic truncal monoradiculoneuropathy.
Diabetic myelities.
Diabetic myelopathy.
158. Woman 59 years old, has been illed by Diabetes
Mellitus during 20 years. Treated oneself by the oral
preparations that reduce the level of sugar. There is diabetes in
the state of decompensation. Does not adhere to the diet.
Stomach-aches and swelling are disturbed, emptying is liquid,
125
a.
b.
c.
d.
e.
f.
g.
h.
i.
j.
their incontinence. A skin is dry, with easy hyper
pigmentation. The tongue is red, ”geographical”. A stomach
hurts at palpation. The lower edge of liver comes forward on 4
centimeters from under a costal arc. Your previous diagnosis?
Diabetic enteropathy
Cholecystitis
Diabetic hepatosis
Hepatitis
Colitis
159. Patient 42 years old is ill by Diabetes Mellitus type
1, entered permanent establishment with with complaints about
the loss of the weight, growing the weakness. He ills by
diabetes for a year. He treats by insulin in a dose 12 OD
Humudari R of rapid actuion and 18 OD Humudari B of
prolong action. Diabetes is stable, hypoglycemia is not present.
At a review: considerable grow thin, especially muscles.
Pulsation on the distal parts of arteries of the feet is good, the
trophic changes are not exposed. Achilles' and knee’s reflexes
are considerably slow from the both sides. Vibration, tactile,
thermal sensitiveness are not damage. Your previous
diagnosis?
Diabetes Mellitus type 1 in the stage of decompensation
Diabetes Mellitus type 1, asthenia of somatogene.
Diabetes Mellitus type 1, peripheral polyneuropathy
Diabetes Mellitus type 1, total motor polyneuropathy
Diabetes Mellitus type 1, total sensor polyneuropathy
160. Patient 38 years old is ill by Diabetes Mellitus type
1, complains about impotence and the loss of the weight,
progressing the weakness. . He ills by diabetes for 12 years.
Metabolic compensation of disease is good. Independently
actively watches on the glucose level in the blood.Ttreats
oneself by insulin in the 4th injections in a dose 36 OD per day.
126
a.
b.
c.
d.
e.
f.
g.
h.
i.
j.
The symptoms indicated earlier began to disturb
during a year. At a review: large loss of the weight. The step is
slow. The abdominal and cremasters reflexes are sickly.
Vibration, tangent sensitiveness are reduced. Hyperesthesia of
the skins is on the thighs. Your previous diagnosis?
Diabetes Mellitus type 1, autonomous neuropathy
Diabetes Mellitus type 1 in the stage of decompensation
Diabetes Mellitus type 1, chronic sensory neuropathy
Diabetes Mellitus type 1, peripheral polyneuropathy
Diabetes Mellitus type 2, in the stage of decompensation
161. Patient 56 years old is ill by Diabetes Mellitus type
2, complains about dizziness, darkening in the eyes at the
getting up. These phenomena accompany perspiration on the
face, neck, thorax. The reception of the meal does not give the
facilitation. He ills by diabetes for 42 years. The last year he
adopted Mannil in a dose 1 or 2 pills per day. Compensation of
the disease is satisfactory. The indicated complaints coincided
with resistance tachycardia. AP – 140/95 mm.hg., standing –
90/60 mm.hg. Your previous diagnosis?
Diabetes Mellitus type 1. Diabetic vegetative neuropathy of a
heart
Diabetes Mellitus type 1, climacteric syndrome. Dismetabolic
cardiopathy
Diabetes Mellitus type 1. Diabetic miocardiopathy
Diabetes Mellitus type 1,atherosclerotic cardiosclerosis
Non of this variant.
162. Patient 52 years old, is ill by Diabetes Mellitus
type 1, complains about about the diarrheas which disturb her
mainly at night, is carried the imperative character. The amount
of emptying arrives at 30 or 40 for a night.She feels discomfort
from the side of digestion organs during 2 years. Disturbed
swelling in a epigestrial area, periodic obstipations and
127
a.
b.
c.
d.
e.
a.
b.
c.
d.
e.
emptyings, that was interpreted as chronic pancreatitis.
Adopted enzymic preparations without the special success. He
ills by diabetes for 18 years. She treats by the insulin monotard
in a dose 46 OD in the morning. The fasting level of the
glucose in scopes from 11 to 13 mmol/L. Without regard to
emptyings she doesn’t have the loss of weight. Your previous
diagnosis?
autonomous enteropathy
ulcerous colitis
syndrome of the malabsorbtion
syndrome of irritation of the colon
sharpening of the chronic pancreatitis
163. At a patient with the heart attack of myocardium
the level of the glucose is 8.2 mmol/L. 2 years passed from the
beginning of the disease. What is the most expedient
inspections to appoint the patient for estimation of the state of
carbohydrate exchange?
Determination level of the Hb-alc. proteins
Test of the tolerance to glucose
Determine the fasting level of the glucose during 3 days
Determination of day's glucoseurine
Determination level of the glucose in the blood for a day
164. A patient K., 36 years old, which complains on the
headache, eyestrain, loss of appetite. He is ill during 18 years
on the diabetes mellitus 1. During the examination: pale, swell
face, heart increase in the left sight on 1,5 sm. Pulse – 70/min,
rhythmical, pressure 180/100 mm.hg.. Stomach is soft, liver
doesn’t increased. Pasternackyj symptom’s is negative with
both sight. Urination – accelerating, mainly in the night.
Analysis of the blood: Hb 98 g/l, er. 3* 1012 /L.lk. 8,2 *109 /l,
urea 15,8 mM/l. Cholesterin 7,6 mM/l, albumen – 81 g/l,
creatynine 177 mcM/l, glucose 8,2 mM/l.
128
F.
G.
H.
I.
J.
A.
B.
C.
D.
E.
Analysis of urin : specific gravity 1010,
albumen 0,6 g/l, er. 1-3, l. 10-15, glucose 2 g/l, ECG:
hypertrophy of the left ventricle. Your preliminary diagnosis?
Diabetes mellitus 1, hard form in the stage of decompensation.
Diabetic nephropathy.
Diabetes mellitus 1, high pressure disease, stage 3.
Chronic glomerulonephritis.
Diabetes mellitus 1, hard form in the stage of compensation.
Chronic pyelonephritis, high pressure disease.
Non of this variant.
165. Which regime of insulinotherapy is the most
correct for a patient, who is ill by diabetes mellitus and which
is complicated by encephalopathy?
A. Injection by insulin of long – term action in
the morning and addition
injection by insulin
short – term action before taking food.
B. One take injection by insulin of long – term
action.
C. Repeated injection by insulin of rapid – term
action.
D. Two injection by insulin per day.
E. Non of this variant.
166. Which is the most effective method of
prophylactic proliferous retinopathy?
Laser photocoagulation.
Courses of anticoagulants.
Anticoagulants straight action.
Courses of angioprotectors.
Anabolic steroids.
167. In a boy of 5 years, after measles vaccination a
temperature of the body is 37,8 0C, a big thirst, polydipsia,
polyuria. Weakness is increasing during 6 hours from the
129
A.
B.
C.
D.
E.
beginning of the disease, noisy breathing, sleepy. Your
preliminary diagnosis?
Diabetes mellitus.
Postvaccination syndrome.
Typical postvaccination reaction.
Postvaccination allergy.
Non of this variant.
168. In case of hypoglycemic coma following remedies
must be immediately prescribed:
a. short-acting insulin
b. biguanides
c. sulfanilurine preparations
d. glucose
169.To help the patient out of the hyperosmolar coma
besides insulins the following remedies must be prescribed
using intravenous droppering:
a. hypotonic Na chloride solution
b. isotonic Na chloride solution
c. 4% Na hydrocarbonate
d. 10% Na chloride solution
170.With the hypoglycemic coma the following can be
revealed:
a.
Kussmaul’s respiration
b.
acetone odor in the expired air
c.
blood omolarity increases
d.
normal state of the eye apples
e.
glucosuria
171.A 9 year old girl has been suffering from diabetes
mellitus for 3 years. The patient has fallen ill with acute
respiratory infection. Her state became worse during the last
two days, the temperature reached 38 C, the thirst and the
frequency of urination increased. General weakness enlarged.
130
Nausea and vomiting appeared. General condition is
grave. Consciousness is dizzy. Rubeosis. The tungue is dry.
Heart tones are rythmic, accelerated. What is your diagnosis?
a.
hypoglycemic coma
b.
ketoacidotic coma
c.
hyperosmolar coma
d.
pneumonia
172.To the number of the main insulin effects we refer:
a.
stimulating glucagone synthesis
b.
lithogenesis stimulating
c.
stimulating proteon synthesis
d.
all mentioned above
173.The patient K, of 56 years old, has suffered from
the diabetes mellitus for 8 years. Owerweight of her body is 8
kg. The state of her health become much worse after she had
enterocolitis (vomiting, diarrhea). She was urgently
hospitalized. Consciousness is absent. Expired air does not
contain the odor of acetone. Breathing perfunctory accelerated,
skin and mucous coats dryness. Lowering the eyeballs’ tone.
The diagnosis will be confirmed with determining:
a.
the level of sugar in the blood
b.
the level of sugar in the urine
c.
osmolarity
d.
all mentioned above
174.With a differential diagnostics between the hyper
and hypoglycemic comas the following should be taken into
account:
a.
the beginning of the coma
b.
apple eyes tone
c.
Kussmaul’s respiration
d.
acetone odor in the air
e.
all mentioned above
131
175. The patient M. suffers from the diabetes
mellitus from the age of 6. When brought into the department,
her state was grave. A day before, after the diet breaching
vomiting and nausea appeared. Because of the lack of appitite,
mother injected half a dose of insulin. The girl drank cold tea.
The following day the morning dose of insulin was missed and
the girl was brought into the endocrinological department. The
child is somnolent, her consciousness is intricate. There is stiff
respiration in the lungs. Heart sounds are rhythmic,
accelerated. Her liver is palpated 4 cm lower than the costal
arch. Your preliminary diagnosis is:
a.
hyperosmolar coma
b.
lactacidemic coma
c.
hypoglycemic coma
d.
ketoacidotic coma
176. For the lactacidemic coma is not characteristic:
a.
Kussmaul’s respiration
b.
tachicardia
c.
hypotonia
d.
oliguria, anuria
e.
hyperthermia
177. For helping the patient out of the lactacidemic
coma it is necessary to inject intravenously:
a.
physiological salt solution
b.
Na bicarbonate solution
c.
5% glucose solution
d.
all mentioned above
178. What from these states are indicated for
prescription of insulinotherapy in patients with diabetes
mellitus type 2?
A. All it's true
B. Proliferative angioretinopathy
132
A.
B.
C.
D.
E.
A.
B.
C.
D.
E.
C. Ketoacidotic state
D. Myocardial infarction
E. Aorto-coronal shunting
The right answer : A
179. Patient, 24 years, is ill with diabetes mellitus
during 8 years. He uses insulinotherapy. He was hospitalized
unconscious. Which from listed actions is need for treatment of
hypoglycemic coma?
IV stream introduction of 40% glucose
IV droply introduction of 10% glucose
IV droply introduction of 40% glucose
IV droply introduction of 5% glucose
IV droply introduction of 40% glucose and 6-8U of insulin
The right answer: A.
180. Patient, 54 years, is ill with diabetes mellitus
during 8 years. He uses insulinotherapy. He was hospitalized
with glycemia 45 mmol/l unconscious. Which from listed
symptoms are not typical for hyperosmolar coma?
Ketoacidosis
Local neurological symptomatic
High hyperglycemia
Dehydration
Hypopnoe, tachypnea
The right answer: A.
181. Patient, 63 years, is ill with diabetes mellitus type
2. High hyperglycemia, sharp dehydration, hyperchloremia,
hypernatriemia were developed during acute stroke. Ketonemia
and acetonuria are not present. Define true diagnosis:
A. Hyperosmolar coma
B. Acute renal failure
C. Hyperketonemic diabetic coma
D. Chronic renal failure
133
E. Hyperlactatacidemic coma
The right answer: A.
182. Coma developed in man, 25 years, that is ill with
diabetes mellitus during 8 years. Objectively: a skin is dry,
turgor is reduced, Cussmaul's breathing's present, BP - 105/60
mmHg, HR -116/min, smell of acetone in mid air. What is the
type of coma in this patient?
A. Ketoacidotic
B. Hyperosmolar
C. Lacatacidotic
D. Hypoglycemic
E. Cerebral
The right answer: A.
183. Coma developed in man, 28 years, with
bronchopneumonia. Objectively: HR - 122/min, extrasystoles.
BP - 80/45 mmHg. Skin is dry, turgor is reduced. Breathing is
deep, noisy, liquid. Strong smell of acetone in mid air. Liver
+5cm. Glycemia -32 mmol/l. рН of blood - 6,9. What solution
is most directed to operate on normalization of metabolism?
A. 4,2% solution of sodium hydrocarbonatis
B. 5% solution of glucose
C. 0,9% solution of sodium chloride
D. 1% solution of potassium chloride
E. Rheopolyglukin
The right answer: A.
184. Woman, 18 years, is ill with diabetes mellitus
during 5 years. She intakes 36U of insulin a day. The state
became worse sharply after start of pneumonia : thirst
increased, abdominal pain developed, nausea, vomits,
somnolence increased considerably. Patient refused to meal in
the evening, did not get evening dose of insulin, and she lost
consciousness in the morning . Objectively: unconscious, a
134
skin is dry, turgor is reduced. Tongue is dry, breathing
is noisy and deep, strong smell of acetone from a mouth. Body
temperature - 36,6 °С, pulse - 100/min, BP - 90/50mmHg. In
urine positive reaction on an acetone is present. Glycemia 33mmol/l. What is previous diagnosis?
A. Ketoacidotic coma
B. Hyperosmolar coma
C. Lactatacidemic coma
D. Hepatic coma
E. Cerebral coma
The right answer: A.
185. Patient was hospitalized unconscious. He is ill with
diabetes mellitus during 5 years. He intakes prolonged action
insulin in dose 24U in the morning and 18U in the evening.
Suddenly he lost consciousness. A skin is moist, tone of
muscles of extremities is promoted. Tone of eyeballs is normal.
Ps - 96/min, BP - 120/80mmHg. Cardiac tones are normal.
Breathing is rhythmic. Language is moist. Менінгеальні
symptoms are absent. What emergency therapy is need?
A. Introduction of 40% glucose solution
B. Introduction of short action insulin
C. Introduction of 4% sodium hydrocarbonatis solution
D. Introduction of hydrocortizone
E. Introduction of adrenalin subcutaneously
The right answer: A.
186. Whatever feature is not typical for myocardial
infarction in patients with diabetes mellitus?
A. It is accompanied by development of hypoglycemia
B. Q-miocardial infarction
C. Absent ECG-signs of myocardial infarction
D. Absent increase level of markers of myocardial
necrosis
135
E. Absent development of complications of
myocardial infarction
The right answer : A
187. What is the typical sign of Kimelstil-Wilson's
syndrome?
A. Nephrotic syndrome
B. Strong hypertension
C. Proteinuria, cilinderuria
D. Leukocyturia
E. Microalbuminuria
The right answer : A
188. Specify the early marker of diabetic nephropathy:
A. Microalbuminuria (30-300mg/day)
B. Proteinuria (more than 300mg/day)
C. Hypertension
D. Decreasing of glomerular filtration rate less
10ml/min
E. Glucosuria (more than 10g/l)
The right answer : A
189. A family doctor had been called to a patient with
type 1 diabetes mellitus. Apparently: the patient is adynamic (a
condition of deep sleep), he comes out of it only under
intensive stimulus, noisy respiration, the skin is dry, retracted
abdomen. Members of family told, that patient’s condition has
gone worse for last 2 days. What is the tactic of the family
doctor here?
A. *Urgent hospitalization of the patient to a hospital.
B. To organize a treatment at home, in order to observe
rehydratation.
C. To prescribe to the patient plentiful drink.
D. To seek for an endocrinologist’s consultation at the
patient’s home (for taking the decision about his further
136
treatment).
E. To make patient’s examination the next day.
190. A 72 years old man, has come to a surgeon in
polyclinic complaining a pain in the lower extremities before
which he felt numbness and crawling of bugs under the skin
there over and at night especially having cramps in
gastrocnemius muscle. Diabetes mellitus was found out during
examination. What complications of diabetes mellitus can take
place over here?
A. *Diabetic neuropathogenesis.
B. Diabetic macroangiopathogenesis.
C. Diabetic microangiopathogenesis.
D. Plexopathogenesis.
E. Radiculopathogenesis.
191. A woman being 59 years old, has been suffering
UNdiabetes mellitus for last 8 years, complains nail-bed
hemorrhages on both first toes with no mechanical injury with
raised fragility of nails along with the pain in lower
extremities. What can be the most probable reason of these
symptoms?
A. *Diabetic microangiopathogenesis.
B. Atherosclerotic angiopathogenesis of vessels of lower
extremities.
C. Fungal lesions of nail.
D. The raised fragility of blood vessels due to the lack of
vitamin РР.
E. Pain connected with impaired innervations due to
osteochondrous lumbosacral radiculitis.
192. In a woman of 23 gestational diabetes mellitus is
developed during pregnancy. An endocrinologist prescribed
diabetic diet and insulin therapy to her. Diabetes was in the
stage of compensation during pregnancy. What may be tactics
137
of supervision after labour?
A. *Insulin treatment is not necessary but making the
glucose tolerant test annually.
B. To continue treating with insulin.
C. To discontinue insulin but to appoint medicine of
sulphonilureas.
D. To discontinue insulin but to appoint biguanides.
E. To administer biguanides along with insulin.
193. At the man of 25 years who is ill with a diabetes of 8
years, illness complication, a coma: a condition heavy, a skin
dry, turgor skin it is lowered, an acetone smell, breath
Kussmaul’s, blood pressure 105/60, pulse-116/minute. What
kind of a coma can be suspected?
A. Lactacidemia
B. Hyperosmolar coma
C. *Ketoacidosis
D. Hypoglycemic coma
E. Brain coma
194.The ill M., 12 years old, after the carried windy pox in a
heavy form there was the promoted thirst (drinks to 5 liters in
a day), poliuriya , dry skin, pruritus. The grandfather of patient
is ill diabetes mellitus. Level of glycemia – 15 mmol/l, acetone
of urine +. To define a diagnosis.
A.
* Diabetes mellitus is type 1, stage of decompensation,
ketosis ;
B.
Diabetes mellitus type 1, subindenifications;
C.
Diabetes mellitus 2 types, easy motion;
D.
Diabetes mellitus type 2, to middle weight, stage of
indemnification;
E.
Diabetes mellitus type 1, heavy form, ketoacidosis.
138
A.
B.
C.
D.
E.
195.A 30-year-old nursing student presents with
confusion,sweating, hunger, and fatigue. Blood sugar is noted
to be 40 mg/dL. The patient has no history of diabetes mellitus,
although her sister is an insulin-dependent diabetic. The patient
has had several similar episodes over the past year, alloccurring
just prior to reporting for work in the early morning. On this
evaluation, the patient is found to have high insulin levels and a
low C peptide level. The most likely diagnosis is
Factitious hypoglycemia
Reactive hypoglycemia
Early diabetes mellitus
Insulinoma
Diabetes insipidus
196. In hospital the woman of 25 years without consciousness
is delivered. Mother of the patient has told, that the daughter is
ill with a diabetes since the childhood. Daily receives 40 UN
insulin. Several days ago has autocratically stopped treatment.
There was a thirst, weakness, appetite has decreased. In the
morning the patient could not wake. A skin dry, turgor it it is
lowered, eyeballs soft. Breath Cussmaul’s, is an acetone smell.
A tachycardia 110 1 minute the BP of 100/45 mm hg, the
stomach soft; a liver on 3 sm below edge of a costal arch.
Sugar of blood of 17,5 mmol/l. Urgent therapy should include:
A. *Fractional treatment by insulin in a dose 10 UN
in an hour
B. Dehydration - furosemid
C. Insulin 100 UN IV
D. Reopolyglucin to 1,0 l
E. Prednisolon 60-90 ml IV
197. The woman of 27 years is found out without
consciousness by a conductor in a train compartment.
Objectively: a skin dry, tongue dry, a smell of fruit, a short
139
wind, breath deep, noisy. HR 120/ 1 minute, the BP
80/50 mm hg, the stomach is strained, at пальпации the
painful. In blood leukocytes 17,0 . 10 9, glucose of 21 mmol/l,
creatinin 0,28 mmol/l, рН 7,2. In the urine received catheter,
reaction with nitroprussid sodium +++. Appoint most effective
treatment in the given situation:
A. Insulin of average duration of action 100 UN IV
B. Insulin of fast action 100 UN IV
C. Insulin of fast action in a dose 6-10 UN/hour IV
D. 4 % a solution of bicarbonate of sodium 400,0
E. Glucagon 1,0 mg s/c or IM
UNIT 2
1. Most common presentation of endemic goitre is:
A.
Hypothyroid*
B.
Diffuse goitre
C.
Hyperthyroid
D.
Solitary nodule
E.
Subacute thyroiditis
2. A 42-yr-old woman presents thyrotoxicosis with
decreased Radioiodine uptake. Put diagnosis, please.
A.
Subacute thyroiditis*
B.
Thyrotoxicosis
C.
Hashimoto's thyroiditis
D.
Grave's disease
E.
Toxic multinodular goiter
3. A 41-yr-old patient presents with sore throat
followed by midline tender swelling with pain, red hot skin
on the anterior part of the skin. Put diagnosis, please.
A.
Subacute thyroiditis
B.
Acute thyroiditis*
140
C.
Thyroglossal cyst
D.
Toxic thyroid nodule
E.
Chronic thyroiditis
4. A 72-yr-old-female was admitted to the hospital
in unconscious status. Myxedema coma was diagnosed.
What treatment can you prescribe?
A.
Hydrocortisone
B.
IV fluids
C.
T33 injections
D.
All of the above*
E.
None above them
5. Thyroid carcinoma associated with hypocalcemia is
A.
Follicular carcinoma
B.
Medullary
carcinoma*
C.
Anaplastic
carcinoma
D.
Papillary carcinoma
E.
All of them
6. A 42-yr-old woman presents subacute thyroiditis All the
following are true of DeQuervan's Thyroiditis EXCEPT
A.
Pain
B.
Increased ESR
C.
Increased radioactive iodine uptake*
D.
Fever
E.
Increased lymphocyte level
7. Needle biopsy of solitary thyroid nodule in a 31-yr-old
woman with palpable cervical lymph nodes on the same
sides demonstrates amyloid in stroma of lesion. Likely
diagnosis
A.
Medullary carcinoma thyroid*
B.
Follicular carcinoma thyroid
C.
Thyroid
adenoma
D.
Multi
nodular goiter
141
A.
B.
C.
D.
E.
A.
B.
C.
D.
E.
A.
B.
C.
D.
E.
DeQuervan's Thyroiditis
8. A 52-yr-old woman presents to her GP for fatigue,
depression and weight gain. She also complains of
constipation and poor memory. On examination, she has a
smooth peaches and cream complexion, thin eyebrows and
a large tongue.
What treatment of thyroid disorder can you recommend?
Diuretics
Propanolol
Carbimazole
Thyroxine*
Propylthiouracil
9. 63. A 17-yr-old girl presents with lethargy and
weight gain. She is depressed and sensitive to cold. She
would like something done about her excessive weight. The
other children abuse her.
Prescribe investigations, please.
Dexamethasone suppression test
CAT scan of the skull
Water deprivation test
T3, T4 and TSH*
Serum TRH
10. A previously well 71-yr-old woman has been
noticed by her daughter to be increasingly slow and
forgetful over several months. She has gained weight and
tends to stay indoors with the heating even in warm
weather.
Put diagnosis, please.
Cerebral malignancy
Hashimoto's thyroiditis
Subacute thyroiditis
Hypothyroidism*
142
E. Graves' disease
11. Habitant of the Ivano-Frankivsk region of age of 28
years, pregnancy. In marriage 7 years, there were a few
pregnancies and all ended by fetal loss. During a review the
goiter of 1 B grade is determined, thyroid gland is soft. During
the inspection of plasma of blood, the level of thyriod
hormones are normal. Middle iodineuria at the habitants of this
region is 50 mkg/l. The treatment of iodine deficit state first of
all usually include:
A. Iodized salt
B. Consumption of meal rich in iodine
C. L-thyroxinum 75 mkg per day
D. IodBalance 200 mg per day
E. Antistruminum 1 tab. 2 times per a week
12. Pregnant women, 28 years old, habitant of the
Carpathian region. In anamnesis – 10 years ago was treatment
of endemic diffuse nontoxic goiter of 1 B grade. Objective
review: thyroid gland enlarged 2 grade, smooth, normal texture
and homogenous. Periorbital puffiness, Stellwag's, Dalrymple's,
Rosenbach's signs are negative. Level of thyroid hormones are
normal. What are principles of treatment?
A. Potassium iodide 200 mg per day
B. Potassium iodide 150 mg per day
C. Consumption of meal rich in iodine
D. Only iodized salt
E. Diet, rich in iodine and potassium iodide 200 mg per day
diet, rich in iodine
13. Patient U., 26 years old, complains of swallowing,
weakness, feeling of a “loop” round the neck. In anamnesis –
10 years ago was treatment goiter. Objective review: thyroid
gland enlarged III grade, normal texture, homogenous.
Periorbital puffiness, Stellwag's, Dalrymple's, Rosenbach's
143
A
B
C
D
E
signs are negative. Result of ultrasound examination:
a thyroid gland is increased, total size is 36 cm³, echogenicity
is not changed. Level of thyroid hormones are normal. Median
of iodine excretion with urine 100 mkg/l. Substantiate
diagnosis:
A. Nodular goiter
B. Endemic diffuse nontoxic goiter of 3 grade
C. Diffuse nontoxic goiter of 2 grade
D. Sporadic diffuse nontoxic goiter of 2 grade
E. Diffuse euthyroid goiter of 2 grade
14. A previously healthy 19 year old female complains of
a fast heart rate, weight loss, and fatigue over the past 2 months.
Her family history is significant for a grandmother and aunt with
Hashimoto thyroiditis. Objective review: temperature - 37,0° C,
pulse - 110 beats/min, blood pressure - 120/50, cardiac tones
normal, skin is moist, warm, a mild tremor. Thyroid gland
enlarged, smooth, normal texture and homogenous. High level of
T3 and T4, undetectable TSH. Thyroid stimulating
immunoglobulin assay is positive. Your diagnosis?
Diffuse toxic goiter
Hashimoto thyroiditis
Diffuse nontoxic goiter
Nodular goiter
Sporadic diffuse nontoxic goiter
15. A 24 years old woman with chronic schizophrenia is
referred for evaluation of abnormal thyroid function tests after
experiencing an acute exacerbation of psychosis. She has no prior
history of thyroid dysfunction but several family members have
been affected by autoimmune thyroid disease. Over the preceding
three months she has lost 8 kg and has noted insomnia,
tachycardia, heat intolerance, and irregular menses. Physical
examination demonstrates an anxious and restless young
144
woman with resting tachycardia (rate 120 beats/min),
tremor of extended fingers, bilateral lid lag and “stare”, warm
moist skin, brisk reflexes, and an impalpable thyroid gland.
Thyroid function tests are as follows: free T 4 - 4,2 ng / dL
(normal 0,8 – 2,0 ng / dL), TSH < 0,01 μU / L (normal 0,4 –
4,5 μU / L), thyroglobulin – 5 ng / mL (normal < 40 ng / mL).
Which diagnosis is most likely?
A Nervous exhaustion
B Adenoma of thyroid gland
C Struma ovarii
D Diffuse toxic goiter
E Nodular goiter
16. Patient A., 38 years old. Operated a diffuse toxic
goiter. After the operation the state a patient became worse,
palpitation, shortness of breath, diarrhea, fever. Objective
review: fever is 38.6°С, pulse - 160 beats/min, blood pressure 85/40. Preliminary diagnosis:
A. Thyrotoxic crisis.
B. Thyrotoxic hepatitis
C. Pneumonia
D. Adrenal crisis
E. Acute pancreatitis
17. The patient M. is 55, appealed with complaints
about the increase of thyroid. From the inspections the diffuse
increase of thyroid is exposed to 2 grade, function is abnormal:
high level of T3 and T4, undetectable TSH. The treatment of
diffuse toxic goiter first of all usually include. Choose agent,
which inhibits synthesis of thyroid hormones:
A. Thiamazole
B. Potassium perchlorate
C. Potassium iodide
D. Iopanoic acid
145
E. Dexamethasone
18. A 50 year old man presents with enlargement of left
anterior neck. He has noted increased appetite over past month
with no weight gain, and more frequent bowel movements over
the same period. Physical examination: temperature of 37,4 °С,
the heart rate is 92 and the blood pressure is 110/50. There is an
ocular stare with a slight lid lag. The thyroid gland is
enlargement of 3 grade and asymmetric to palpation, nodule in
left lobe of the thyroid gland. Result of ultrasound examination:
a thyroid gland is increased, total size is 40 cm³, there is a 3 x
2.5 cm firm nodule in left lobe of the thyroid. Level of thyroid
hormones are abnormal: high level of T3 and T4, undetectable
TSH. Which diagnosis is most likely?
A Adenoma of thyroid gland
B Nodular goiter 3 grade, thyrothoxicosis
C Multinodular goiter
D Diffuse toxic goiter
E Nodular goiter
19. Patient V., 26 years old, during 3 months was ill
diffuse toxic goiter III, to treatment – thyrozol 30 mg per day.
After grippe the patient complaints: palpitations, tremor, high
fever, diarrhea. Objective review: thyroid gland enlarged,
smooth, normal texture homogenous. Abdominal pain, vomiting.
Tachycardia – 140 beats/min, blood pressure 140/50. Fever – 40
° C. Establish your diagnosis?
A. Diffuse toxic goiter in decompensation: Thyrotoxic crisis.
B. Nodular goiter
C. Toxic goiter in pregnancy
D. Adenoma of thyroid gland
E. Diffuse toxic goiter in compensation
20. A 53 year old woman came to the polyclinic. She
had no symptoms but gave a history of a lump in her neck
146
being noticed by her primary care physician during a
routine ‘well-woman’ check. There was no family history of
thyroid disease and she had a blameless past medical history.
She had not noticed any change in her voice, or difficulty
swallowing or breathing. Examination was entirely normal,
except thyroid gland enlarged II grade, normal texture,
homogenous for a 3 × 2 cm single nodule in the left lower
thyroid gland. Blood tests showed that her total T4, free T3, TSH
- normal, and thyroid autoantibodies were not present in serum.
What is the probable diagnosis?
A. Nodular goiter
B. Endemic diffuse nontoxic goiter
C. Diffuse nontoxic goiter
D. Sporadic diffuse nontoxic goiter
E.
Diffuse euthyroid goiter
21.Which of given below symptoms is characteristic for
hypothyrosis:
a.
bradycardia
b.
body mass lessening
c.
increased basal metabolism
d.
all mentioned above
22.Name of the most dangerous complication of the
thyrotoxic crisis:
a.
acute violation of the cerebral blood circulation
b.
acute renal insufficiency
c.
acute cardiac and coronary insufficiency
d.
acute respiratory insufficiency
23.Name a thyrostatic remedy:
a.
thyroidine
b.
thyrocomb
c.
mercazolil
d.
thyroxin
147
e.
thriiodothyronine
24.What violation of the cardiac rhythm is characteristic
for a thyrotoxic heart?
a.
respiratory arrhythmia
b.
sinus bradycardia
c.
paroxysmal ventricular tachycardia
d.
ventricles’tremor
e.
glimmering arrhythmia
25.Woman of 42 came to the polyclinic for the primary
examination. Her weight was decreased. She had tremor of the
upper extremities. The woman was excited, dissatisfied with
everything and she behaved herself outrageously. Suddenly she
lost consciousness. Tachypnea and cardiac rhythm violation
were evident. Her arterial pressure was 190/110 mm, her pulse
was 160 beatings per minute, her temperature was 38.8 C. The
most veritable diagnosis is:
a.
thyrotoxic crisis
b.
hypothyroid crisis
c.
hypertensive crisis
d.
sympathoadrenal crisis
26.Patient M. of 39 years old having a thyrotoxic goitre
the second day after the subtotal resection of the thyroid gland
the temperature rose up to 41 C, the puls was 180 beatings per
minute, the glimmering arrythmia has developed, the tremor of
the extramities appeared. How could the development of the
thyrotoxic crisis have been prevented?
a.
to remove the phenomena of the thyrotoxicosis
before the operation
b.
to refuse from the operative treatment
c.
to prescribe iodine preparations
d.
to prescribe thyrostatic preparations during the postoperative period
148
27.The thyrotoxic crisis can be developed as a result of:
a.
inadequate treatment of thyrotoxicosis
b.
addition of an infectious disease if patients are suffering
from the thyrotoxicosis
c.
cancellation of taking tyrostatic preparations
d.
operative intervention
e.
all mentioned above
28. A solitary node in the left lobe of the thyroid gland
of a 25 year old man was detected. Which of the investigations
mentioned below is to be made in the first turn?
a.
T3,T4 determination
b.
TTH determination
c.
fine-needle aspiration biopsy of the node
d.
the USD of the thyroid gland
29. Afther the subtotal resection of the thyroid gland the
patient began to suffer from increasing weakness, drowsiness
and fatigability. Oedema of eyelids appeared. Hypothyrosis
was suspected. What is it necessary to determine?
a.
TTH
b.
T3, T4
c.
the level of antythyroid antibodies
d.
the level of the radioactive iodine absorption
30. He patient P, 60 years old, was brought into the
therapeutical department with pneumonia. Has suffered from
the hypothyrosis for almost 10 years. Took L-thyroxin in a
dose of mkg. The last month stopped taking preparations. It
was detected that her general state is grave. Her reflexes are
inhibited and the oedemas of the face and extremities are well
seen. Her body’s temperature is less than 34 C. she has
hypotonia and weakened respiration in the lungs. There is
crepitation on the left. Your diagnosis is:
149
a.
b.
c.
d.
hypothyroid coma
thyrotoxic crisis
sympathoadrenal crisis
meningoencephalitis
31. Patient (woman), 25 years, plans pregnancy and
appealed to endocrinologist to recommendations in relation to
the use of iodine with a meal. Specify a daily necessity in an
iodine for the biosynthesis of thyroid hormones for adult
healthy person:
A. 100-150μg
B. 14-17mg
C. 1-2g
D. 98-104ng
E. 123-228μmol
The right answer: A.
32. Woman, 27 years, appealed to endocrinologist with
endemic goiter 2 degree, euthyrosis. What is the tactic of
treatment in this patient?
A. To prescribe L-thyroxin in complete dose during 612 months, after diminishment of size of gland - maintenance
dose of L-thyroxin, if it's normalization of gland function iodated salt
B. To conduct preoperative preparation with iodides
and follow radical surgical treatment
C. To prescribe medical dose of iodine-131
D. Complex treatment by polyvitamins with
microelements
The right answer: A.
33. Woman 27 years, appealed to the doctorendocrinology with a diagnosis endemic goitre of a 2 item,
еутиреоз. What at this sick does it follow to choose tactic of
treatment during pregnancy?
150
A. L-thyroxine
B. Thyamazol
C. Metizol
D. Merkazolil
E. Lugol's solution
The right answer: A.
34. Enlargement of thyroid gland was defined in
woman, 25 years, during examination. She lives in Carpatian
mauntain region. Abnormalities in internal organs was not
fixed. Thyroid gland is diffusely enlarged to II degree, it's
softly-elastic, smooth, painless. Levels of thyroid hormones
are normal. US: thyroid gland is enlarged, echogenicity is not
changed. Note the most credible diagnosis:
A. Endemic diffuse euthyroid goiter, II degree
B. Autoimmune thyroiditis, euthyrosis
C. Nontoxic diffuse euthyroid goiter, II degree
D. Nodular goiter
E. Chronic thyroiditis
The right answer: A.
35. Patient (woman), 48 years, complains about
irritability, hyperhydrosis, tremor of hands, palpitation, weight
loss, progressing weakness, violation of sleep. She is ill during
one year. Objectively: skin is moist, warm, thyroid gland is
diffusely enlargement to ІІ degree, it has elastic consistency;
positive Cocher, Grefe, Stelvag's symptoms. Pulse - 118/min,
BP - 150/60mmHg. Cardiac tones are increased, systolic noise
is on apex. Preliminary diagnosis:
A. Diffuse goiter, II degree, thyrotoxicosis
B. Chronic autoimmune thyroiditis
C. Diffuse goiter, II degree, euthyrosis
D. Diffuse goiter, II degree, vascular dystonia
The right answer: A.
151
36. Asymmetric enlargement of thyroid gland (II
degree) is exposed in patient, 27 years. The gland is sickly
palpatory with irradiation into lower jaw. Temperature of body
- 38°С. Patient was ill with flu week ago. Blood count increased ESR. Note preliminary diagnosis:
A. Subacute thyroiditis
B. Diffuse toxic goiter
C. Toxic adenoma of thyroid gland
D. Autoimmune thyroiditis
E. Fibrous thyroiditis
The right answer: A.
37. Patient (woman), 32 years, complains about
permanent irritability, palpitation, pain in eyes, watering,
weight loss - 10kg during 4 months. Objectively: skin is warm,
moist, mild exophthalmos, Grefe, Cocher, Mebius' symptoms
are positive. A thyoid gland is diffusely enlarged, unsickly.
Pulse - 108/min, BP - 140/66mmHg. Shallow tremor of fingers
on hands. Your diagnosis?
A. Diffuse toxic goiter
B. Acute thyroiditis
C. Medular carcinoma of thyroid gland
D. Neurasthenia
E. Subacute thyroiditis
The right answer: A.
38. What complication can after usage medicines of
iodine during long period of time?
A. Iodine-induced thyrotoxicosis
B. Hypothyroidism
C. Autoimmune thyroiditis
D. Diffuse-nodular goiter
E. All it's true
152
A.
B.
C.
D.
E.
A.
B.
The right answer : A
39. What from medicines is the most effective for
treatment of sterility in patients with hypothyroidism?
A. Thyroxin
B. Vitamin E
C. Progsteron
D. Estrogens
E. Chorionic gonadotropin
The right answer: A
40. What from these is the most typical for endemic
goiter?
A. Low excetion of iodine with urine (less 50mg/day)
B. Promoted absorption (more 50%) by the thyroid
gland І131 during 24 hours
C. Normal level of total Т4 and reduced free Т4
D. Reduced level of TSH
E. Promoted level of АB to ТPO
The right answer: A
41. A 62-yr-old woman presents with stiff joints,
myopathy and constipation. Plain radiographs reveal a
right calculus and, evidence of osteitis fibrosa cystitica.
Put diagnosis, please.
Parathyroid adenoma*
Pheochromocytoma
Prolactinoma
Corticosteroma
Follicular carcinoma thyroid
42. A 49-yr-old woman complains of perioral
paraesthesia, carpopedal spasm and generalised seizures.
Put diagnosis of metabolic disturbances, please.
Hypokalaemia
Hyperkalaemia
153
C. Hypocalcaemia*
D. Hypercalcaemia
E. Hyponatraemia
43. On a routine blood examination a 42-yr-old
woman is found to have very high serum calcium level She
has complained recently of bouts of abdominal pain and
recurrent UTI. On physical examination you find an
enlarged thyroid gland.
Put diagnosis, please.
A. Aytoimmune thyroiditis
B. Hyperthyroidism
C. Parathyroid carcinoma*
D. Secondary hyperparathyroidism
E. Hypoparathyroidism
44. A 56-yr-old presented with following reports on
a routine screen: Calcium-2.86 mmol/l, phosphate—0.8,
ALP—111, PTH—raised, 25-OH vitamin D—low-normal.
Put diagnosis, please.
A. Primary hyperparathyroidism*
B. Secondary hyperparathyroidism
C. Tertiary hyperparathyroidism
D. Hypoparathyroidism
E. Hyperparathyroid with ectopiq PTH
45. A 36-yr-old woman presents with weight loss,
muscular weakness, oligomenorrhoea, diarrhoea and
blurring of vision. On examination, there is exophthalmos
and proximal myopathy.
Put diagnosis, please.
A. Subacute thyroiditis
B. Hypothyroidism
C. Hashimoto's thyroiditis
D. Graves' disease*
154
E. Follicular carcinoma
46. A 32-yr-old pregnant woman presents to her
physicion with anxiety. On examination she is nervous
woman with exophthalmos, warm peripheries and atrial
fibrillation.
What treatment of thyroid disorder can you recommend?
A. Calciferol
B. Propanolol
C. Carbimazole
D. Thyroxine
E. Propylthiouracil*
47. A 49-yr-old woman presents with fever,
tachycardia restlessness, hypertension and vomiting. On
examination she has diffuse swelling of the thyroid gland
and strabismus with diplopia.
Put diagnosis, please.
A. Subacute thyroiditis
B. Graves' disease*
C. Hashimoto's thyroiditis
D. Thyroid carcinoma
E. Hypothyroidism
48. A 24-yr-old young woman is warm, even when
resting. She turns the central heating off, opens the
windows and annoys her family. Her pulse rate is high and
her skin is moist.
Put diagnosis, please.
A. Toxic adenoma
B. Hyperthyroidism*
C. Parathyroid carcinoma
D. Hypoparathyroidism
E. Follicular carcinoma
49. A 46-yr-old woman presents with tachycardia,
155
A.
B.
C.
D.
E.
A.
B.
C.
D.
E.
atrial fibrillation, sweating, double vision and
swelling above her ankles. She has lid lag on examination.
Prescribe investigations, please.
Fasting blood glucose
Neck USG
ESG
T3, T4 and TSH levels*
Basal plasma protein
50. A female, 62 years old, suffers from pernicious
anaemia for which she has received 1 mg cyanocobalamine
intramuscularly every 3 month for the last 10 years. At a
routine visit the patient is found with a puffy swollen face due
to a non-pitting edema. Her skin is dry and cold, the heart rate
is 55 beats/ min, her hair is sparse, and she complains of
constipation and fatigue. A series of blood tests reveals the
following: high levels of microsomal autoantibodies against the
thyroid gland and autoantibodies against her parietal cells. The
TSH concentration in the plasma is high, whereas the T3, T4 are
low. The haematological variables are satisfying. What is the
probable diagnosis?
Hypothyroidism
Cardiac insufficiency
Pernicious anaemia
Autoimmune (Hashimotos) thyroiditis, hypothyroidism
Nephrotic syndrome
51. A 28 year old woman with recent tiredness and
difficulty concentrating had experienced a decline in memory
over the last several months. She also noted decreased
frequency of bowel movements and an increased tendency to
gain weight. She felt chilled without light sweater, even in
warm weather. In the anamnesis - hypothyroidism in her
mother and older sister. Objectively: She had a slightly puffy
156
A.
B.
C.
D.
E.
face and her eyebrows were sparse, especially at the
lateral margins. The thyroid gland is not palpated. Heart rate –
58 beats/min, BP is 100/60 mmHg. Tones of heart of low
sonority. The deep tendon reflexes were normally contractive,
but showed delayed relaxation. What laboratory tests would
you order to evaluate this patient?
A. Ultrasound thyroid gland
B. Test for anti-thyroid antibodies (anti-thyroglobulin and
anti-microsomal)
C. Blood tests: levels T3, T4 and TSH
D. ECG
E. All methods
52. At patient M., 29 years old, the asymmetric increase
of thyroid gland of the II stage is exposed, a gland is painful at
palpation, pain irradiates in a left ear and upper jaw.
Temperature of the body 38,7 °C. A week ago carried a
tonsillitis. Most reliable that patient has:
Fibrotic thyroiditis
Diffuse toxic goiter
Autoimmune thyroiditis
Subacute thyroiditis
Toxic adenoma of thyroid gland
53. The patient N., 26 years old complains of the
increase of body mass, weakness, constipations, worsening of
memory, some lowering of the voice and increase in fatigue.
After delivery, she nursed the infant for 1 week. She then
stopped nursing, but galactorrhea and amenorrhea continued
for the next 5 months. In the anamnesis - she had menarche at
age 16 and had regular periods. She married at age 24 and was
not able to conceive. After receiving therapy for 7 months for
treatment of extensive endometriosis, she became pregnant and
delivered after 36 weeks' gestation. Her sister had autoimmune
157
thyroiditis. Objectively: a skin by touch is dry. A
thyroid gland is enlarged II grade, smooth surface. Pulse – 58
beats/min, diminished sonority of tones of heart. The laboratory
results: high levels of microsomal autoantibodies against the
thyroid gland, the levels of TSH and prolactin were elevated,
low levels of T3, T4. What is the diagnosis?
A. Autoimmune thyroiditis, hypothyroidism
B. Diffuse nontoxic goiter
C. Autoimmune thyroiditis without violation of the thyroid gland
function
D. Hypothyroidism
E. Syndrome of Van – Vik – Ross – Geness
54. A female, 72 years old in the grave condition
hospitalization on emergency. Objectively: Temperature of the
body - 35,8 °C. Blood pressure – 80/50 mmHg, pulse - 56
beats/min, diminished sonority of tones of heart, breathing - 12
/min. A skin is pale, cold, moderate edema of face and
extremities. The hairs are liquid, thin, on a head areas of
alopecia. Most reliable that patient has:
A. Addisonic crisis
B. Myxedema coma
C. Lactacidotic coma
D. Hypoglycemic coma
E. Hypocalcemia
55. A patient B., 59 years old complains of the presence
of nodule on the front surface of neck. Became ill 3 years ago. A
nodule was enlarged in sizes, the timbre of voice changed,
feeling of pressure appeared. Objectively: in the right lobe of
thyroid a nodule is palpated 5 cm in a diameter, painless. The
functional state of thyroid is not changed. What is the
diagnosis?
A.
Nodular euthyreoid goiter
158
B.
C.
D.
E.
A.
B.
C.
D.
E.
Nodular hyperthyroid goiter
Cancer of thyroid gland
Chronic lymphomatous Hashimoto thyroiditis
Chronic fibrous Ridel's thyroiditis
56. At a patient M., 45 years old at the palpation of
thyroid gland a nodule
2,0 х 3,2 cm is exposed in a left
lobe, compacted, moderately painful during palpation. "Cold"
nodule at scyntygrafia with I131. What inspection is most
expedient for clarification of diagnosis?
Ultrasound of thyroid gland
Aspirational biopsy
Determination of the TSH, T3 and T4 level in a blood
Immunodetection
Computed tomography scan (CT-scan) of thyroid gland
57. A patient S., 29 years old, for three days after
thyroidectomy for thyroid gland cancer has complains of
paraesthesia, muscle fibrillations, convulsions in extremities.
What is the possible diagnosis?
A. Secondary hypoparathyroidism
B. Hypothyroidism
C. Distant metastases
D. Myeloma
E. Primary hypoparathyroidism
58. A patient Z., 58 years old has been operated on for
multinodular goiter III grade. For one month after the operation
cramps in upper extremities had appeared, which persisted for
1-2 min. and accompanied with numbness in face. Cramps
occur 2-3 times a day, commonly at a daytime. Pulse is 86
beast/min, rhythmic; blood pressure is 100/70 mmHg. Visceral
organs are not damaged. Trousseau's, Hvostek's I and II,
Schlesinger's, Hoffmann's symptoms are positive.
What is the diagnosis?
159
A.
B.
C.
D.
E.
A.
B.
C.
D.
E.
Postoperative hypoparathyroidism
Postoperative hypothyroidism
Postoperative hypoparathyroidism and hypothyroidism
Pseudohypoparathyroidism
Epilepsy
59. A patient Q., 60 years old, having urolithiasis for 15
years, has coral kidney stone size 22x30mm in right kidney.
Menopause has been obtained at 52 years. Laboratory tests:
serum calcium - 2,95 mmol/L, serum phosphate - 0,6 mmol/L,
creatinine, urea are normal. Bone X-ray examination: systemic
osteoporosis, subperiosteal resorption of bones, cysts, spine
deformation. Determine possible diagnosis:
Pedjet disease
Primary hyperparathyroidism
Secondary hyperparathyroidism
Postmenopausal osteoporosis
Primary hypoparathyroidism
60.Determine operative treatment of the nodal goitre.
a.
node size is up to 1 cm
b.
node size is up to 2 cm
c.
node size is up to 3 cm
d.
senile age of the patient
61.What form of the hypothyrosis is the increasing level
of the TTH characteristic for?
a.
primary
b.
secondary
c.
tertiary
d.
peripheral
62. The myxedema coma is characterized by:
a.
deep hypothermia
b.
hypothermia
c.
tachicardia
160
d.
hypertension
63. The following frequency of the cardiac contractions
is characteristic for an easy grade of gravity of the diffusive
toxic goitre:
a.
up to 100 beatings per minute
b.
up to 120 beatings per minute
c.
more than 120 beatings per minute
d.
more than 140 beatings per minute
64. How much time must the thyrotoxicosis treatment
take?
a.
1 month
b.
2-4 months
c.
4-6 months
d.
1-1,5 year
65. Which of the below mentioned preparations is used
in the complex treatment of thyrotoxicosis?
a.
β-adrenoblockers
b.
ATE’s inhibitors
c.
statins
d.
diuretic preparation
66. With what TTH level of L-tyroxin shouldn’t be
prescribed?
a.
up to 3 mu/litre
b.
up to 5 mu/litre
c.
up to 7 mu/litre
d.
up to 10 mu/litre
67. A 65 year old patient addressed the doctor
complaining of tachycardia, uneven heart beating, shortness of
breath, oedema of shins and feet, pain behind the sternum. Has
been ill nearly 2 months.
Detected: pulse 130 beatings per minute. The pulse is
allorrhythmic with weak filling. Cardiac tones are muffled.
161
Glimmering take place. Her belly is soft,
tachyarrhythmia, the liver +3 cm., the thyroid gland is
diffusively enlarged up to the second grade. It is elastic.
Tremor of the fingers when arms are stretched out. Grefe
symptom is positive. Oedema of lower extremities takes place.
Make the diagmosis.
a.
diffuse toxic goitre
b.
stenocardia of the strain
c.
hypertonic disease
d.
rheumatism
68. A 48 year old patient addressed the doctor
complaining of eye balls oedema, cutting and lacrimarrhage in
the eyes, progressive decreasion of the vision . Has been ill
nearly 1 year.
Detected: pulse 72 beatings per minute. The pulse is
rrhythmic with normal filling. AP 115/80 mm. Pathlogy of all
the rest organs and systems is absent. Oedema of eyelids,
unilateral exophtalm takes place. Eye-chinks are not closed.
Initial atrophy of the optic nerves on the ophtalmoscopy take
place. Make the diagmosis.
a. autoimmune ophthalmopathy
b. thyrotoxicosis
c. autoimmune thyroiditis
d. hypothyrosis
69. Name factors which promote the development of
the hypothyroid coma:
a.
stoping of taking thyroid preparations
b.
overcooling
c.
surgical intrusion
d.
all mentioned above
70. Excited state, paresthesias on the face, extremities,
dyspnea, retrosternal pain, periodically convulsions of shin
162
muscules appeared in woman, 46 years, after subtotal
resection of thyroid gland. HR- 89/min. Breathing rate 20/min. BP - 150/100mmHg. What medicines are indicated for
prophylaxis of sharp exacerbation of state?
A. Preparations of calcium
B. Neuroleptics
C. Tranquilizers
D. Nitrates
E. Beta-blockers
The right answer: A.
71. Inspiratory dyspnea, constringent retrosternal pain,
parasthesias on face and extremities appeared in woman, 52
years, after strumectomy on the occasion of the cancer of
thyroid gland. What symptom appearance is the most credible?
A. Chvostek's
B. Babinsky's
C. Stelvag's
D. Mebius'
E. Grefe
The right answer: A.
72. Treatment with L-thyroxin in dose 50μg a day was
prescribed in woman, 56 years, in connection with
postoperative hypothyroidism. Level of TSH is normal.
Constringent retrosternal pain with irradiation to left shoulderblade after physical activity appeared. AT -150/100mmHg.
What changes of therapy are need after the removal of pain?
A. To reduce the L-tyroxin dose
B. To add beta-blockers
C. To prescribe nitrates
D. To replace L-thyroxin on triiodthyronin
E. To prescribe thyrotom
The right answer: A.
163
73. Woman, 36 years, with primary
hypothyroidism intakes 50μg L-thyroxin a day. What
inspection is indicated for estimation of efficacy of dose of
medicine?
A. Definition of TSH level
B. Definition of T3 level
C. Definition of T4 level
D. Definition of thyroglobulin level
E. Definition of АB to ТPО
The right answer: A.
74. Elderly woman was hospitalized in the grave
condition. Objectively: BP - 90/60mmHg, pulse - 56/min.
Breathing rate - 12/min. Temperature of body - 35,8 °С. Skin is
pale, cold, very dry. Hairs are liquid, thin, on a head are places
of alopecia. Cardiac tones are dull, noises are absent. Stomach
is enlarged, the peristalsis is not auscultated. Edemata are
present on shins and round eyes. Glycemia - 3,2 mmol/l. What
the most reliable pathology does patient have?
A. Hypothyroid coma
B. Addisonic crisis
C. Hypoglycemic coma
D. Hypercalciemic crisis
E. Lactatacidotic coma
The right answer: A.
75. Patient (woman), 52 years, complains about
increase of body mass, weakness, constipations, worsening of
memory. The indicated symptoms slowly grew during the last
1,5 year. Objectively: skin is dry, there is moderate edema of
face and extremities, cardiac borders are enlarged, tones are
dull, pulse - 66/min., BP -110/70mmHg, thyroid gland doesn't
palpated.
In a blood antibodies to thyroglobulin (+) and thyroid
164
peroxydase (+++) are present; level of TSH - 15,2
mU/l. US of thyroid gland: sizes are diminished, structure is
heterogeneous. Define diagnosis:
A. Autoimmune thyroiditis, hypothyroidism
B. Autoimmune thyroiditis, euthyrosis
C. Endemic goiter
D. Subacute thyroiditis
E. Diffuse untoxic goiter
The right answer: A.
76. Patient (woman), 44 years, complains about general
weakness, periodic pain in cardiac area, somnolence. She's ill
during 3 years. She treated without an effect in a cardiologist
and neurologist. Objectively: height 166cm, weight 70kg. Skin
has ordinary color and it's moist by touch. A thyroid gland is
diffusely enlarged (II degree), resiliently-elastic consistency,
with a smooth surface, mobile. Pulse - 96/min. BP 140/70mmHg. Cardiac tones are impaired, systolic noise is
present on the apex. Breathing is vesicular. Pathology of
abdominal cavity organs isn't detected. Additional data: blood
count and urine analysis are without pathology. US of thyroid
gland: a gland is enlarged, a capsule is impact, a contour is
clear, echogenicity of tissue is reduced, alternation of
echopositive and negative areas. There is presence of
antibodies to thyroid peroxydase. Define clinical diagnosis:
A. Autoimmune thyroiditis
B. Diffuse toxic goiter (II degree)
C. Cancer of thyroid gland
D. Fibrous thyroiditis
E. Diffuse euthyroid goiter
The right answer: A.
77. What level of blood pressure is the most typical for
uncomplicated form of diffuse toxic goiter?
165
А.Increased systolic and decreased diastolic BP
B. Increased systolic and diastolic BP
C. Increased diastolic and normal systolic BP
D. Diastolic BP is increased and systolic BP is decresaed
E. Decreased systolic and diastolic BP
The right answer : A
78. Specify the main mechanism of antithyroid effect of
glucocorticoides:
А. Inhibition of formation of thyroidstimulated
antibodies
B. Change of sensitivity of thyrotrophes to thyroliberin
C. Decreasing of converting of Т4 to Т3
D. Decline of synthesis of steroid hormones
E. All it's truel
The right answer : A
79. Specify features of clinical displays of diffuse toxic
goiter inelderly?
А. Prevalence monosymptomatic forms with
visceropathy
B. Expressed ophthalmopathy
C. Expressed neuro-psychologic disorders
D. Expressed weight loss
E. Frequent diarrhea
The right answer : A
80.A 38-year-old female complains of palpitations, fatigue, and
insomnia. On physical exam, her extremities are warm and she
is tachy cardie. There is diffuse thyroid gland enlargement and
proptosis. There is a thickening of the skin in the pretibial area.
Which of the following lab values would you expect in this
patient?
A. Decreased TSH, increased total thyroxine
B. Increased TSH, total thyroxine, total T3
166
C. Increased T3 uptake, decreased T3
D. Decreased TSH, normal T4
E. Decreased TSH, decreased T4
81. At the patient of 48 years the diffuse increase in a thyroid
gland, an exophthalmus, weight loss at 4 kg for 2 months, a
hyperhidrosis, the HR - 105/mines, BP - 140/70 mm hg a feces
normal is revealed. What therapy is recommended in that case?
A. A thyroxin
B. Radioactive iodine
C. A propranolol
D. Solution Lugol’s
E. *A mercasolil
82. At the patient of 56 years with a diffuse toxic goitre
the vibrating arrhythmia from state of emergency - 110/mines
is observed, an arterial hypertensia, BP – hg. What treatment
along with mercasolil it is necessary to recommend 165/90 mm
in this case?
A. Radioactive iodine
B. *A propranolol
C. A novocainamid
D. Verapamil
E. A nifedipine
83. The man of 57 years marks palpitation, a
hyperhidrosis, dream infringement., accruing weakness, loss in
weight. It was treated concerning the IDH without effect.
Objectively: t ° = 36,8°С, the HR 128 in minute, pulse 112 in a
minute, arrhythmic, BP of 160/70 mm hg. The skin warm,
damp. A tremor of fingers of the extended hands. Tones of
heart are strengthened, systolic noise over apex. The Thyroid
gland is not palpated. What from listed research is the most
important for specification
A. Ultrasonic research of a thyroid gland
167
B. A lipid spectrum of blood
C. Test with the dosed out physical activity
D. *Researches of level of thyroid hormones in blood
E. Ultrasonic research of heart
84. The patient of 24 years who is ill with a diffuse
toxic goitre within 5 years, complains of irritability, fatigue,
tearfulness, a hyperhidrosis, palpitation. At survey: an
insignificant exophtalm, shine of eyes, a positive symptom
Grefe. The thyroid gland is diffusely increased to П-Ш st.; a
tremor of fingers of hands, tongue. Рs - 100 in a minute. In a
disease pathogenesis has leading value:
A. Hypoproduction of a thyroid-stimulating hormone
B. Hypoproduction of hormones of a thyroid gland
C. Hyperproduction of antibodies to a thyreoglobulin
D. Hyperproduction of antibodies to a microsomal
thyreoglobulin
E. *Hyperproduction of hormones of a thyroid gland
85. The patient of 27 years complains of irritability,
insomnia, a hyperhidrosis, a weight loss at the kept appetite.
About th: the skin warm, damp, weight of a body is lowered.
The thyroid gland is diffusely increased, a uniform bilaterial
exophtalm. A tachycardia, чсс 110 in minute. Level of the TSH
in whey of blood below norm. Your diagnosis?
A. An euthyroid goitre
B. *A hyperthyrosis
C. A thyroid gland cancer
D. A hypothyroidism
E. An acute thyroiditis
86. The woman of 52 years after surgery operation
concerning a thyroid gland cancer had inspiratory dyspnea,
compressing pains behind a breast, paresthesia in face,
finitenesses. What occurrence of a symptom most is probable?
168
A. Babinsky
B. *Hvostek’s
C. Shtelvag’s
D. Moebius
E. Grefе
87. At the boy of 15 years last 3 months attacks of
spasms of chewing muscles, muscles of hands, spasms muscles
flexural. Spasms painful, symmetric. Positive symptoms
Hvostec’s, Trusso. Calcium in blood of 1,8 mmol/l. The most
probable diagnosis:
A. Spasmophilia
B. Epilepsy
C. Hyperparathyroidism
D. A tetanus
Е * Hypoparathyroidis
UNIT 3
A.
B.
C.
D.
E.
1. In persons with congenital adrenal hyperplasia
resulting from inherited defects of adrenal steroid C-21
hydroxylase, excessive androgen production is the result of
autonomous adrenal production of steroids
autonomous pituitary production of ACTH
*failure of production of an adrenal product necessary for
negative feedback on pituitary ACTH secretion
extraglandular formation from large amounts of nonandrogenic
adrenal steroids
positive feedback on pituitary ACTH secretion by abnormal
adrenal products
2. A 41-years-old woman with obesity, dermal striae,
and hypertension is referred for endocrinologic evaluation
of possible cortisol excess. The woman receives a midnight
169
A.
B.
C.
D.
E.
A.
B.
C.
D.
E.
A.
B.
C.
dose of 1 mg of dexamethasone; a plasma cortisol
level drawn at 8 a.m. the next day is 386 nmol/L (14µg/dL).
At this point in the evaluation the most appropriate
diagnostic maneuver would be
CT scanning of the pituitary gland
abdominal scanning
measurement of 24-h 17-hydroxycorticosteroid excretion in
urine
*measurement of a 24-h urine free cortisol
a 2-day high-dose dexamethasone suppression test
3. 14. A 37 – year old man has symptoms of
intermittent palpitation, anxiety, excess perspiration.
examination reveals a blood pressure reading of 210/110
mm.hg and glucosuria. basal metabolic rate is + 50 %,
fasting glucose 6,05 mmol/l, and an i/v histamine test
produced a rise in blood pressure reading to 270/160 mm.
hg, whereas a cld pressor test was negative.
The best diagnosis is:
Atherosclerotic heart disease
Diabetic nephropathy
*Pheochromocytoma
Hypertensive disease
Adrenal cortical carcinoma
4. A 48-year-old woman is diagnosed with Addison's
disease (adrenal insufficiency), and treatment with Cortisol
and fludrocortisone acetate (Florinef) is started. One month
later she still complains of weakness and fatigue.
Which of the following would suggest that the dose of
Florinef should be increased?
Ankle edema
Increased blood pressure
*Hyperkalemia
170
A.
B.
C.
D. Hypernatremia
E. Increasing pigmentation
5. A young (36-yr-old) man is found to have
hypertension. Pheochromocytoma is suspected because of
paroxysmal episodes of symptoms, with blood pressure
elevations up to 210/120 mm Hg during the paroxysms.
Which of the following findings would further support a
diagnosis of pheochromocytoma?
A. Central obesity, with a dorsal fat pad
B. Hypoglycemia
C. Low serum potassium level
D. *Postural hypotension
E. Marked fall in blood pressure with spironolactone treatment
6. A 42-yr-old woman complains of fatigue, weight
loss and intermittent abdominal pain. She looks well
tanned, although it is winter. She has postural hypotension
and buccal pigmentation.
What is the cause of fatigue?
A. Anaemia
B. Hypothyroidism
C. Addison's disease*
D. Hemochromatosis
E. Tuberculosis
7. A 57-year-old man complains of fatigue, weight
loss, and joint pains. Laboratory studies reveal that his
serum Cortisol level is undetectable.
Which of the following findings would suggest most
strongly that he has primary adrenal insufficiency rather than
secondary adrenal insufficiency due to hypopituitarism?
Serum sodium and potassium levels are normal.
Serum free thyroxine levels are low.
Blood pressure is low, and decreases further when the patient
171
D.
E.
stands up.
The patient has noted severe weakness and dizziness during
upper respiratory infections.
*The patient has noted some darkening of the exposed areas of
his skin.
8. A 26- year old female patient presents with
hypertension, Na – 146 meq/I, K – 1,4 meq/l, normal serum
creatinine most probable diagnose is:
A.
pheochromocytoma
B.
*Conna’s syndrome
C.
Cushing’s syndrome
D.
renal parenchymal disease
E.
renal vascular disease
9. A 68-yr-old woman presents to her GP with
weight loss and depression. On examination she is noted to
have buccal pigmentation and pigmented scars. She
appears dehydrated. Her BP is 100/60 mm Hg.
What treatment can you recommend?
A. Desmopressin
B. Long-term
replacement
glucocorticoids
and
mineralocorticoids*
C. Carbimazole
D. Propylthiouracil
E. Thyroxine
10. A 27 year old lady has put on weight (16 kg over a period
of 3 years), and has oligomenorrhoea followed by
amenorrohoea for 8 months. The blood pressure is
170/100 mm of Hg. Which of the following is the most
appropriate investigation?
A. Serum electrolytes
B. Plasma cortisol*
C. Plasma testosterone and ultrasound evaluation of pelvis
172
D. T3, T4 and TSH
E. Plasma prolactin
11.A 55 year old woman presented with classical
clinical features of Cushing's syndrome including hypertension,
diabetes mellitus, central obesity and easy bruising. She denied
previous depression or heavy alcohol intake and was not
receiving any steroid containing medication.
Questions:
1. List the causes of Cushing's syndrome and discuss what
initial investigations you would perform?
2. In the light of these results, what additional tests would you
perform?
3. What is the likely cause of this patient's recurrent
Cushing's and what further investigations should be
performed?
12. A 64 year old woman was seen in the Endocrine
clinic because she had noticed increasing hirsutism for 6 years
but worse over the last year. She had also noted increasing
fatigability, and some left-sided abdominal pain. Her voice had
become deeper but she had a normal appetite and no weight
loss. She had a past history of mild hypertension and had had
one child. She smoked 10 cigarettes a day. Examination
revealed marked hirsutism with temporal recession of the
hairline and a beard. The blood pressure was 200/110 mmHg.
Questions:
1. What initial investigations would you perform?
2. Following the receipt of these results, what further
investigations would you perform?
13. Patient M., 34 years old, presents with complaints of
instant weight gain, back pain, headache, decreased
potency, periodical dryness of the mouse, sense of
fatigue.
173
Objective status: height - 169 cm, weight - 102
kg. Fat tissue localized mainly on the chest, belly and hips. On
the shoulders, belly and hips are discovered striae with a dark
blue color. Cordial tones were muffled, blood pressure 170/95 mmHg. Fasting blood glucose – 6,8 mmol/L.
Questions:
1. Substantiate diagnosis.
2. Principle of treatment approach.
14. Patient B., 46 years old, complains of onsets of
high blood pressure, which appear spontaneously,
accompanied by headache, disturbance of eyesight, sense of
anxiety and fear, trembling of extremities, overwhelming
sweating, irritability, shortness of breath, nausea, vomiting,
pain over the belly and chest, pallor or ruddy face. All of which
rapidly end with profuse urination and sweating. The history of
this illness is that all symptoms presented within 6 months.
Questions:
1. What is the most likely diagnosis?
2. Principle of treatment approach.
15. A 35 year old man presented with elevated blood
pressure (188/112 mmHg, seated) at a yearly physical exam.
Previous exams noted blood pressures of 160/94 mmHg and
158/92 mmHg. On questioning, he admitted episodes about
twice a month of apprehension, severe headache, perspiration,
rapid heartbeat, and facial pallor. These episodes had an abrupt
onset and lasted 10-15 minutes. Objective status: 30 min after
the initial blood pressure measurement, the seated blood
pressure was 178/110 mmHg with a heart rate of 90 beats/min.
The blood pressure after 3 min of standing was 152/94 mmHg
with a heart rate of 112 beats/min. The optic field of vision
showed moderately narrowed arterioles with no hemorrhages
or exudates. Result of laboratory tests: routine hematology and
174
chemistry studies were within the reference ranges and
a chest film and EKG were essentially normal.
Questions:
1. What is the most likely diagnosis?
2. Principle of treatment approach.
16. Patient U., 64 year old presented to the emergency
department after experiencing generalized weakness, nausea,
constipation, and an intermittent cough for about 3 weeks. He
subsequently admitted anorexia and weight loss (approximately
10 kg) over the past month. The patient denied fever, chills,
chest pain, abdominal pain, diarrhea, and dysuria. He had
begun treatment with antibiotics 1 week prior to presentation
for a sinus infection. Objective status: temperature 36.2° C;
pulse -106 beats/min, irregularly irregular; respiratory rate - 20
/ min; blood pressure -138/84 mmHg. The patient was thin
with no palpable lymphadenopathy, and no edema or cyanosis.
Laboratory studies: Chemistry - sodium: 108 mmol/ L (normal
133 – 145 mmol/L), potassium: 5,1 mmol/L (normal 3,5 – 5,0
mmol/L), chloride: 76 mmol/L (normal 96 – 108 mmol/ L).
Fasting plasma glucose – 2,4 mmol/L. Cortisol: Baseline - 7.6
μg/dL, 30-min - 7.9 μg/dL (normal 6.0-28.0 μg/dL), 60-min
μg/dL - 8.6 μg/dL (normal >20 μg/dL).
Questions:
1. What is the most likely diagnosis?
2. Principle of treatment approach.
17. Patient D., 38 years old, presents with complaints of
general weakness, weight loss, dizziness, nausea, vomiting,
diarrhea, pain in the cord. Objective status: height - 163 cm,
weight -53 kg. Skin grayish – brown. Tones were rhythmical,
muffled. Blood pressure – 90/60 mmHg, pulse – 90 beats/min.
Questions:
1. Describe the methods used to diagnose chronic
175
adrenal insufficiency?
2. Principle of treatment approach.
18. Patient R.H., 35 year old husband and father of
three children. Over the past six months he has experienced
headaches and palpitations of increasing frequency and
severity. In addition, he has had periods of intense anxiety and
panic attacks. His wife has noticed that R.H.’s face is often
pale and that he sweats more. Upon examination by his
physician, R.H. was found to be severely hypertensive and in
atrial fibrillation. R.H.’s physician suspected that he might
have a pheochromocytoma and ordered a battery of tests,
which confirmed his diagnosis.
Questions:
1. Name the hormones secreted by the adrenal
medulla.
2. How are hormones of adrenal glands classified?
3. Why are patients with a pheochromocytoma often
hyperglycemic?
4. Describe the methods used to diagnose a
pheochromocytoma.
5. How are pheochromocytomas usually treated?
19. A 27 year old woman presents with depression,
insomnia, increased facial fullness and recent increase in facial
hair. She had also had an episode of depression and acute
psychosis following uncomplicated delivery of normal baby
boy 9 months previously. Her menses have been irregular since
their resumption after the birth (she is not breast-feeding).
Objective status: the heart rate was 90 beats/min and the blood
pressure was 146/110 mmHg. Her face was puffy with an
increase in facial hair and ruddy complexion. There was no
obesity, peripheral wasting, or striae. Laboratory studies: serum
electrolytes, white cell count, and hemoglobin and hematocrit
176
1.
2.
1.
2.
were all within normal limits.
Questions:
What do you think the patient's primary problem is?
What laboratory tests would you order to evaluate this
patient?
20. A 40 year old woman presents with a 6 month
history of increasing fatigue. For the past three months she had
suffered recurrent upper respiratory infections, poor appetite,
abdominal cramps and diarrhea. During this time, she lost 11
kg. She had also noted joint pains, muscle weakness, a dizzy
spells following exercise, and she had not menstruated for the
past 3 months. Objective status: height - 165 cm and weight –
46 kg. Her heart rate was 86 beats/min and the blood pressure
was 120/65 mmHg when she was supine. After one minute of
quiet standing, the heart rate was 120 beats/min, the blood
pressure was 90/58 mmHg, and she became dizzy.
Questions:
What do you think the patient's primary problem is?
What laboratory tests would you order to evaluate this
patient?
21.On the CT of the 38 years old patient a round
formation with the diameter to 25 mm was found in the right
adrenal. The patient has the stable arterial hypertension
180/110, shin oedematose. Trophic skin disorders and
stretchings are absent. Potassium level in the blood is 2.8
mml/l. Formulate a preliminary diagnosis.
a. pheochromocytoma
b. Itsenko-Kushing’s syndrome
c. Konn’s syndrome
d. Itsenko-Kushing’s disease
e. innate hyperplasia of adrenic cortex
22.Patient, 28 years old, moonlike plethoric face,
177
dysplastic adiposity with predominant fat fibre on the
body, stretchings on the front abdominal wall. On the USD and
CT there is a solitary formation in the left adrenal with the
diameter to 40 mm. Formulate a preliminary diagnosis.
a. pheochromocytoma
b. Itsenko-Kushing’s syndrome
c. Konn’s syndrome
d. Itsenko-Kushing’s disease
e. innate hyperplasia of adrenic cortex
23.Stretchings appearance under the syndrome of
Itsenko-Kushing’s is conditioned by:
a. hemorrhages
b. scratching
c. trophic skin modifications
d. venous excrescences
e. melatonin accumulation
24.Medicinal therapy of primary hyperaldosteronism
consists in prescribing:
a. veroshperone 200-400 mg/day
b. hydrochlorothiazide 100-200 mg/day
c. indapamide 2,5-5 mg/day
d. loratadine 25-50 mg/day
e. furosemide 100-200 mg/day
25.At the lesion of adrenals by a tubercular process with
development of insufficiency of their function this
insufficiency is defined as:
a. primary
b. secondary
c. tertiary
d. peripheral
e. any
26. For the primary hyperaldosteronism are
178
charecteristic:
a. high levels of potassium and aldosterone in the
blood
b. low levels of potassium and aldosterone in the
blood
c. high level of potassium and low aldosterone in the
blood
d. low level of potassium and high aldosterone in
the blood
e. levels of potassium and aldosterone do not change
27.To diagnose Konn’s syndrome it is possible to
make:
a. small dexamethasone test
b. test with furosemide
c. large dexamethasone test
d. test of stimulation ACTH
e. test of prednisolone oppression
28.To the methods of visualization of adrenals do not
belong:
a. scintigraphy
b. punctural biopsy
c. USD
d. CT
e. MRT
29.Chronic adrenal insufficiency is accompanied:
a. by the decrease of hyperglycemia and natremia
b. by the increase of hyperglycemia and natremia
c. by the decrease of hyperglycemia and increase of
natremia
d. by the increase of hyperglycemia and decline of
natremia
e. levels of sodium and potassium do not change
179
30.Arterial hypertension develops with all
adrenal illnesses, except:
a. pheochromocytoma
b. Kushing’s syndrome
c. Konn’s syndrome
d. adrenal insufficiency
e. Kushing’s disease
31.For a patient with the kidney form of unsaccharine
diabetes it is expedient to apply following diuretic in therapy:
a. furosemide
b. spironolactone
c. hydrochlorothiazide
d. torasemide
e. mannite
32. Woman, 39 years, complains about headache,
weakness and parasthesias in extremities, polyuria.
Objectively: cardiac tones are dull, HR - 94/min, BP 190/105mmHg. Glycemia - 5,5 mmol/l, sodium of plasma 148 mmol/l, potassium of plasma - 2,7 mmol/l. In urine:
relative density - 1012, protein -0,1 g/l, pH is alkaline, WBC3-4 in area vision, RBC - 2-3 in area visiov. Specify the most
credible diagnosis:
A. Primary hyperaldosteronism
B. Amyloidosis
C. Diabetes insipidus
D. Chronic glomerulonephritis
E. Hypertension
The right answer: A.
33. Patient, 36 years, is ill during 1-1,5 years. He
complains about weakness, anorexia, nausea. Weight loss is 10
kg during last year. Skin is hyperpygmented. Folds of skin
mammillas are dark. Pulse - 60/min. BP -80/50mmHg. There is
180
high level of ACTH in blood plasma. Your previous
diagnosis?
A. Chronic adrenal insufficiency
B. Diabetes mellitus
C. Hypopituitarism
D. Chronic gastritis
E. Chronic hepatitis
The right answer: A.
34. Pain in a lumbar area with irradiation in left leg
appeared in man, 28 years. Objectively: height - 186cm, weight
- 92 kg. BP - 170/100mmHg, pulse - 84/min., Breathing rate 18/min. Distributing of fatty tissue on upper type, for except
extremities. Purple strias present on stomach and thighs. X-ray:
ostoporosis of vertebral bodies, compression fracture LIV. In
blood: RBC - 5,5х1012/l, НЬ -190 g/l, WBC - 9х109/l. Calcium
-3,3 mmol/l. What is the most credible reason of fracture?
A. Cushing's syndrome
B. Addison's disease
C. Compensatory policytemia
D. Primary hyperparathyroidism
E. Alimentary-constitutional obesity
The right answer: A.
35. Patient (woman), 46 years, much was in the
sunshine in rest period at seaside, without regard to prohibition
of doctors. In this time she weight loss, but didn't draw
attention that "a tan sticks very quickly" and also pigmentation
of mamillas and smoke-coloured spots on the mucus shell of
lips appeared. Define diagnosis:
A. Аddison's disease
B. Medicinal disease
C. Cancer of mammary glands
D. Mastopathy
181
E. Allergic dermatitis
The right answer: A.
36. Patient (woman), 36 years, complains about
frequent episodes of increase of BP to 260/120mmHg during
the last months. There are not patients with hypetension in
family . Intensive headache, palpitation, concern appear in the
morning. BP 260/120mmHg, HR - 115/min. During attack:
glycemia - 7.3 mmol/l, leukocytosis, expressed polyuria
appeared after decreasing of blood pressure. Prominent the
most effective antihypertensive drugs for this patient:
A. alfa-blockers
B. beta-blockers
C. Calcium antagonists
D. ACE inhibitors
E. Diuretics
The right answer: A.
37. Patient (woman), 42 years, after the physical
overload lost consciousness. BP 40/20mmHg. In anamnesis she
is ill with bronchial asthma during 5 years, constantly she uses
glucocorticoides. Last week she didn't use medications.
Objectively: skin has ordinary color and normal humidity,
cardiac tones are dull, pulse 100/min, weak filling, rhythmic.
Glycemia - 3,0 mmol/l, sodium
- 117 mmol/l, potassium - 6,0 mmol/l. Set a previous
diagnosis:
A. Acute adrenal insufficiency
B. Pheochromocytoma
C. Cardiogenic shock
D. Asthmatic status
E. Hypoglycemic coma
The right answer: A.
38. Woman, 35 years, felt dizziness, weakness after the
182
emotional overstrain. There are anorexia, weight loss,
nausea, diarrhea in anamnesis. Blood pressure 60/40mmHg in
vertical position. Pulse 102/min, small, rhythmic. Glycemia 3,1
mmol/l.
Hyponatriemia.
Hypercalciemia.
Hyperpigmentation of skin. Previous diagnosis:
A. Bleeding in the abdominal region
B. Vascular dystonia
C. Pheochromocytoma
D. Chronic adrenal insufficiency
E. Diabetes mellitus, hypoglycemia
The right answer: A.
39. A patient, 36 years old, has been suffering from
arterial hypertension for 4 years. Physician prescribed different
hypotensive medicines without result. Conn’s syndrome was
diagnosed after supplementary examination. What are basic
clinical and laboratory symptoms of this syndrome?
А. *Resistant arterial hypertension, hypokaliemia,
nycturia,
В. Periodical increase of blood pressure after stressful
situation, normokaliemia,
С. panic attacks, frequent urination after they, normal
level K+ in blood,
D. Palpitations, nervousness, headache, good effect from
sedatives,
Е. Arterial hypertension, good effect after treating by
hypotensive medicines.
40. A 43 years old woman, complains us about elevated
blood pressure, including paroxysmal high blood pressure,
palpitations, anxiety often resembling that of a panic attack,
headaches often, pallor sometime, diaphoresis during and after
paroxysms. Pheochromocytoma was suspected. What are basic
symptoms of this disease?
183
А.*Catecholamines (adrenalin, noradrenalin),
vanililmandelic acid in urine,
B. Increased levels of potassium, sodium, chlorides in
blood,
C. General cholesterol, triglycerides, beta lipoproteins
found in blood,
D. Increased levels of calcium, phosphates in blood,
calcium in urine,
Е. Increased levels of 17-OKS in urine and ACTH in
plasma.
41. A 42-year-old alcoholic male is being treated for
tuberculosis, but he has not been compliant with his
medications. He complains of increasing weakness and fatigue.
He appears to have lost weight, and his blood pressure is 80/50
mmHg. There is increased pigmentation over the elbows.
Cardiac exam is normal. The next step in evaluation should be:
A.
Early morning serum Cortisol and cosyntropin
stimulation
B.
CBC with iron and iron-binding capacity
C.
Erythrocyte sedimentation rate
D.
Blood cultures
E.
Level of serum K+
42. A 25-year-old woman is admitted for hypertensive
crisis. In the hospital, blood pressure is labile and responds
poorly to antihypertensive therapy. The patient complains of
palpitations and apprehension. Her past medical history shows
that she developed hypotension during an operation for
appendicitis. Hct: 49% (N=37-48). WBC: 11 x 103 mm
(N=4.3-10.8). Plasma glucose: 160 mg/dL (N=75-115) Plasma
calcium: 11 mg/dL (N=9-10.5). The most likely diagnosis is:
A.
Pheochromocytoma
B.
Renal artery stenosis
184
A.
B.
C.
D.
E.
C.
Essential hypertension
D.
Insulin-dependent diabetes mellitus
E.
Cushing syndrome
43. A 36-year-old woman has a 3-year history of
oligomenorrhea that has progressed to amenorrhea during the
past year. She has observed loss of breast fullness, reduced hip
measurements, acne, increased body hair, and deepening of her
voice. Physical examination reveals frontal balding, clitoral
hypertrophy, and a male escutcheon. Urinary free Cortisol and
dehydroepiandrosterone sulfate (DHEAS) are normal. Her
plasma testos terone level is 6 ng/mL (normal is 0.2 to 0.8).
The most likely diagnosis of this patients disorder is
A.
Arrhenoblastoma
B.
Cushing syndrome
C.
Polycystic ovary syndrome
D.
Granulosa-theca cell tumor
E.
Thyroid carcinoma
44. At patient Р., who treat from septic condition,
whom expressed weakness, adynamia, has suddenly appeared,
vomiting, a diarrhoeia. Sopor. Deficient pulse, 110 /minutes,
ВР 60/40 mm hg. On an electrocardiogram: a tachycardia,
decrease in a voltage of all waves. Laboratory data:
hyponatremia,
hypochloremia,
hyperpotassemia,
hypoglycemia. Specify the reason of development of such
condition:
Hypothalamic crisis;
Acute insufficiency of adrenal glands;
Hypoglycemic coma;
Panhypopituitarism
Acute heart attack of myocardium
45. Patient D, 42 years, after a physical overstrain has
lost consciousness. BP has decreased to 40/20 mm hg In the
185
anamnesis long (5 years) reception of glucocorticoids,
in connection with presence of a bronchial asthma. Last 4 days
glucocorticoids does not accept. About th: it is braked, a skin
of usual colour, normal humidity, tones of heart deaf persons,
pulse 100 in minute, weak filling, rhythmical. Glucose level in
blood - 3,0 mmol/l, sodium - 117 mmol/l, potassium - 6,0
mmol/l. Your preliminary diagnosis.
A. Cardiogenic shock;
B. Adrenal crisis;
C. Acute adrenal insufficiency;
D. Hypovolemic shock;
E. Hypoglycemic coma
46. At sick К, 29 years, with the compensated diabetes
of 1 type have appeared frequent hypoglycemia, a nausea,
intestines frustration, skin hyperpigmentation (bronze colour),
ВР - 70/50 mm hg., Hb 100 g/l. Than decrease ВР can be
caused?
A. Chronic insufficiency of adrenal glands
B. Diabetic enteropathy
C. Diabetic gastropathy
D. Overdose of antidiabetic preparations
E. Development diabetes insipidus
47. Patient U, 18 years, is taken to hospital by an acute
care unconsciousness. According to native it is known, that last
two weeks it was observed.
Rise in temperature of a body to 39º. With, that was
regarded, as flu signs. Four hours there was a sharp pain in a
stomach, decrease ВР. At survey: epileptic convulsion,
meningeal syndrome. From additional researches: level
increase potassium in blood to 8 mmol/l, level cortisol- 18 mkg
/ 100 ml of plasma. On an electrocardiogram – high waves T.
Computer tomography - sign hemorrhage infarction of adrenal
186
glands. What diagnosis is most probable?
A. Acute insufficiency cortex of adrenal glands
B. Hyperosmolar coma
C. Hyperlactacidemic coma
D. Hyperthyroid coma
E. Hypoglycemic coma
48. The patient 43 rivers it is delivered in a grave
condition. According to the husband, is ill with Addison's
disease. Constantly accepted 5 mg prednisolone. Last week a
preparation did not accept, as there was a pain a stomach,
appetite has worsened, yesterday did not eat in connection with
a nausea and vomiting. The patient in soporose state. A skin
and mucous hyperpigmentation. Turgor skin and muscles the
lowered. Tones of heart are muffled, accelerated, ВР-60/40
mm hg, НR-96/minute Sodium in blood - 130 mmol/l,
potassium - 5.5 mmol/l. What deficiency of a hormone plays
the basic role in complication development?
A. Aldosterone;
B. Adrenocorticotropic hormone, ACTH
C. Adrenaline;
D. Noradrenaline;
E. Cortisol.
49. At sick with suspicion on pheochromocytoma
during the period between attacks the BP - within norm; it is
marked tendency to a tachycardia. In urine of a pathology is
not present. It decided to spend provocative test with
histamine. What preparation should be had for rendering of the
emergency help at positive result of test?
A. Mesaton
B. Pipolphen
C. Nifedipine
D. Phentolamine
187
E. Prednisolone
50. At 38 summer women paroxysmal raises the BP of
240/120 mm Hg, thus the nausea, vomiting, a tachycardia,
raised perspiration, comes to light hyperglycemia. After an
attack the plentiful discharge of urine. При sonography kidneys
is revealed an additional education adjoining to the top pole of
the right kidney, probably, concerning an adrenal gland. What
laboratory test will help to specify the diagnosis?
A. Level definition rennin in blood
B. Definition of insulin and C-peptid in blood
C. Definition of speed glomerular filtration on
endogenous creatinin
D. Definition thyroxin and ACTH in blood
E. Definition excretion catecholamine and vanillic
mandelic acids with urine
51. At the patient 35th years muscular weakness,
spasms of the feet, moderated polyuria. Medicines did not
accept. Weight of a body of 75 kg at growth of 170 sm, the BP
of 160/100 mm hg. Blood glucose on an empty stomach 3.5
mmol/l, potassium blood 2.4 - 2.5 mekv/l; sodium of 140
mekv/l; creatinine 0.08 mmol/l. Fluctuations of relative density
of urine 1005-1015; glycosuria and proteinuria is not present, a
deposit in norm. What probable cause of illness
A. Diabetes insipidus
B. Hyperaldosteronism
C. Insulinoma
D. Insufficiency of adrenal glands
E. Hyperparathyroidism
52. At sick 45 years the central type of adiposity of 2
degrees, purple-red striae on a stomach, hips, increase the BP
to 200/120 mm hg, glycemia on an empty stomach - 9,3
mmol/l, infringement menstrual cycle is marked. After big
188
dexamethasone test excretion 17- oxycorticosteroids,
17-CS has not changed. The most probable diagnosis:
A. Corticosteroma.
B. Cushing’s disease.
C. A metabolic syndrome.
D. Syndrome Shtejn-Levental’s.
E. Carcinoid syndrome.
UNIT 4
A.
B.
C.
D.
E.
A.
B.
C.
D.
E.
1. A 49-yr-old man is admitted for investigation and
glycosuria. His wife comments that his appearance has
changed over the last few years and "everything seems to
have got bigger". He also cojnplains of tingling in his left
hand and excessive sweating.
Prescribe investigations, please.
Dexamethasone suppression test
Serum cortisol
T3, T4 and TSH levels
Look at old radiographs*
Water deprivation test
2. A 31-year-old male complains of polydipsia she
drinks nearly 12 L of water during the day, polyuria,
weakness. Several weeks ago he has cerebral trauma.
Postprandial glucemia level is 6,3 mmol/l. In analysis
glucose is absent, there is low urine osmolality. What is your
diagnosis?
psychogenic polydipsia
hypopituitarism
diabetes insipidus*
syndrome of inappropriate ADH secretion
diabetes mellitus
189
3. A 31-year-old man is married during 8
years but hasn’t children. Wife is healthy. He is 188 cm
tall, his weight is 76 kg. Ginecomastia. Secondary sexual
signs are developed purely. Penis is normal size, testes are
small, firm. Azospermia is present. Which of the following
karyotypes is most likely to be present?
A. 45 XO
B. 46 XY
C. 46 XX
D. 47 XXY*
E. 47 X0
4. A young (29-yr-old) women comes with
secondary amenorrhea and galactorrhoea. MRI shows a
tumor of < 10mm diameter. What kind of treatment can you
recommend?
A.
Hormonal therapy for withdrawal bleeding
B.
Radiotherapy
C.
Chemotherapy
D.
Bromocriptine*
E.
Surgery
5. A 31 year old women presents a history of amenorrhea and
impaired vision of five month's duration. Physical
examination is normal except for pale optic discs and
diminished visual acuity. The most likely diagnosis is:
A.
Pituitary adenoma*
B.
Craniopharyngioma
C.
Hypothalamic glioma
D.
Benign intracranial hypertension
E.
Encephalitis
6. All are components of Laurence-Moon-Biedl
syndrome EXCEPT:
A. Mental retardation
190
B.
C.
D.
E.
Digital anomalies
Hypogonadism
Asthenic body built*
Obesity
7. A 21 year old woman is seen for primary
amenorrhea. She is short, has poorly developed breasts
and a webbed neck. Her chromosome pattern is most likely
to be:
A.
XY
B.
XO*
C.
XXY
D.
XYY
E.
XXYY
8. A 62 - year-old female with polydypsia. Water
deprivation test was performed and the values obtained
are as follows: (Baseline : Na+-138 mEq, K+-3.6m Eq,
Osm. plasma-274/L Osm, urine-151 mosm. After
vasopressin: Na+-143) mEq, K+-3.9 mEq, Plasma osm 271
mosm/L, Urine Osm-489 mosm/L). Diagnosis is:
A.
Cranial diabetes insipidus*
B.
Psychogenic polydypsia
C.
Nephrogenic diabetes insipidus
D.
SIADH
E. Diabetes mellitus
9. A 22 year old woman presents with complaints of primary
amenorrhoea. Her height is 152 cms, weight is 50 kg. She
has well developed breasts. She has no pubic or axillary
hair and no hirsuitism. Which of the following is the most
probable diagnosis?
A.
Turner's syndrome
B.
Stein-Leventhal syndrome
C.
Premature ovarian failure
191
D.
Complete
androgen
insensitivity
syndrome*
E.
None above them
10. A 37 year old man presents with vomitings and confusion.
On examination Na+ 119 m mol/L, K+ 4.2 m mol/L, Uric
acid 2 mg/dl. Patient is no edematous. The diagnosis is:
A.
Cerebral toxoplasmosis with SIADH*
B.
Hepatic failure
C.
Severe dehydration
D.
Congestive heart failure
E.
None above them
11. A 32 years old woman complains of evaluation of
chronic thirst, polydipsia, and polyuria. The symptoms have
been present for many years and seem to increase during her
pregnancies. Several years before, she consulted another
physician who “tested my sugar”, performed a dehydration test,
and told her not to worry because she simply “drank to much”.
She is otherwise healthy and takes no drugs. She was adopted
at an early age and knows nothing of her biological family.
Objective status and result of laboratory are within normal
limits. However, her 24 – hour urine volume and osmolality are
6,5 liters and 1002 respectively. A fluid deprivation test
increases her urine osmolality and plasma sodium to maxima
of 465 mmol/kg and 141 meg/L, respectively, after four hours.
It also increases her thirst. At this point, the best approach
would be to:
A. Assess the response of urine osmolality to the injection of
desmopressin
B. Order MRI of the brain before and after gadolinium infusion
C. Measure plasma vasopressin before and after an infusion of
hypertonic saline
192
D. Start treatment with desmopressin and reevaluate her
symptoms in two weeks
E. Reassure her that she simply drinks excessively and should
restrict her fluid intake
12. An 18 year old young man is tall and has narrow
shoulders, a large pelvis, an adult woman pattern of hair distribution, and oxyphonia. Mental retardation is also present.
Based on these symptoms, the provisional diagnosis of
Klinefelter's syndrome was made by a doctor. What genetic
method can confirm the diagnosis?
A. Cytogenetic
B. Pedigree analysis
C. Population-statistical
D. Biochemical
E. Study of twins
13. During the analysis of the buccal mucosa
epithelium of a male patient two Barr bodies in each nucleus of
most cells were found and in neutrophil nuclei two
"drumsticks" in each were found. What syndrom is it typical
of?
A. Patau syndrome
B. Turner's syndrome
C. Klinefelter's syndrome
D. Down's syndrome
E. Edward's syndrome
14. An 18 year old girl complained to a doctor of the
absence of menstruation. The patient had such features: height
-140 cm, a short neck with typical folds ("neck of sphinx"),
wide shoulders, a narrow pelvis, absence of secondary sexual
characters, underdeveloped ovaries. What was the provisional
diagnosis of the girl?
A. Turner's syndrome
193
B.
C.
D.
E.
A.
B.
C.
D.
E.
Patau syndrome
Morris's syndrome
Klinefelter's syndrome
Down's syndrome
15. A 24 years old man is refferred because incidentally
discovered pituitary adenoma. He relates that he was involved
in an automobile accident three weeks earlier. Magnetic
resonance imaging performed to evaluate the consequences of
head trauma revealed a 7 mm area in the right side of the
pituitary that did not emit as bright a signal following
gadolinium as did the rest of the pituitary. On questioning, he
said he had been felling generally well. Specifically, he had
experienced headaches for a week after the accident but not
before or since. He had not noted a change in libido, potency,
or appearance. Objective status: height -160 cm; weight -75,5
kg, blood pressure was 118/82 mmHg; pulse - 72 beats/min
and regular. He didn’t appear acromegalic or to have Cushing’s
syndrome. He had no lid lag or lid retraction. His thyroid gland
was not palpably enlarged. Serum rolactin concentration was
12 ng/mL (normal<15 ng/mL). Which one of the following
tests should you order next?
Thyroid Stimulating Hormone (TSH)
Visual fields
Luteinizing Hormone (LH)
Follicle Stimulating Hormone (FSH)
Insulin-like growth factor-1 (IGF – 1)
16. Patient F., 57 years old, complains of general weakness,
headache, weight gain, menstrual irregularity, hirsutism,
frequent urination and thirst. Objective status: height - 157 cm,
weight - 92 kg. Localization of fat tissue is disproportional,
being found mostly on the face, neck and trunk with relatively
thin extremities. Face is ruddy, “moon face”. There are dark
194
A.
B.
C.
D.
E.
A.
B.
C.
D.
E.
red striae under her arms, on the inner surfaces of the
hips and on the belly. Skin is normal, moist, pulse - 70
beats/min, blood pressure – 180/100 mmHg. Over her upper lip
and chin large amount of hair growth (patient shaves).
Laboratory tests: 17-OHCS is elevated, oral glucose tolerance
test (OGTT) – 6.2-10.8-9.4 mmol/l, glucose in urine – 0.5%.
What is the most likely diagnosis?
Diabetes mellitus
Arterial hypertension
Itsenko-Cushing syndrome, steroid diabetes
Obesity
Steroid diabetes
17. A 45 year old man was referred by his dentist for
prognathism and malocclusion. Shoe size had increased from 6
to 8 in the last six years and the patient complained of
excessive sweating, headache, but denied change in sexual
activity.
Objective status: the patient had a prominent jaw,
spacing of the teeth, thick skin and the thyroid was enlarged 2
times normal size, and felt firm and finely nodular. The rest of
the examination was normal except for large, spade-like hands
and feet. Laboratory evaluation revealed: fasting growth
hormone increase 1,5 times normal, fasting glucose was 6,5
mmol/L. The most likely diagnosis is:
Gigantism
Obesity
Acromegaly
Physiologic Growth Hormone variation
Hypothyrosis
18. Patient S., 33 years old, complains of a tiresome thirst
(drinks 8-12 l/day), mouth dryness, frequent urination, weight
loss and headache. In the anamnesis - tree months ago patient
195
A.
B.
C.
D.
E.
A.
B.
C.
D.
E.
suffered a craniocerebral injury. Objective status:
height – 174 cm, weight – 53 kg, skin is dry, turgor decreased.
Blood pressure – 110/60 mmHg, pulse –70 beats/min. Result of
laboratory tests: specific gravity 1002-1004, oral glucose
tolerance test – 4.7-7.1-5.5 mmol/l. X-ray of the cranium –
normal, eye fields – normal. What diagnosis would you
suggest?
Diabetes mellitus
Diabetes insipidus
Panhypopituitarism
Sheehan’s syndrome
Hypothyrosis
19. Patient Y., 34 years old, complains of absence of
libido, dizziness, feeling of cold, constipation, frequent
urination, paresthesias, worsening of eyesight and memory,
increased disability, thirst with 2-4 l/day of fluids being
consumed. The disease commenced approximately 2 years
previously, with an ophthalmological consultation because of a
narrowing of eye fields and a consequent diagnosis of pituitary
tumor. Patient has under gone a radiation therapy to ablate the
tumor. Objective status: height – 168 cm, weight – 57 kg, skin
is dry and cool. Blood pressure – 85/50, pulse – 56 beats/min.
17-OHCS is decreased, T3, T4, TSH, LH levels are decreased.
What is the most likely diagnosis?
Hypothyrosis
Diabetes insipidus
Panhypopituitarism
Addison’s disorder
Pituitary tumor
20. A patient R., 54 year old complains of headache,
increased sweating, periodical palpitation, elevation of blood
pressure, severe weight gain (30 kg in 3 years), sexual failure.
196
A.
B.
C.
D.
E.
The patient considers himself to be ill for 3 years. He
connects his disease with having craniocerebral trauma.
Objective status: height is 187 cm, weight is 128 kg. Body
mass index 42 kg/m2. There are numerous pink thin stria on the
hips and abdomen, the skin is highly wet. The left heart border
is displaced to 2 cm laterally. Cardiac sounds are dull, there is
an accent of the second heart sound above the aorta, blood
pressure is 180/110 mmHg on the left arm and 160/100mmHg on the right arm. Lungs are without pathology. Abdomen is
enlarged because of adipose tissue, painless. Liver is near the
border rib's are. Thyroid gland is not enlarged. What is the
clinical diagnosis?
Hypothalamic obesity, class III
Cushing disease, obesity, class II
Alimentary constitutive obesity, class II
Obesity class III
Arterial hypertension
21. A 53 year old patient who is suffering from diabetes
melitus, after acute cerebral blood circulation disorders feels
thirst, dryness in the oral cavity, polyuria to 10-12 litres/ day.
Blood glucose during a day is 4,6-6,9-6,0 mmol/litre, urine
glucose is absent, relative density of urine is 1002 c.u. Such
changes can testify:
a. decompensation of diabetes
b. psychical disorders
c. autonomous nueropathy of urinary bladder
d. joining of unsaccharine diabetes
e. joining of pyelonephritis
22.A patient of 24 years old who had a stress has thirst,
dryness in the oral cavity, polyuria to 10-12 litres/day. Glucose
of blood during a day is 4,6-6,9-6,0 mmol/litre, urine glucose is
absent, relative density of urine – 1002 c.u. thirst, dryness in
197
the oral cavity appeared, polyuria to 10-12 litres/day.
relative density of urine is 1002-1003 c.u. This symptomatics is
conditioned:
a. by insufficiency of aldosterone
b. by insufficiency of vasopressin
c. by insufficiency of prolactin
d. by insufficiency of thyrotropin
e. by insufficiency of adrenalin
23.Hypersecretion of somatotropic hormone in adult
age, besides acromegaly development, often leads to:
a. hypocorticism
b. hyperaldosteronism
c. saccharine diabetes
d. pheochromocytoma
e. insulinoma
24.Galactorrhea appearence requires differential
diagnostics among:
a. prolactinoma and polycystosis of ovaries
b. prolactinoma and menopause
c. prolactinoma and aldosteroma
d. prolactinoma and endometriosis
e. prolactinoma and hypothyreosis
25.For a 38 year old patient with the syndrome of
constant galactorrhea-amenorrhea
the preasence of adenomas of hypophysis was
established. Conturs of the Turkish saddle are not changed,
intracranial hypertension is absent. Preparation for
treatment of this patient is:
a. bromcriptine
b. bromcamphora
c. bromelain
d. bromantan
198
e. bromhexine
26.The next dynamics of STH in reply to the oral
loading of glucose 1,75 grammes/kg will be characteristic for
acromegaly:
a. decline of STH
b. absence of changes
c. increase of STH
d. the level of STH is not estimated
e. all mentioned above
27.A diagnostic criterion of hyperlipidemia under the
metabolic syndrome X is:
a. level of blood triglycerides is below 1,7 mmol/l
b. level of blood triglycerides is below 2,2 mmol/l
c. level of blood triglycerides is higher 1,7 mmol/l
d. level of blood triglycerides is higher 2,2 mmol/l
e. level of blood triglycerides is higher 3,5 mmol/l
28.Adiposity is determined as abdominal when the
circumference of waist is:
a. for men more than 102 cm, women more than 88 cm
b. for men more than 90 cm, women more than 70 cm
c. for men more than 88 cm, women more than 102 cm
d. for men more than 80 cm, women more than 60 cm
e. for men more than 116 cm, women more than 102 cm
29.The piculiarity of metabolic syndrome for women is:
a. more expressed adiposity
b. absence of hypertension
c. henoidal adiposity
d. inclination to suicide
e. The symptomatics is more closely connected with
menopause than with adiposity
30.The dense knot in the thyroid gland of the 16 year
old boy was found. The size of the knot is 10 mm. Local
199
lymphatic nodes are not increased. The function of
thyroid gland is not disturbed. Choose the tactics of treatment.
a. prescribing of mercazolyl
b. prescribing of propiltiouracyl
c. prescribing of iodide potassium
d. prescribing of thyroxyne
e. surgical treatment
31.The development of papillary cancer of the thyroid
gland is mainly connected with:
a. radiative irradiation
b. hypothyrosis
c. thyrotoxicosis
d. iodine deficit
e. iodine surplus
32.The development of medullary cancer of the thyroid
gland is mainly connected with :
a.radiative irradiation
b. hypothyrosis
c. thyrotoxicosis
d. iodine deficit
e. iodine surplus
33.The advantage of a thin-neeled aspirative biopsy of
the thyroid gland under the USD control can be everything
except:
a. possibilities of gland’s function estimation
b. possibilities of making the biopsy when the size of
knots is 10 mm and less under the condition of their palpation
c. economical justification
d. the absence of risk for the patient
e. possibility of mastering the method
34.The following factors:
a. presence of local lymphadenopathy
200
b. clear contours of the knot with
hydrophyl rim
c. head and neck irradiation in the anamnesis
d. rapid rates of knot growth
e. age up to 20 do not belong to those which specify
the high probability of thyroid gland cancer.
35.The probability of thyroid gland cancer is high if it is
discovered that the patient has:
a. multinodal goitre
b. fluctuant and soft nodes
c. nodal form of Khashimoto thyroiditis
d. a solitary node with clear contours
e. a solitary node with washed out contours
36.The malignant formation of thyroid gland during
histological research of punctation is shown by:
a. presence of plenty of watery colloid
b. multinuclear cells
c. preasence of macrofages with foamy cytoplasm
d. cyst or cystic degeneration of the node
e. presence of lymphocytes
37.For the histological investigation of medullary
cancer the following is not characteristic:
a. presence to the amyloid
b. the positive coloration for calcetonine
c. fusiform cells with eccentric nuclei
d. intranuclear includings
e. presence of colloid in plenty amount
38.To the methods of thyroid gland visualization do not
belong to:
a. ultra-sound research
b. proton-emission tomography
c. computer tomography
201
d. magneto-resonant tomography
e. punctional biopsy
39.To the malignant diseases of thyroid gland belong to
a. degenerated colloid node
b. diffuse untoxic goitre
c. multinodal goitre
d. adenocarcinoma
e. colloid cyst
40. Disorders of carbohydrate metabolism in patients
with acromegaly can be conditioned:
A. Contrainsular influence of growth hormone
B. Decreasing level of half-life of insulin
C. Disorders of insular apparatus function
D. Activation of sympathoadrenal system
E. Increasing of estrogens secretion
The right answer : A
41. What is the typical picture of data of adrenals CT in
patients with Cushing’s disease:
A. Both adrenals are enlarged, regular shape
B. One adrenal gland is enlarged, unregular shape,
others is diminished
C. One adrenal gland is enlarged,unregular shape,
others has normal size
D. Both adrenals are diminished, regular shape
E. Both adrenals aren’t changed
The right answer : A
42. At Which is mental development in patients with
hypophysial nanism, as a rule?
A. Normal
B. Slowed
C. Accelerated
D. It is slowed in puberty
202
E. It is slowed in babyhood
The right answer : A
43. Patient, 37years, complains about thirst, polyuria (to
10l a day), weakness. He relates a disease with carried cranialcerebral trauma. Pathology of internal organs was not detected.
Laboratory data: blood count - without pathology; urine
analysis: relative density - 1001, protein - absent, sugar absent, leukocytes - 2-3 in area vision. Previous diagnosis?
A. Diabetes insipidus
B. Acute renal failure, stage of polyuria
C. Diabetes mellitus
D. Primary hyperaldosteronism
E. Chronic glomerulonephritis
The right answer: A.
44. Man, 48 years, pitman, appealed to the doctor with
progressing headache, changes of appearance. In anamnesis chronic tonsillitis. He practises upon an alcohol, tobacco.
Objectively: height - 178cm, weight - 92kg. There are enlarged
superciliary arcs, nose, ears, tongue. Prognatism. There are
enlarged hands and feet. A skin is thickened, greasy. Liver
+2cm. Stagnant optic papilla. Fasting glycemia - 7,2 mmol/l.
What the most credible reason of development of this state?
A. Tumour of brain
B. Chronic infection
C. Abuse by an alcohol
D. Terms of labour
E. Chronic overeating
The right answer: A.
45. Woman, 32 years, complains about a weakness,
apathy, sensitiveness to the cold, amenorrhea, which are
developed for a year after the births complicated by the strong
bleeding. Objectively: height 165cm. Weight 54kg. Pulse -
203
56/min. BP -90/50mmHg. A skin is pale, dry. Fasting
glycemia - 3,3 mmol/l. What complication of sharp bleeding is
the most probably?
A. Pituitary insufficiency
B. Primary insufficiency of the adrenals
C. Primary hypogonadism
D. Hypothalamic syndrome
E. Adenoma of pituitary
The right answer: A.
46. Man, 19 years, hospitalized with complaints about
polydipsia, polyuria - to 9 litres a day, weakness, anorexia. The
first symptoms of disease appeared after the carried flu 2 weeks
later. Glycemia - 4,2 mmol/l. What additional examination
must be conducted for definition of clinical diagnosis?
A. Analysis of urine by Zimnitsky
B. Common analysis of urine
C. Analysis of urine on acetone
D. Analysis of urine by Nechiporenko
E. Blood count
The right answer: A.
47. Patient (woman), 24 years, put of the weight on 7kg
during last year, she has frequent headaches, general weakness.
Objectively: accumulation of fatty tissue is more expressed on
a neck, thorax, abdomen. There are red strias on the lateral
surfaces of abdomen. Menses absent, there is hairiness to
masculine type. BP - 160/100mmHg, HR - 92/min. Additional
data: X-ray-examination of cranium - expressed osteoporosis,
extended sella turcica. Clinical diagnosis?
A. Cushing's disease
B. Cushing's syndrome
C. Pubertal dyspituitarism
D. Primary hypoparathyrosis
204
E. Stein-Levental's syndrome
The right answer: A.
48. Patient, 38 years, complains about permanent
headache, thirst, promoted fatigue, increase BP and gain in
weight (46 kg during 3 years). The disease is related with flu.
Objectively: height - 176cm, weight -143kg, distributing of
fatty tissue: mainly on trunk, thighs, abdomen, red strias. BP 180/100mmHg. Your previous diagnosis?
A. Cushing's disease
B. Hypothyroidism
C. Hypothalamic syndrome, neuroendocrine form
D. Cerebral obesity, III degree
E. Alimentary-constitutional obesity, ІІІ degree
The right answer: A.
49. Patient, 25 years, appealed to endocrinologist with
complaints about problems in sexual life (violation of potency,
infertility). Objectively: eunuchoid type of constitution, height
191cm, weight 78 kg, sense of glandular tissue is present
palpatory. Sexual organs are formed by masculine type.
Testicles are packed, diminished in a size. Sexual chromatin
32%. Cariotype 47ХХY/46ХY. Specify a diagnosis?
A. Kleinfelter's syndrome
B. Clean dysgenesia of gonads
C. Terner's syndrome
D. Primary hypogonadism
E. Mayer-Rokitansky-Kustner's syndrome
The right answer: A.
50. What from clinical signs is not typical for the
Conn’s syndrome?
A. Hyperkaliemia
B. Muscular weakness
C. Alkaline reaction of urine
205
D. Hypokaliemia
E. Hypernatriemia
The right answer : A
51. When during days is it better to prescribe
glucocorticoid therapy in patients with Addison’s disease?
A. In the morning after sleep
B. From 9 to 12 o'clock a.m.
C. Before every use of meal
D. After dinner
E. Before sleep
The right answer : A
52. What influence has catecholamines on metabolism
of sodium in organism?
A. Increasing of reabsorption
B. Increasing of excretion
C. Decreasing of rabsorption
D. Without influence on metabolism of sodium
E. B and C are true
The right answer : A
53. A 27 years old man, has height 121 cm. Complaints
are absent. Apparently: body parts are proportional, pathology
of the internal organs is absent. What can be the reason of he is
being small?
А. *Pituitary pathology,
В. Hypothalamus pathology,
С. Thyroid gland pathology,
Д. Genetic disorders,
Е. Normal variant.
54. The patient of 34 years shows complaints on
polydipsia (drinks more than 5 l of a liquid a day), pollakiuria,
a polyuria, fatigue, a weight loss. Has transferred
neuroinfection. Dryness of integuments, a mucous oral cavity.
206
Urine: the density - 1004, is not present some fiber,
sugar, leukocytes - 3-5 in the field зр., эр. - 0-1 in the field зр.
Creatinin - 0,13 mmol/l. Specify the most probable
pathogenetic mechanism of the given
A. *Decrease distal reabsorption waters at a lack
antidiuretic hormone.
B. Decrease proximal reabsorption waters at a lack
antidiuretic hormone.
C. Osmotic diuresis.
D. Increase glomerular filtration because of polydipsia
E. Decrease distal reabsorption waters owing to an
atrophy canal epithelium.
207
Навчальне видання
Методичні вказівки
до практичних занять
із дисципліни «Ендокринологія»
для іноземних студентів спеціальності
222 «Медицина» денної форми навчання
(Англійською мовою)
Відповідальний за випуск Н. В. Деміхова
Редактор О. О. Кучмій
Комп’ютерне верстання Н. В. Деміхової
Формат 60х84/16. Ум. друк. арк. 12,09. Обл.-вид. арк. 10,67.
Видавець і виготовлювач
Сумський державний університет,
вул. Римського-Корсакова, 2, м. Суми, 40007
Свідоцтво суб’єкта видавничої справи ДК № 3062 від 17.12.2007.
208